Jump to content

Wikipedia:Reference desk/all

From Wikipedia, the free encyclopedia
Wikipedia Reference Desk – All recent questions
 
Shortcut:
WP:RD/ALL
WP:RD/ALL redirects here. You may also be looking for Wikipedia:Resolving disputes, Wikipedia:Redirect or Wikipedia:Deletion review.

This page lists all the recent questions asked on the Wikipedia reference desk by category. To ask a new question, please select one of the categories below. To answer a question, click on the "edit" link beside the question.

For information on any topic, choose a category for your question:

Computing reference desk
Science reference desk
Mathematics reference desk
Humanities reference desk
Computers and IT Science Mathematics Humanities
Computing, information technology, electronics, software and hardware Biology, chemistry, physics, medicine, geology, engineering and technology Mathematics, geometry, probability, and statistics History, politics, literature, religion, philosophy, law, finance, economics, art, and society
Language reference desk
Entertainment reference desk
Miscellaneous reference desk
Reference desk archives
Language Entertainment Miscellaneous Archives
Spelling, grammar, word etymology, linguistics, language usage, and requesting translations Sports, popular culture, movies, music, video games, and TV shows Subjects that don't fit in any of the other categories Old questions are archived daily
Help desk
Village pump
Help desk Village pump
Ask general questions about using Wikipedia Ask about specific policies and operations of Wikipedia
Help manual MediaWiki handbook Citing Wikipedia Resolving disputes Virtual classroom
Information and instructions on every aspect of Wikipedia Information about the software that runs Wikipedia How to cite Wikipedia as a reference For resolving issues between users An advanced guide on everything Wikipedia

Computing

[edit]

September 18

[edit]

Android move file

[edit]

I have a file in Downloads that I need to move to a folder used by the relevant app. However samsung's(?) "my files" app doesn't appear able to navigate to that folder.

How do I move the file using the phone itself and WITHOUT using a connected PC? -- 2A00:23CC:D222:4701:9DAE:549F:77B9:C1A3 (talk) 18:39, 18 September 2024 (UTC)[reply]

Do you see something called "internal storage"? The folder you want may be in there. If not, a different file manager may help.  Card Zero  (talk) 19:20, 18 September 2024 (UTC)[reply]
"My Files" doesn't defaultedly list the desired destination, even though it lists internal storage.. -- SGBailey (talk) 21:35, 18 September 2024 (UTC) (I post logged on on PC and logged off on phone.)[reply]
try Ghost Commander. I can get to /storage/emulated/0/Download through there. Some other open source ones here Komonzia (talk) 00:09, 21 September 2024 (UTC)[reply]

September 20

[edit]

Windows Encrypted Drive

[edit]

When Windows automatically encrypts the Windows partition on a laptop drive, my research makes me think it uses Bitlocker by default. Assuming that is true, where is the key it uses to decrypt? Is it elsewhere on the hard drive? Is it in the BIOS? Is it on a device somewhere in the computer? I am thinking of this scenario: A laptop is run over and crushed. The hard drive still works. You put it in another computer to get the files off it. How does Windows know how to unlock the drive? 75.136.148.8 (talk) 18:47, 20 September 2024 (UTC)[reply]

ask google 69.181.17.113 (talk) 20:06, 20 September 2024 (UTC)[reply]
It seems it's stored in the Trusted Platform Module, so in fact Windows wouldn't know how to unlock the drive. Having said that, BitLocker#Encryption_modes mentions some alternatives, such as a USB key, a password, or a recovery key.  Card Zero  (talk) 20:08, 20 September 2024 (UTC)[reply]
So, am I correct in assuming that you do not want your hard drive to automatically be "bitlockered" because if anything happens to your computer, your files are encrypted and basically lost. 75.136.148.8 (talk) 21:49, 20 September 2024 (UTC)[reply]
I don't know what the default mode is, but I get the impression that it's usual to opt to keep a recovery key. The 2024 Crowdstrike incident was exacerbated by the difficulty of typing these long keys into herds of machines that wouldn't boot (and thus couldn't unlock themselves).  Card Zero  (talk) 22:13, 20 September 2024 (UTC)[reply]
Have you seen this document? manya (talk) 09:18, 23 September 2024 (UTC)[reply]

September 24

[edit]

Retrieving data from the wayback machine.

[edit]

I need to know how I can grab for my own leisurely research the last known full version of a website no longer online but archived at the waybackmachine. I've unfortunately been away from a PC for 6 or 7 years and I'm lost as a fart in a whirlwind. I was last using win7 and now trying to learn 11. Help a techno peasant out? TIA! Brad (talk) 20:28, 24 September 2024 (UTC)[reply]

Brad101: I find the Wayback Machine browser extensions useful for this. They're listed at https://web.archive.org/ under "Tools". Visiting any website with one of these extensions active will enable searching and visiting archived versions if they are available. — ClaudineChionh (she/her · talk · contribs · email) 00:26, 25 September 2024 (UTC)[reply]
If there is no extension for your favourite browser, you
can prepend  https://web.archive.org/*/ to the url of an archived web page. For example, visiting
https://web.archive.org/*/https://en.wikipedia.org/wiki/Wikipedia:Reference_desk/Computing
allows you to find all archived versions of this page, from August 29, 2006, up to the p[resent.  --Lambiam 14:51, 25 September 2024 (UTC)[reply]
To clarify... I want to bring all of the archived data down to my local machine. I need to know how to do that. Thanks. Brad (talk) 20:01, 25 September 2024 (UTC)[reply]
https://superuser.com/questions/828907/how-to-download-a-website-from-the-archive-org-wayback-machine has several answers from various years. They may be too complex for you. --Error (talk) 23:20, 25 September 2024 (UTC)[reply]

September 25

[edit]

Bedroom wall mounted entertainment system

[edit]

We haven't had a TV in years and don't want one; instead we view streaming services on computers and Android devices. We are going to renovate a bedroom and would like to watch these services in bed on a wall-mounted monitor. What hardware and type of device makes sense? It couldn't be a touch-screen. I guess we could manage to use a mouse. Could that let us access an on-screen keyboard? Should this be computer-based or Android? We would want to be able to turn it on and off remotely. (We have laptop computers but don't want to involve them for this.) Do I need to provide any further information? Hayttom (talk) 14:47, 25 September 2024 (UTC)[reply]

So, a Smart TV? Perhaps you'd prefer some kind of free software smart TV platform to avoid possible vendor lock-in.  Card Zero  (talk) 15:51, 25 September 2024 (UTC)[reply]
Aha, exactly, a Smart TV. I should have guessed. Thanks for the tip about vendor lock-in. Also, having read the article, I think I want one without a camera. (Or I'll cover it up.) Hayttom (talk) 16:07, 25 September 2024 (UTC) [reply]
Resolved
Let's not be hasty, there are many horror stories about security holes and unblockable adverts: Smart TVs sending sensitive user data to Netflix and Facebook (2019), You watch TV. Your TV watches back (2019), I spent $3,000 on a Samsung Smart TV and all I got were ads and unwanted content ... meanwhile, it's possible, with I think practically any TV you buy these days, to screencast from an android tablet to your wall-mounted screen. In which case what you really want is the dumbest TV possible that will accept wireless screen echoing.  Card Zero  (talk) 16:16, 25 September 2024 (UTC)[reply]
With this screencasting, would I get the maximum resolution on the dumb TV? Hayttom (talk) 17:48, 25 September 2024 (UTC)[reply]
That is an issue, yes. I don't know. We also have a Home theater PC article, and it's worth noting that small bluetooth keyboards exist and can be used with Android, and I think with smart TVs, in case that ends up being relevant. Another frequent complaint I see about smart TVs is that the interface may be slow, awkward, confusing, or filled with auto-playing adverts. Some people use a gaming console for streaming TV instead. I have a vague notion that denying your new TV its own network access (not telling it the wifi password), while connecting it physically to some other small device, such as a small computer, controlled remotely, might work out better. (This might be more straightforward than obtaining one of the increasingly rare not-smart TVs, although Sceptre is apparently a manufacturer of that kind.) As another editor mentioned, you then have potentially burdensome effort when turning the whole contraption on, and launching software and telling it to connect - or the effort of working out how to automate this. Fan noise may also be an issue if the device is powerful.  Card Zero  (talk) 18:19, 25 September 2024 (UTC)[reply]
You mention it being a bedroom wall. I don't have a TV in my bedroom, but I do have one in my living room mounted on a wall. Behind that wall is a long closet. I ran everything through the wall. The power cable. The antenna cable. The internet cable. The game system cables. Etc... The only thing visible in the living room is the television. I feel that is what you would want in a bedroom. You don't want to see cables and electronic components cluttering up the space. So, while remodeling, think about what is behind the wall. Can you run everything on the other side? As for a smart TV, it comes with a very easy to use remote. You don't have to have a keyboard and mouse next to you run it. Also, you can cast most phone displays to most smart TVs. So, you can watch on your phone and then send the display to the TV when you are ready. If you used a computer, you will lose the simplicity of a remote control and the ability to cast your display from your phone unless you get some rather complicated devices and programs running as well. 75.136.148.8 (talk) 17:15, 25 September 2024 (UTC)[reply]
Already planned all of that, and already mentioned I don't want to use my computer, but thanks. Hayttom (talk) 17:48, 25 September 2024 (UTC)[reply]
Smart TVs are unlikely to have software updates for all the years the hardware will work. There can be problems with the user interface, security and others, as they are actually computers managed by an external company. I have a bookmark to a now lost tweet mentioning LG commercial panels that are used for hotels and displays and have not TV tuner or apps. Searching for "commercial panel" gives me solar panels. It seems that LG calls them commercial displays. I guess that other brands offer similar hardware. With them you will need a home theater PC as Card Zero said. There are commercial solutions or you could search for free software solutions based on Raspberry Pi or similar hardware. Check how its 10-foot user interface is since you will use it a lot. Category:Streaming media systems may have useful articles. Since you probably watch commercial streaming, you may be limited by what operating system the official applications for your streaming service run on. --Error (talk) 23:15, 25 September 2024 (UTC)[reply]
Some research on smart TVs and privacy from a couple of weeks ago: apparently they send screenshots of anything shown on the screen when connected to a device by HDMI, but not when mirroring a phone. Also you can opt out, but it's an involved process, with six settings to change on a Samsung TV and eleven on an LG TV.  Card Zero  (talk) 09:21, 27 September 2024 (UTC)[reply]

October 1

[edit]

Science

[edit]

September 18

[edit]

Is there any physical theory, claiming that every elementary particle can turn into some other elementary particle?

[edit]

HOTmag (talk) 13:05, 18 September 2024 (UTC)[reply]

Quarks are the only known particles whose electric charges are not integer multiples of the elementary charge. Therefore, in physical theories that accept both the Standard Model and the law of charge conservation, a quark cannot turn into another particle but a quark. But the types of quarks all have different masses, so all such quark–quark changes violate the law of conservation of mass.  --Lambiam 17:57, 18 September 2024 (UTC)[reply]
If you are referring to a single elementary particle, so why didn't you mention the electron, besides the quark?
If that's because an electron colliding with a positron turns (together with the positron) into a pair of photons, then also a quark colliding with an anti-quark turns (together with the anti-qurk) into a pair of gluons.
Anyway, in my question I allow a given elementary particle to collide with its anti-matter for becoming another elementary particle.
More important: My question is theoretical, so it's not only about known particles, but rather about all possible particles, including those which haven't been discovered yet. HOTmag (talk) 18:21, 18 September 2024 (UTC)[reply]
If someone claims all swans are white, it suffices to debunk the claim by finding one purple swan. Maybe there also blue, brown or black swans, but it is not necessary to search for further counterexamples. Likewise, if some physical theory claims every elementary particle can turn into some other elementary particle, it suffices to debunk the claim by finding just one elementary particle that cannot turn into some other elementary particle. I just started with the top line of File:Standard Model of Elementary Particles.svg. There may be many other counterexamples (like the Higgs boson), but why bother to keep searching?
The as of yet undiscovered bunkon and trashon, whose properties are still unknown except that they are postulated to be different elementary particles, can turn into each other. A difficulty in finding them is that their properties are unknown, so experimental physicists don't know where to look. There may be many more such pairs, which may never be discovered.  --Lambiam 10:17, 19 September 2024 (UTC)[reply]
By mistake, I thought you meant the quark was the only particle that couldn't turn into another particle but a quark, but now I see this was not what you meant, so I take my first sentence back.
However, I still emphasize that my question allows a given elementary particle to collide with its anti-matter for becoming another elementary particle.
Re. your senetnce: "The as of yet undiscovered...different elementary particles, can turn into each other": Is there any physical theory claiming what you've claimed in that sentence? Actually, this was my original question... HOTmag (talk) 10:44, 19 September 2024 (UTC)[reply]
I'm pretty sure that that sentence was a joke. Look at the names: BUNKon and TRASHon. --User:Khajidha (talk) (contributions) 11:25, 19 September 2024 (UTC)[reply]
No "maybe" about black swans. They were recorded by Europeans in 1697, possibly earlier. 2A00:23D0:F6F:1001:2D07:A712:8909:7D91 (talk) 11:56, 19 September 2024 (UTC)[reply]
All right. HOTmag (talk) 12:35, 19 September 2024 (UTC)[reply]
See also black swan and black swan theory. -- Jack of Oz [pleasantries] 18:37, 19 September 2024 (UTC)[reply]
OP may have in mind something more like the particles created from particle-antiparticle annihilation, such as those in the chart at Annihilation § Electron–positron annihilation, as opposed to something like the weak decay of quarks.
It seems to me that OP could mean either: 1) a single elementary particle can spontaneously become another single elementary particle (with the help of another particle that remains unchanged), in which case I think the answer may be no for any particle; or 2) for two given particles, there's an interaction in some condition where it's meaningful to say that one specified particle is in the input, and it becomes in the output the other specified particle [Edit: which may include any number of other particles in the reaction doing anything else]. Not sure (I didn't do particle), but I think (2) might be considered more or less accurate (to the extent the fuzziness of the wording necessarily allows). SamuelRiv (talk) 16:10, 19 September 2024 (UTC)[reply]
I adopt your option 1# if it means Particle decay, and I adopt your option 2# if it means annihilation (i.e. by colliding with the anti-particle).
But contrary to the way you divided you answer: option 1# for all particles, or option 2# for all particles, I didn't exclude a third option which is: option 1# for some particles, and option 2# for the rest of the particles (without excluding particles that satisfy both 1# and 2#).
All agree, that some particles satisfy option 1#, and that some particles satisfy option 2#.
My question is about whether there is any physical theory claiming, that every particle (including any particle that hasn't been discovered yet), satisfies either 1# or 2# (or both). HOTmag (talk) 10:25, 20 September 2024 (UTC)[reply]
I was trying to interpret your question literally. #1 is one single particle becoming one different single particle with everything else unchanged -- this does not happen at all afaik, nor in general in theory (User:Lambian gives a simple example for quarks in their answer above). #2, the way I worded it, afaik can (and does) happen for all particles in the Standard Model.
What I'm trying to convey with this #1/#2 description is that it's not a particularly meaningful one, if you can claim "every elementary can turn into some other elementary particle" just by comparing one reactant to one product in a complex interaction.
(As maybe a sorta-ok example, consider the chemical reaction of a strong acid + base into salt + water: HCl + NaOH -> NaCl + H2O. Would you say the HCl (reactant) turns into salt? turns into water? Or does it change nothing at all because both the reactants and products are largely remain just free ions in aqueous solution? This is why I'm not sure what you're trying to ask is very meaningful.) SamuelRiv (talk) 17:24, 20 September 2024 (UTC)[reply]
I'm not asking about a single particle becoming one different single particle with everything else unchanged. Let's put it this way: Is there any physical theory claiming that all particles in the dark matter can turn into other particles? (whether by a decay or by annihilation or by any other way). HOTmag (talk) 01:30, 22 September 2024 (UTC)[reply]
There can never be such a theory. Why? Dark matter is named so because it never interacts with ordinary matter, except by gravity. As soon as it interacts in any other way it's not dark matter anymore but a new kind of ordinary matter. By interaction by gravity no distinction of particles is possible, only sum and distribution of mass is measurable. So dark matter can decay all it want, no human physicists are able to prove it or disprove it. By definition of the word. It may be that some particles, that we now subsume in dark matter, are later discovered to be not dark. And then we would know the conditions where they participate in the normal decay of ordinary matter. But that can never tell us about the real dark matter, as long as there's real dark matter. 176.0.153.105 (talk) 18:20, 23 September 2024 (UTC)[reply]
Most particles decay into other particles. The only "stable" ones are Electrons, Protons, Photons and (to a degree (see here)) Neutrinos. But even these can (under the right conditions) either combine or "destroy" and "create" with each other. 176.0.165.39 (talk) 12:20, 20 September 2024 (UTC)[reply]
These interactions can all be depicted in a Feynman diagram as lines meeting in a vertex. The lines correspond to particles. In a Feynman diagram, a vertex is always the meeting point of three lines. A particle → particle change would correspond to a Feynman diagram in which just two lines meet in a vertex.  --Lambiam 22:47, 20 September 2024 (UTC)[reply]
Or two lines merging in two vertices that are connected by two lines in the form looking like an eye. 176.0.153.105 (talk) 14:24, 23 September 2024 (UTC)[reply]
Yes, I know that. but you're talking about partciles of the Standard Model, while I'm asking about a theory that claims that all particles, including those which haven't been discovered yet, can decay turn into other particles. HOTmag (talk) 01:30, 22 September 2024 (UTC)[reply]
In your question and first response you talked about a particle that "can turn into some other elementary particle", and people are trying to clarify what that can mean. But now you're asking about decay: "Particle decay" is where only one particle goes in, and some other number of different particles come out, but as others have said there are stable particles that do not decay. (There are of course experimental bounds to what we currently know of this, and there are interesting subtleties in the theory for example why a photon does not decay.)
There's no theory which can claim anything meaningful about all particles that have not been discovered yet. A theory predicts new particles, and the theory becoming successful may lead to building experiments to verify empirically the particles ("discover" them, although a "discovery" was equally done when the theory was written).
You could imagine another type of theory that might say all particle physics theories are really at a fundamental level part of X-theory, and in X-theory everything decays into X-dust in 20 billion years, but that's not a particle theory. SamuelRiv (talk) 05:48, 22 September 2024 (UTC)[reply]
Re. your first paragraph: I'm sorry for not being clear in my recent response. Thanks to your comment, I will make it clearer, as I've alraedy made it in my first post.
Re. your second paragraph: I mean, something like the supersymmetric theory, claiming something about all particles, including those which haven't been discovered yet. So, again, I'm asking whether there's a theory that claims that all particles, including those which haven't been discovered yet, can decay turn into other particles, whether by a decay or by annihilation or by any other way. HOTmag (talk) 12:45, 22 September 2024 (UTC)[reply]

Although the above discussion may imply that the words "hypothesis" and "theory" can be used interchangeably, a scientific hypothesis is not the same as a scientific theory.

An answer is No, there is no scientific theory that just claims anything. A scientific theory offers a generalized explanation of how nature works described in such a way that scientific tests should be able to provide empirical support for it, or empirical contradiction ("falsify") of it.

The OP's question using the word "theory" to mean a claim that at best will remain unproven or speculative actually looks for a Hypothesis meaning an educated guess or thought about something that cannot satisfactorily be explained with the present scientific theories. The OP seems to be thinking aloud[1] a new hypothesis. Philvoids (talk) 10:37, 22 September 2024 (UTC)[reply]

See Supersymmetric theory. It claims something about all particles, including those which haven't been discovered yet, and it's still called a "theory". Anyway, I'm not focusing on terminology but rather on an idea: Is there any theory, or a well known hypothesis, or a well known conjecture, or whatever, claiming that all particles, including those which haven't been discovered yet, can turn into other particles, whether by a decay or by annihilation or by any other way. HOTmag (talk) 12:45, 22 September 2024 (UTC)[reply]
Wikipedia editors are careful about terminology. By "Supersymmetric theory" you link to the article titled Supersymmetry. Its first line clarifies that it refers not to a theory but a theoretical framework. Read further to see how it anticipates what might characterise a supersymmetry theory without specifying any one for attention. Merely calling a supposed bosonic superpartner to the electron a selectron hardly amounts to a falsifiable hypothesis and obviously says nothing about undiscovered particles. Beside our care with terminology, we are not in the business of prediction. Philvoids (talk) 09:17, 23 September 2024 (UTC)[reply]
I've linked to our article [page] Supersymmetric theory, which does exist in Wikipedia, so when I used the term "supersymmetic theory" I used a term used in Wikipedia as well. Indeed, it passes to the article Supersymmetry, but also this article does point out - in its second paragraph - that "Dozens of supersymmetric theories exist", whereas the first paragraph of this article - does point out that the suppersymmetry "proposes that for every known particle, there exists a partner particle with different spin properties. There have been multiple experiments on supersymmetry that have failed to provide evidence that it exists in nature."
Anyway, my original question was not about the terminology used in Wikipedeia, but rather about the very idea, and I allow you to call it: theory, theoretical framework, hypothesis, conjecture, proposal, suggestion, idea, or whatever, but the main idea still remains, as long as I understand you and you understand me (I guess this is the case). HOTmag (talk) 10:07, 23 September 2024 (UTC)[reply]
Anyway your claim "I've linked to our article Supersymmetric theory, which does exist in Wikipedia" is a false claim indeed. Philvoids (talk) 11:48, 26 September 2024 (UTC)[reply]
Try now to click again on Supersymmetric theory, and this time you will see that this article [page] does exist in Wikipedia, giving a link to the other article, but without passing to the other article. Actually the article [page] Supersymmetric theory exists in Wikipedia since 14 January 2006, as you can see in its history page. Generally, every article [page] that passes to another article [page] must exist in Wikipedia: If it hadn't existed, it couldn't have passed to the other article [page]. HOTmag (talk) 22:31, 26 September 2024 (UTC)[reply]
Please see WP:REDIRE. A Wikipedia redirect is a page that automatically sends visitors to another page, usually an article or section of an article. It aids navigation and searching but a redirect page is not itself a Wikipedia article nor is its mere existence a reliable source. The excuse of an honest mistake won't justify an attempt to argue black as white by misrepresenting links. Philvoids (talk) 11:47, 27 September 2024 (UTC)[reply]
My only innocent mistake was my replacing "page" by "article". Anyway, when I wrote "article" I really meant "page", and I was sure this would also be what you would interpret when you read the word "article" in my responses. But since I'm realizing now this was not what you interperted, I'm striking out every "article" and replacing it by "[page]".
As for what you call "a reliable source", I've already quoted the second paragraph of our article Supersymmetry: "Dozens of supersymmetric theories exist". This quote proves that there is no fundamental difference between linking "UK" to "United Kingdom" and linking "Supersymmetric theory" to "Supersymmetry": All of these four terms are legitimate. HOTmag (talk) 12:46, 27 September 2024 (UTC)[reply]
Now you only have to define what you mean by particle. For instance in Standard Model you have the everyday particles (Electron,Proton...). Nothing of that is stable in your definition. Then you have Quark. No quarks are ever single. So it is a question of the neighbouring quarks which reactions are possible. But even then there is no stability in your definition. But never is one particle turned into exactly one other particle. Even if that were possible you only have to look at a different level of abstraction and a group of particles would turn into a different group of particles. 176.0.158.114 (talk) 09:34, 23 September 2024 (UTC)[reply]
By elementary particle I mean what physicists mean by that term: quarks, leptons, gauge bosons, and also elementray particles that haven't been discovered yet, like axions. Anyway, I really meant what you suggested in yout last sentence: "a group of [elementray] particles would turn into a different group of [elementray] particles". HOTmag (talk) 10:07, 23 September 2024 (UTC)[reply]
Then I can give you a definite answer. Any group of ordinary matter particles (whether known or not) can turn into another group of particles if they encounter the right conditions (even if the conditions could not be achieved in a laboratory or somewhere near Earth). For dark matter particles that can not scientifically be said. And never will be possible to say with science. If someone says something about changing about dark matter it is and never will be science. That is part of the definition of the word "dark" in "dark matter". If that definition changes nothing about the future I have written will continue to be valid. 176.0.153.105 (talk) 19:28, 23 September 2024 (UTC)[reply]
Memorandum: My question was about all elementary particles, and by elementary particle I mean what physicists mean by that term: quarks, leptons, gauge bosons, and also elementray particles that haven't been discovered yet, like axions.
To sum up: The question is whether, for every group of elementary particles, including those which haven't been discovered yet, there exist (what you call) "right conditions", under which this group can turn into another group of elementary particles. HOTmag (talk) 13:10, 24 September 2024 (UTC)[reply]
Then as I said all the way before, with my "#2", then if it can be called a "particle", yes you can always make it turn into stuff.
But you should understand what people here are trying to say: this becomes rather meaningless as your understanding of what a theory means, in the sense of the "discovery" of particles, is not very accurate, so when you're trying to force these incompatible notions into your question it makes it difficult to give meaningful answers. (Mine, as I said before, is not particularly meaningful.) SamuelRiv (talk) 13:28, 24 September 2024 (UTC)[reply]
Theories don't necessarily discuss discoveries, and a theory about elementary paricles doesn't necessarily discuss what you call "discovery" of elementary paricles, it can also say something about elementary paricles that haven't been discovered yet, e.g. gravitons, axions, electrinos, gravitinos, axinos, and the like. Anyway, my question is about all elementary particles, including those which haven't been discovered yet. Are you referring to all of them in your first sentence? HOTmag (talk) 18:26, 24 September 2024 (UTC)[reply]
YES! Even for particles that are not discovered yet, the right conditions are possible to determine. For an example see the article about the Axion. Even although the particle is not discovered yet, the right conditions for some way to turn into another group of particles are already known (strong magnetic field). Of course there will probably be other ways but one is enough to answer your question. And there is even a general answer. Most particles will turn at the surface of a Neutron star. If you are in doubt, there will almost always be the right conditions. Is there some particle that does not turn at the surface of a neutron star? Of course the neutron. But that turns at enough distance from the surface. And it is possible that the Alpha particle is a special Quantum state of twelve Quarks. Should that be the case a neutron would even be turned on the surface of the neutron star into an alpha particle. 176.0.147.163 (talk) 13:03, 26 September 2024 (UTC)[reply]
Have you got any source for your claim, that all elementary particles (other than neutrons), including those that haven't been discovered yet, e.g gravitons (and gravitinos and electrinos and axinos and likewise), will turn at the surface of a nuetron star? HOTmag (talk) 22:13, 26 September 2024 (UTC)[reply]
First my claim was for almost all particles, what you acknowledge by excepting the neutron. Second, no I don't have a verifiable source, because it is based on a personal communication. But it is easy to verify. Elementary logic and quantum mechanics are sufficient. Every physicist knows that quantum mechanics and high gravity or strong electric or strong magnetic don't fit together. As soon as a quantum particle interacts with something that needs relativity to describe, the equations begin to change. Of course the particles will change with them. I can give you an example, that does not need an exotic environment. If you read any Wikipedia article about a heavy element you will read about relativistic effects, that affect the electrons. And at the same time the core will go more and more unstable. Sometimes the elements are only "observationally stable". And the more affected elements are really radioactive. Until it comes to U238. There's a rule. Every number of charge has an associated ideal atomic weight. If the weight is too large neutron are not stable anymore and decay. If the weight is too small, coulomb force wins and an alpha particle, the least energetic particle in the core, will be ejected. Now to U238. According to its pattern of decay it's both too heavy and too light. Of course that is relativity playing havoc with the quantum mechanics. And now imagine what will go on where relativity really comes into play. Such as at the surface of a neutron star. 176.0.144.177 (talk) 02:36, 27 September 2024 (UTC)[reply]
PS. The claim about the surface of a neutron star is meant here. For this claim exceptions are possible. The surface of a neutron star is an example for the right conditions and a good bet,but not guaranteed. Further above was another claim about the right conditions. That claim is universally true, because there is always an environment where the wave function can be distorted beyond recognition. Which is the whole point of the claim. 176.0.144.177 (talk) 02:49, 27 September 2024 (UTC)[reply]
I deliberately asked about what you call "exotic environment" (e.g. gravitons, electrinos) because, while your example regarding U238 is about matter quite known to us - experimentally speaking, you're using a kind of generalization, from what we have already encountered (e.g. U238), to what - we haven't indeed encountered yet - but we can assume something about in (your own?) theoretical framework stating that: since (as you begin) "as soon as a quantum particle interacts with something that needs relativity to describe, the equations begin to change", so (as you conclude) "of course the particles will change with them". Of course your conclusion is possible and legitimate, but you haven't proved that it's the only one possible - theoretically speaking - mainly as far as (what you call) "exotic environment" is concerned.
To sum up: It seems like you're using your own theory. This is legitimate of course, but when I asked for a "theory" (see the title), I mainly meant: a (well sourced) theory - although I didn't add this condition in the title (but only in my previous response). HOTmag (talk) 08:49, 27 September 2024 (UTC)[reply]
How I said, there's private communication involved, so it's possible that through misunderstanding a private theory emerged, but the goal was to talk about the normal quantum physics. I never claimed that quantum physics is the only theory where universal decay is possible/mandated. That quantum physics and general relativity are mutually incompatible is well sourced. The example of U238 was only to show that the equations begin to change in the region of both relativity and quantum physics. And that the trend in this regard goes to less stability instead of more. Of course there are exceptions, particles that are normally not stable but can be stabilised by the exotic environment, better the changed equations in the exotic environment. Of course every exotic environment changes the equations in its own way. So for every needed change a matching exotic environment can be (theoretically) constructed. And then there is the well sourced Hawking radiation. If you want a random change an appropriately sized black hole will provide an exotic environment that is guaranteed to destroy any quantum state (well sourced No-hair theorem) and emit another random quantum state. 176.0.167.84 (talk) 00:16, 28 September 2024 (UTC)[reply]
I let you use any (well known) theory, including Quantum physics, for claiming that as soon as a quantum particle interacts with something that needs relativity to describe, the equations begin to change. I only asked whether your conclusion, that if the equations begin to change then the particles will change with them, must be concluded from this (well known) theory.
As for the option of black holes, yes, this is what I'd thought, and I'm asking now a further question in my following thread, regarding this option. HOTmag (talk) 18:14, 28 September 2024 (UTC)[reply]
See Richard Feynman as he talks about Path integrals. If the formula changes, the sum will change. That is clearly mathematics. I don't know if somebody has written about these mathematical relations, because it is only article worthy if there is a whole theory in the conclusion, but that is not how far we know the formulas yet. But even as you begin to try it, you will see immediately that there is change imminent. And the example of U238 shows that the change is not an artefact of the mathematical formulation but is reality. And the ultimate weapon (the black hole) drives the point home. Only the exact relations are unknown today. 176.0.164.155 (talk) 20:24, 28 September 2024 (UTC)[reply]
"If the formula changes, the sum will change". IMO, it's still too obscure, because:
As long as the particles haven't been discovered, we don't know what properties those particles carry, so we don't know how the equations involving those particles look like, so we don't know how those equations are going to change, so we don't know whether this change may influence the issue of whether the particles involved in those changing equations may turn into other particles.
As I've already said, IMO it's still too obscure, as long as we don't know what particles we are talking about. HOTmag (talk) 22:39, 28 September 2024 (UTC)[reply]
Every particle, discovered or not, must obey the Schrödinger equation. Part of the solution of the Schrödinger equation is an Integral. One way to do it is the Path integral formulation. That formulation has the advantage that the Spacetime is part of the explicit input. Relativity Theory modifies space. Modified input results in modified output, according to Mathematics.
That is essentially the whole point of the "theory". 176.0.164.155 (talk) 00:15, 29 September 2024 (UTC)[reply]
"Modified input results in modified output". For claiming that, you must make sure that this equation reflects a one-to-one correspondence between input and output. HOTmag (talk) 01:05, 29 September 2024 (UTC)[reply]
That's not true. In the pure mathematical sense there may be a function where that applies. But never in physics. Even for the Sine and cosine function it's not true, if you really think about it. In the Schrödinger equation there is a double Differential operator. So for the same output you need the same value, the same first differential and the same second differential. And now you can imagine a function that is at least two times differentiable, has at two inputs the same output, has at least a third input with a different output (constant function do not count) and is not (theoretically) reversible in the limits of our universe. 176.0.164.155 (talk) 12:05, 29 September 2024 (UTC)[reply]
You did not prove that the Schrödinger equation reflected a one-to-one correspondence. You have only presented some kind of intuition for claiming that. I still wonder if this intuition is rigorously provable. HOTmag (talk) 17:41, 29 September 2024 (UTC)[reply]
No, I didn't, because it is not true. But I argued (intuitively) that it's not necessary. And some functions (like sine) are theoretically not one-to-one, but really they are,because there are only 111 digits for all values in our universe. And the Wave function has some of the properties of the sine function (because they are both solutions to second derivative differential equations), but here the value is dependent on the location. That means, even when a value at some location does not change, at another location the value will change. 176.0.159.38 (talk) 13:22, 30 September 2024 (UTC)[reply]
You are pointing at some analogy between the Schrödinger equation and the Sine/Cosine, but I suspect I don't even accept what you're claiming about the Sine/Cosine, so how can I figure out your analogy? Let me explain:
While you're claiming "some functions (like sine) are theoretically not one-to-one, but really they are, because there are only 111 digits for all values in our universe", our article Sine and cosine explicitly states: "sine and cosine are not injective", i.e. they are not one-to-one correspondences. This is true, both theoretically and in reality: Here is a concrete example taken from our real universe: so modified input in the Sine function does not necessarily result in modified output - as opposed to what you'd claimed about the Schrödinger equation you'd compared with the Sine function.
Unless you think that zero or pi does not exist in the universe as an exact numeric value, but if so, then no value exists in the universe as an exact numeric value, so all physics may collapse. But it doesn't, while it uses both zero and pi in so many equations, including the well known equations in Quantum physics we're talking about. HOTmag (talk) 19:49, 30 September 2024 (UTC)[reply]
Pi exists in our universe. But not really that value, but rounded to 111 digits. And now you insert that value into your equation. Of course you need to round the results to 111 digits too. I don't know if the sine has not (by random chance) points where both roundings are cancelling each other, but theoretically there shouldn't be such points. The wave function has better behaviour in this case because the number of potential equivalents is finite. In contrast to the sine, where they are infinite. 176.0.159.38 (talk) 00:09, 1 October 2024 (UTC)[reply]
I don't know if the sine has not (by random chance) points where both roundings are cancelling each other. Me either, and this was exactly my point, without any rounding. For being sure there aren't any points like those you're talking about, we have to trust a reliable theory claimig there aren't, and this is the theory mentioned in the title. HOTmag (talk) 08:42, 1 October 2024 (UTC)[reply]
Without any rounding? That is not possible in our universe. And that's the difference between physics and mathematics. 176.0.159.38 (talk) 09:56, 1 October 2024 (UTC)[reply]
When I wrote "without any rounding" I was referring to my exact "point", i.e. to my exact argument, which involved no rounding, i.e. no round corners. HOTmag (talk) 11:18, 1 October 2024 (UTC)[reply]

September 21

[edit]

Variables in chemistry?!

[edit]

I've seen the symbols M and X in chemistry, what do they mean? HAt 04:07, 21 September 2024 (UTC)[reply]

Well M could represent a metal, and X could represent a halogen in a chemical formula, eg NaX could be a halide salt of sodium. X is probably F, Cl, Br or I. Theoretically it could be At, but not in any visible amount. If the symbols are in italic text they may represent a number. Graeme Bartlett (talk) 06:04, 21 September 2024 (UTC)[reply]

September 22

[edit]

Besides a link to our article electroweak interaction that mentions the B-boson without telling anything about it, we have no article about this boson. Why?

I'm eager to know some basic data about it, e.g. mass, electric charge, spin, isospin, stability, experiments trying to detect it, and likewise. But... nothing? HOTmag (talk) 16:27, 22 September 2024 (UTC)[reply]

You'll find it at B meson. Sorry, although the B meson is a boson, it is not the beast that's mentioned in electroweak interaction. --Wrongfilter (talk) 17:23, 22 September 2024 (UTC)[reply]
What a pity. HOTmag (talk) 08:57, 23 September 2024 (UTC)[reply]
The B-boson isn’t a well-known particle in the standard model of particle physics. Are you thinking of the Higgs boson that was discovered in 2012? Philvoids (talk) 08:47, 23 September 2024 (UTC)[reply]
No, I'm looking for basic data (e.g. mass charge spin isospin parity etc.) of the B-boson mentioned in our article electroweak interaction. HOTmag (talk) 08:57, 23 September 2024 (UTC)[reply]
The thing is that the B boson doesn't exist as an entity in our Universe. It "exists" before symmetry breaking, but since our world has a broken symmetry the observable bosons are the Z0 and the photon instead of the W3 and B. As the B cannot be observed, many of your questions are moot. --Wrongfilter (talk) 09:07, 23 September 2024 (UTC)[reply]
Got it. HOTmag (talk) 09:31, 23 September 2024 (UTC)[reply]


September 24

[edit]

Cloning the Carolina parakeet

[edit]

In theory, would it be possible to clone the extinct Carolina parakeet by taking whatever existing DNA we have of the species and filling any gaps with DNA from the sun conure (closest living relative and very common pet) and then putting that into a sun conure egg? I have read that it's very difficult to clone birds though. 146.200.126.178 (talk) 23:12, 24 September 2024 (UTC)[reply]

Here's an Audubon Magazine write up on the topic. It foolishly says that the Carolina parakeet, ivory-billed woodpecker, and passenger pigeon could be invasive, which is nonsense; only the Carolina parakeet could conceivably be successful beyond its former range. See monk parakeet for why. Abductive (reasoning) 23:55, 24 September 2024 (UTC)[reply]
Yes, there's been a lot of talk about removing monk parakeets from areas where they are non-native. But the public tend to get extremely angry about the idea of culling parrots specifically, even to the point of taking direct action when nests are removed. I believe there was even a politician a few years ago who said something along the lines of "absolutely not" when it came to the idea of removing that species being raised by his advisors. 146.200.126.178 (talk) 00:19, 25 September 2024 (UTC)[reply]
Would you then get an actual Carolina parakeet, or merely a hybrid that kind of looks like it? ←Baseball Bugs What's up, Doc? carrots01:30, 25 September 2024 (UTC)[reply]
Yes, I was thinking that myself. How much of the Carolina parakeet's DNA can be replaced by another (very closely related) species before it can no longer be considered a Carolina parakeet? 146.200.126.178 (talk) 01:42, 25 September 2024 (UTC)[reply]
Has the genome of any of these extinct species been mapped? I don't think the article says. But that would be a way to maybe get closer to a real clone. And I understand what they mean by invasive. If a species wasn't someplace, and then appears in that place, by definition it's invasive. That doesn't necessarily mean it will be harmful to other species in that place, but it could be. ←Baseball Bugs What's up, Doc? carrots07:18, 25 September 2024 (UTC)[reply]
See Ship of Theseus. -- Jack of Oz [pleasantries] 17:41, 25 September 2024 (UTC)[reply]
"Reconstructed", bit by bit. In the case of an organism, some careful selective breeding could eventually render the species closer to the original. ←Baseball Bugs What's up, Doc? carrots00:55, 26 September 2024 (UTC)[reply]
We have an article on the concept De-extinction. It mentions the passenger pigeon example discussed in Abductive's link. It also briefly mentions the Carolina parakeet and claims a "full genome of the carolina parakeet was sequenced" in 2019 although I'd suggest reading the sources as I suspect this isn't a complete genome. Revive & Restore seem to be the ones most involved in this for birds or at least the ones who seemed to have received the most publicity. (Colossal Biosciences is also popular but although they did make noises about the dodo they seem more concentrated on mammalian species. And while I personally think anyone involved in this is fairly PR oriented, Colossal Biosciences chosen projects seem to be more flash.) At the moment, in terms of actual "de-extincting" species they seem to be concentrating on the above mentioned passenger pigeon which is their flagship project with also some work on the heath hen. Interesting they've set a goal of the "first generation of new Passenger Pigeons before 2025" but considering their progress page [2] hasn't been updated since 2019, I have doubts we're going to see this first generation in the next 3 months. In fact, even their 2023 report [3] doesn't seem to mention the passenger pigeon. With the heath hen project they at least seem to have been less optimistic about promising stuff by certain dates [4]. However that too doesn't seem to have had an updated since 2020 [5]. Possibly COVID-19 related disruption to their work hasn't helped, still it's been a while since it should have been a barrier to any lab work etc. BTW, I'd say Revival of the woolly mammoth and Thylacine#Research are perhaps the projects which have generally received the most historic interest so might be a good place to look beyond general discussions about de-extinction about the feasibility of such proposals and whether the result if we did succeed should really be compared with the original species. Although the research related to the Maclear's rat mention in our article might be another recent place to look in to. Nil Einne (talk) 10:09, 27 September 2024 (UTC)[reply]

September 25

[edit]

Etymology of wheldone

[edit]

What is the etymology of wheldone? The references doi:10.1021/acs.orglett.0c00219 and doi:10.1021/acs.jnatprod.4c00649 only described the chemical as a fungal metabolite isolated from the coculture of Aspergillus fischeri and Xylaria flabelliformis. --Leiem (talk) 09:00, 25 September 2024 (UTC)[reply]

The clue is in the acknowledgments section of the first paper: "This Letter is dedicated to the late Daniel (Dan) Clive Wheldon". And since Wikipedia has everything: Dan Wheldon. --Wrongfilter (talk) 09:57, 25 September 2024 (UTC)[reply]
I understand. Thank you! --Leiem (talk) 15:38, 25 September 2024 (UTC)[reply]
One might even say...well done. :P SnowRise let's rap 22:39, 25 September 2024 (UTC) [reply]

Botanical nuts

[edit]

I'm trying to split Nut (fruit) back into separate articles for food and fruit (because currently a lot of the article content is about things that aren't botanical nuts), but there's not much left after you strip out the food content. Are there any botanists in the house who can recommend sources on botanical nuts and/or weigh in on what should be covered and isn't? AlmostReadytoFly (talk) 10:10, 25 September 2024 (UTC)[reply]

Not a botanist, but I think there's a bit of a problem for having a botanical nut. It is an even fuzzier idea than that of tree. But there's an article on trees so I suppose the could be one like that on nuts. You'd have to decide whether it is a very resticted topic on the botanical idea of a nut which doesn't contain most of what people mean by a nut or a wider one like the article on trees. Would you include pine nuts, coconuts peanuts, and the stones of drupes like almonds or pecans or plums? NadVolum (talk) 14:33, 25 September 2024 (UTC)[reply]
The plan is for them to go back to Nut_(food) as part of the unmerge. The unmerge discussion is here. AlmostReadytoFly (talk) 08:51, 26 September 2024 (UTC)[reply]

Measure relative UVB / UVC intensity

[edit]

For SODIS water disinfection I'm interested in testing how much UVB and UVC passes through various types of plastic and other materials. I don't care so much about absolute levels as relative intensity compared to unfiltered sunlight. I'd also like to check out whether my cheap sunglasses are letting any UV through. I see this device says peak spectral response 300-350mm which is describes as UVA (I thought that was the A-B range). Is that sufficient, i.e. is something that transits UVA unlikely to block UVB/UVC? Thanks. 2601:644:8581:75B0:0:0:0:C813 (talk) 22:21, 25 September 2024 (UTC)[reply]

I doubt that it's any indication one way or the other, different materials can have widely different transmission spectra both within and outside the visible band; one would have to test each one specifically (if references aren't available). {The poster formerly known as 87.81.230.195} 94.1.171.3 (talk) 02:20, 26 September 2024 (UTC)[reply]

September 26

[edit]

Body Roundness Index formula

[edit]

Body_roundness_index#Calculation gives the formula

364.2 − 365.5  × √(1 − [waist circumference in cm / 2π]2  /  [0.5 × height in cm]2)

which seems unnecessarily complicated. Why isn't it simply simplified to

 ?

Moreover, as both numerator and denominator have length units, the unit doesn't matter as long as they are the same e.g. cm, mm or inches.

Thanks, cmɢʟeeτaʟκ 04:31, 26 September 2024 (UTC)[reply]

Probaly because the editor who added that didn't know how to render mathematical formulae. You're welcome to change it. Shantavira|feed me 09:16, 26 September 2024 (UTC)[reply]
Thanks. I don't mean the formatting but the division by 2π on top with multiplication by 0.5 below.
Presumably, they meant waist / (2π) instead of (waist / 2)π – BEMDAS is confusing! cmɢʟeeτaʟκ 10:17, 26 September 2024 (UTC)[reply]
Because it's left over from its derivation from the eccentricity formula for an ellipse, , where the semi-minor axis, b, is the "radius" of the waist and the semi-major axis, a, is half the body height. AlmostReadytoFly (talk) 11:05, 26 September 2024 (UTC)[reply]
Makes sense, thanks! cmɢʟeeτaʟκ 11:34, 26 September 2024 (UTC)[reply]

September 27

[edit]

Demonstration of Planck's law in radiation power per unit area?

[edit]

To demonstrate his law, Max Planck converted radiation power measurements per unit area into energy density ().
Is there a demonstration directly with the radiation power from the experiment without conversion into energy density?
I can't find any, but maybe I'm not seeing the correct wording for search engines.
Malypaet (talk) 21:28, 27 September 2024 (UTC)[reply]

The Stefan-Boltzmann constant can be derived by integrating the spectral radiance of a blackbody. Spectral radiance is the power emitted by a surface per unit area per unit solid angle per unit frequency (or wavelength). The Stefan-Boltzmann law states that the power radiated per unit area of a black body is proportional to the fourth power of its absolute temperature. Answer generated with Bing CoPilot — Preceding unsigned comment added by 217.180.227.58 (talk) 20:38, 30 September 2024 (UTC)[reply]

September 28

[edit]

Is the following argument sufficient, for logically proving that any material can become energy - without using Einstein's formula?

[edit]

Here are three accepted assumptions:

1. There are black holes.

2. A given black hole can-theoretically absorb any given material.

3. A given black hole can-theoretically evaporate, by becoming Hawking radiation.

Hence, logically, any given material can-theoretically become energy: Just let this material be absorbed by a black hole, and then let the black hole evaporate and become Hawking radiation.

Apparently, all of this is done without using Einstein's formula . So, it seems that Einstein's formula is not needed for proving that any given material can-theoretically become energy, right? HOTmag (talk) 18:00, 28 September 2024 (UTC)[reply]

Hawking radiation is not all energy. It contains particles (and anti) too. To theorize about Hawking radiation you (or Hawking) need(ed) Einstein's equation. So you need it, but you need not write about it. 176.0.164.155 (talk) 19:55, 28 September 2024 (UTC)[reply]
1. Re. the particles contained in Hawking radiation: So why does the lead of our article Hawking radiation only describe it as "black body radiation", i.e. "electromagnetic radiation", without mentioning any "particles" contained in Hawking radiation?
2. Are you sure the formula is needed for concluding that black holes emit Hawking radiation? HOTmag (talk) 22:09, 28 September 2024 (UTC)[reply]
1 See the first paragraph in Emission
2 See the first paragraph in black hole evaporation
In 1 you need to pay special attention to the word "particle". 176.0.164.155 (talk) 23:35, 28 September 2024 (UTC)[reply]
Re. 1: Yes I'd seen this paragraph, but it doesn't answer my previous question, so let me repeat it: Why does the lead of the article only describe Hawking radiation as "black body radiation", i.e. "electromagnetic radiation", without mentioning any "particles" contained in Hawking radiation? Are you claiming that black body radiation can contain particles (besides energy)?
Re. 2: Yes this paragraph really shows how Hawking uses Einstein's formula for concluding that the black hole, not only creates energy, but also becomes energy. However, this article indicates also that "some [authors] find Hawking's original calculation unconvincing" - because it uses an "infinite frequency" as well as "a wavelength much shorter than the Planck length", while these authors use techniques other than Hawking's one, so I still wonder whether Hawking's technique using Einstein's formula is necessary for concluding that the material in the black hole, not only creates energy, but also becomes energy. HOTmag (talk) 01:06, 29 September 2024 (UTC)[reply]
Particles are energy and electromagnetic waves are particles; they are two aspects of the same. It's just that at typical temperatures used for blackbody radiation, the only particles you can make are photons. (There's enough energy too to make neutrinos, but that requires some weak interactions, so it's unlikely to happen.) Once energies go to the MeV scale (temperatures of gigakelvins), your blackbody radiation will contain other particles.
Not sure what you mean by "creates energy" or "becomes energy". Energy cannot be created or destroyed; it's always there. It just changes shape. Mass is equivalent to energy, that's an intergal part of relativity. And "equivalent to" doesn't mean "can be turned into", it means "is an alternative view of". PiusImpavidus (talk) 09:34, 29 September 2024 (UTC)[reply]
The lede indeed says "black-body radiation" and I think that's misleading. It was introduced here. I've changed it to prevent misunderstandings. --Wrongfilter (talk) 11:44, 29 September 2024 (UTC)[reply]
Thank you for this important correction. HOTmag (talk) 17:29, 29 September 2024 (UTC)[reply]
the only particles you can make are photons. (There's enough energy too to make neutrinos, but that requires some weak interactions, so it's unlikely to happen.) Are you claiming, that the "particles" mentioned in the first paragraph of the chapter Emission only mean "photons" (or neutrinos but it's unlikely), for "regular" tempratures?
Not sure what you mean by "creates energy" or "becomes energy". When I wrote "this paragraph really shows how Hawking uses Einstein's formula for concluding that the black hole, not only creates energy, but also becomes energy", I meant that the first paragraph in black hole evaporation really showed how Hawking used Einstein's formula for concluding that the black hole, not only emitted energy, but also lost mass equivalent to the emitted energy.
Mass is equivalent to energy, that's an intergal part of relativity. Who said that that was not? I only said, that without Einstein's formula [you'd have had no special relativity, so] you couldn't have concluded: "Mass is equivalent to energy".
Energy cannot be created or destroyed; it's always there. Correct, but without Einstein's formula that paragraph couldn't have concluded that "When particles escape, the black hole loses a small amount of its energy and therefore [loses] some of its mass", because without Einstein's formula - one could imagine a body emitting energy - while the body's mass remains the same as before the emission - while the emitted energy does not disappear but is only released ousdise. HOTmag (talk) 17:29, 29 September 2024 (UTC)[reply]
You also need Einstein's equation to prove that black holes can exist (assumption 1). PiusImpavidus (talk) 20:28, 28 September 2024 (UTC)[reply]
I was referring to Einstein's formula, i.e. Are you sure this equation (=formula) is needed for concluding that black holes exist? HOTmag (talk) 22:09, 28 September 2024 (UTC)[reply]
You want to remove special relativity, but maintain General relativity? You can't. The latter relies upon the former. 2A0D:6FC0:767:D900:3439:2201:29C1:1A87 (talk) 08:31, 29 September 2024 (UTC)[reply]
Theoretically, one could consider General relativity without considering Special relativity: Combining both theories, gives us a Pseudo Riemannian manifold - and as a special case - a Lorentz 4D space. But Special relativity alone - would only give us a Pseudo Euclidean 4D space - and as a special case - Minkowsky space, while General relativity alone - would only give us a Riemannian 4D space. To sum up: Theoretically, one could imagine a Generally relativistic 4D space, that ignores Special relativity. The same is true for the issue of mass-energy equivalence you're talking about: Also without Special relativity, one could still consider the Einstein field equations of General relativity, so that the geometry of spacetime would be shaped by the density and flux of momentum and of energy according to these field equations, but without assuming anything about any relation between mass and energy.
But this is a side point. My main question to user:PiusImpavidus was about whether Einstein's formula is really needed for concluding that black holes exist. So I'm still asking: Is it needed? HOTmag (talk) 17:29, 29 September 2024 (UTC)[reply]
No it's not needed. There was a theory of black holes before Einstein (by Gottfried Wilhelm Leibniz,I think), but they were way bigger than according to Einstein. 176.0.162.8 (talk) 12:47, 30 September 2024 (UTC)[reply]
Since Hawking radiation includes particles, no your asumptions don't logically lead to that any material can become energy. NadVolum (talk) 19:06, 29 September 2024 (UTC)[reply]
Yes.
Due to the current thread, the article Hawking radiation has just been corrected by user:Wrongfilter, so now it explicitly states (in the lede) that Hawking radiation includes also particles. But when I posted my original post, the lede of the article had only mentioned electromagnetic radiation. That's why I posted my original post. HOTmag (talk) 19:33, 29 September 2024 (UTC)[reply]

e-bike = 1000 miles per gallon gasoline?

[edit]
  1. 1 gallon gasoline = 127 megajoule (per the gasoline article) = 35KWH thermal energy
  2. If you can convert that to electricity at 28% efficiency (portable generator), that's 10KWH electric
  3. Ebikes can go around 50 miles on a 500 WH battery charge, so 100 miles per KWH
  4. So that's 1000 miles per gallon if you power the bike from a generator.

Questions: 1) Amirite? I.e. does the math above look ok? 2) Why are motorized bikes/mopeds so much less efficient? They typically get 100 mpg or so.

Thanks. 2601:644:8581:75B0:0:0:0:C813 (talk) 22:20, 28 September 2024 (UTC)[reply]

28% is way high for a generator. Tires, mass, are significantly different. Does your genny meet the emissions regs for a moped? Moped's aren't optimised for economy, bicycles are. Greglocock (talk) 08:03, 29 September 2024 (UTC)[reply]
A Stirling engine with 1200 Kelvin input (and 300 Kelvin output) has an efficiency of 75% (theoretically). And 1200 Kelvin is not outrageously high. So an efficiency of 28% is not high, but rather low. 176.0.162.8 (talk) 12:37, 30 September 2024 (UTC)[reply]
Nonsense, Greglocock (talk) 07:52, 1 October 2024 (UTC)[reply]
Care to elaborate? 176.0.159.38 (talk) 10:00, 1 October 2024 (UTC)[reply]
Sure. The most efficient internal combustion engines are far short of 75% efficient, and the engines used in gennies are fairly basic. Yes I have been an engine design engineer. And as the article on stirlings says "Stirling engines cannot achieve total efficiencies typical of an internal combustion engine, the main constraint being thermal efficiency" Greglocock (talk) 10:35, 1 October 2024 (UTC)[reply]
E-bikes (which are basically light electric mopeds; I don't see why the law makes a distinction between those) typically cruise at about 6 m/s (22 km/h). A regular moped cruises at 12.5 m/s (45 km/h), twice as fast. That quadruples drag and energy use. Combine that with the low efficiency of small (but still oversized), two-stroke petrol engines and the much lower rolling resistance of bicycle tyres, in particular when compared to the tyres of motorscooters. PiusImpavidus (talk) 09:08, 29 September 2024 (UTC)[reply]
Yes, aero and higher cruising speeds are the most likely cause. Greglocock (talk) 07:52, 1 October 2024 (UTC)[reply]
If it helps, I am using a gas generator right now because we have no power. It is producing between 4 and 5 KWH per gallon of gas. It is not a bad generator at all. It is a brand new one in the upper price range: Westinhouse WGEN2000C (all the cheaper ones were sold out). I assume that industrial level generators will do better, but I doubt a residential one will get to 10KWH. (I updted the numbers after checking the generator today. After a rough start, it is doing better now.) 12.116.29.106 (talk) 11:01, 1 October 2024 (UTC)[reply]

September 29

[edit]

Must a given body, that has just been a black hole, always remain a black hole, as long as the body exists?

[edit]

In other words, what rules out the following scenario?

1. A body, being right now a Schwarzschild black hole, starts emitting Hawking radiation.

2. However, the body's radius remains constant during the emission.

3. When the body has lost too much energy - along with its equivalent mass, the body's mass inside the constant body's radius becomes less dense, untill the body's current radius becomes bigger than the body's Schwarzschild radius - because of the stability (constancy) of the body's radius, so the body - which has just been a black hole - stops being a black hole and becomes a regular body.

What's wrong with this scenario? Is this really assumption #2 ? HOTmag (talk) 18:54, 29 September 2024 (UTC)[reply]

THe theory says #2 is wrong - black holes become smaller as their mass goes down. NadVolum (talk) 19:10, 29 September 2024 (UTC)[reply]
Yes, both our article black hole and our article Hawking radiation state that when the black holes emit radiation they "shrink", but how do you know that their shrinkage refers, not only to the body's mass, but also to the body's radius? This is the main question of this thread.
Is this because of the internal gravitation, which is the only "force" active inside the black hole? HOTmag (talk) 19:47, 29 September 2024 (UTC)[reply]
I think you're giving the word 'body' too much meaning. The radius is a gravitational result which depends on the mass.You wouldn't notice the surface as you fell through. NadVolum (talk) 20:33, 29 September 2024 (UTC)[reply]
I used the term "body" on purpose. A given body, whether a black hole or a billiard ball, has a radius, literally speaking. It's a fact you can't ignore. Nor can you ignore the influence of the body's radius on the density of the body's mass, hence on the question of whether this body is a black hole, because the body's radius is not necessarily identical to the body's Schwarzschild radius: Actually the former is not bigger than the latter if and only if the body is a black hole. This is the basis of my #3 assumption. If you don't agree to it, please explain what's wrong there, in your opinion. If you do agree to it, then I'm still asking the question in my previous response. HOTmag (talk) 09:10, 1 October 2024 (UTC)[reply]
Assumption #4: a hole is not a body. 176.0.159.38 (talk) 10:04, 1 October 2024 (UTC)[reply]
Our article black hole refers to "the nearest known body thought to be a black hole, Gaia BH1". It also refers to Cygnus X-1 as the first "object" identified as a black hole. Actually when I wrote "body" I meant a region, located in spacetime, and characterized by dense mass which doesn't let light escape when it's close enough. What's wrong in that view? HOTmag (talk) 11:41, 1 October 2024 (UTC)[reply]
Do you think an atom is a body? What use is the concept of density for an atom, where is that density? Density has even less real meaning for a black hole than it does for an atom, you can calculate a number by dividing one number by another but what then - what does it actually refer to? NadVolum (talk) 12:39, 1 October 2024 (UTC)[reply]
Is a stellar black hole a body? Let's check out: Does it have a mass? If it does, then does this mass have a location? If it does, then is this mass located in one geometric point? If it doesn't, then the mass must be located in some region. Does this region have an (average) radius? If it does, then we can sum up: a stellar black hole has a mass located in an (average) radius, so is it a body? HOTmag (talk) 13:55, 1 October 2024 (UTC)[reply]

September 30

[edit]

Electron capture cross-sections for hydrogen

[edit]

Is there any publication out there that discusses what the cross-section for energetic electrons converting free protons into neutrons is? It's energetically unfavourable but with sufficient electron energy it should be possible. Jo-Jo Eumerus (talk) 09:18, 30 September 2024 (UTC)[reply]

Biotransformation of TFA

[edit]

Reference 22 (Kirschner, E., Chemical and Engineering News 1994, 8) does not seem to be appropriate for the statement "Biotransformation by decarboxylation to fluoroform has been discussed." In https://pubs.acs.org/toc/cenear/72/32, there is only one article by Kirschner, but about an entirely different subject. Could somebody please advise what to do? 162.23.30.16 (talk) 17:51, 30 September 2024 (UTC)[reply]

The simplest is to slap a template {{failed verification}} on the citation. But the statement itself may be true. Quoting this secondary source:
Visscher et al. [44] showed that trifluoroacetic acid could be microbially metabolized to fluoroform and consecutively defluorinated to acetate under aerobic and anaerobic conditions, respectively.
...
[44] Visscher, P.T., Culbertson, C.W. and Oremland, R.S. (1994) "Degradation of trifluoroacetate in oxic anoxic sediments". Nature (Land.) 369, 729-731.
I have not inspected the primary source, though, which has many citations ([6]).  --Lambiam 04:44, 1 October 2024 (UTC)[reply]
Thanks I replaced the ref [7] 162.23.30.16 (talk) 14:44, 1 October 2024 (UTC)[reply]

Parasites

[edit]

Do parasites that feed off other parasites exist? ―Panamitsu (talk) 23:13, 30 September 2024 (UTC)[reply]

Do you consider bacteriophages to be parasites? ←Baseball Bugs What's up, Doc? carrots00:19, 1 October 2024 (UTC)[reply]
See Hyperparasite. {The poster formerly known as 87.81.230.195} 94.6.86.81 (talk) 02:14, 1 October 2024 (UTC)[reply]
Big fleas have little fleas... (about halfway down). The overall text doesn't specifically mention any cases of recursive parasitism that I could see, nor does it cite a source for the rhyme. -- Verbarson  talkedits 16:41, 1 October 2024 (UTC)[reply]

October 1

[edit]

Mathematics

[edit]

September 19

[edit]

Property of a matrix

[edit]

If I raise a permutation matrix to a high power it is still a permutation matrix (all values 0 or 1). If I raise a stochastic matrix to a high power it is still a stochastic matrix (all values <=1). But there are a lot of matrices raised to a high power have elements with very large values. Is there a name for the property of a matrix that when raised to a high power remain reasonably valued? RJFJR (talk) 22:12, 19 September 2024 (UTC)[reply]

The spectrum of such a matrix is inside the closed unit disk (a necessary but not sufficient condition). A matrix whose spectrum is in the open unit disk always satisfies this property. I'm not sure there is a general name for this property though. Tito Omburo (talk) 23:07, 19 September 2024 (UTC)[reply]
That's it. Thank you! RJFJR (talk) 02:08, 20 September 2024 (UTC)[reply]
Note that when the spectral radius is strictly less than the powers of the matrix tend to the null matrix, which is not very exciting. I also vaguely remember seeing a theorem that states or implies that when the spectral radius is equal to the growth of the entries is polynomial in the value of the exponent.  --Lambiam 21:55, 20 September 2024 (UTC)[reply]

September 20

[edit]

Adef
=
B+C

[edit]

1. Since the expression in the title is a definition, we can conclude, that A exists if and only if both B and C exist as well.

2. However, we can't conclude that if B or C exist then A exists. Check: A denotes the total price, B denotes the price of the first product, and C denotes the price of the second product: if only the price of the first product exists, we still can't conclude - that the price of the second product exists - nor that the total price exists. The defintion only tells us (besides the relation ), that the total price exists if and only if both - the price of the first product exists - and the price of the second product exists.

3. For concluding, that if any value of the above three values A,B,C exists then all of them exist, it's sufficient to write down three definitions:

4. Question: as I've indicated, it's sufficient (to write down all three defintions), but is there any shorter notation, to make sure that if any value of the above three values A,B,C exists then - all of them exist - and satisfy

HOTmag (talk) 13:26, 20 September 2024 (UTC)[reply]

Are you asking about a shorter notation? I may be looking mighty foolish, but isn't it circular and therefore meaningless to write the system of three definitions like you have above? Or rather, wouldn't writing down the first be exactly as meaningful as writing down all three, since both situations only relate A, B, C to one another? I get they're definitions and not merely statements, but I'm not really seeing the difference here. You'd have to define B, C in terms other than A to reach a more meaningful domain. Again, I could look totally foolish right now, since I've never answered a question here before. Remsense ‥  13:34, 20 September 2024 (UTC)[reply]
It seems you haven't read 2#. HOTmag (talk) 13:40, 20 September 2024 (UTC)[reply]
I don't really understand it, no. You're defining A, B, C as the prices of products, but then you're writing abstract definitions of them in terms of each other. Am I missing something? Oh, did you mean to say A is the total? It makes more sense to me that way. Remsense ‥  13:43, 20 September 2024 (UTC)[reply]
In any case, I still don't understand what the extra two definitions achieve: either B and C are defined outside of A or they're not, right? If we wanted to know them in terms of A, we already got that in the first definition. Remsense ‥  13:45, 20 September 2024 (UTC)[reply]
I've just added an addition to 2#, to make it clear. HOTmag (talk) 13:49, 20 September 2024 (UTC)[reply]
I return to my initial question then, are you just looking for a shorter notation for this? Remsense ‥  13:52, 20 September 2024 (UTC)[reply]
Yep. I've just made it clear in 4# (thanks to your question). HOTmag (talk) 13:55, 20 September 2024 (UTC)[reply]
Sorry for being slow on the uptake. I'm not sure if this needs to fit into any particular system or paradigm: anything wrong with . Sorry if that hurts anyone to see, haven't flexed these muscles in a while Remsense ‥  14:31, 20 September 2024 (UTC)[reply]
Not to bug you, but only since I'm relatively unsure of myself in this area—was this answer something like what you were looking for? Remsense ‥  20:10, 20 September 2024 (UTC)[reply]
You limit the set of the Bs and the Cs to be the positive integers, but my question is general, without limiting anything. HOTmag (talk) 02:05, 22 September 2024 (UTC)[reply]
You gave the example of prices, so that's what I picked the positive reals* based on. Clearly, you can replace the set with whatever you want. Remsense ‥  02:08, 22 September 2024 (UTC)[reply]
If A denotes the total price, B denotes the price of the first product, and C denotes the price of the second product, does your definition let us deduce the other definition: which I would like to deduce, bearing in mind that not all products have a price? Note: Since the latter is a defintion of C, then the existence of A and of B must be derived from the very existence of C. HOTmag (talk) 02:21, 22 September 2024 (UTC)[reply]
It seems the bit about "deducing a definition" articulates a fundamental confusion you have about what you're trying to accomplish. Definitions are stated, not deduced. Remsense ‥  05:34, 22 September 2024 (UTC)[reply]
By asking whether a new definition can be deduced from an old definition, I mean whether a new claim, that was presented before as an additional definition, can be derived as consequence, from a given assumption that was presened before as an old definition.
In our case, the given assumption, was presented before as an old definition: Adef
=
B+C
. The new claim, was presened before as an additional definition: The question is, whether we can deduce the latter from the former, i.e. whether we can deduce the new claim from the given assumption. HOTmag (talk) 13:08, 22 September 2024 (UTC)[reply]
If the values we assign to B and C are logically independent from one another, then no. That's what logical independence means.Remsense ‥  13:17, 22 September 2024 (UTC)[reply]
This is totally confused. The addition operation is special in that it is operand-wise strictly monotonic and therefore has, at least in the integers and reals (but not in the natural numbers) an operand-wise inverse, which we can denote using the subtraction operation . In general, this is not possible.
Defining some quantity by an equation of the form only makes sense if all terms in the right-hand side are defined. It is not just that this fails to define if or is not defined. It just does not make sense. And if is not defined, writing something like only increases the confusion.
There is another issue in which the definedness of a defined term depends on the definedness of another term, namely when defining a function. Suppose we define a new function using existing known functions with a limited domain. For example, we may define real-valued function on the real numbers by the equation
For to be defined by this equation for some given value of it is necessary that both and are defined. This is more a matter of common sense than anything else, and I see no need for some notational device to express this dependency.  --Lambiam 22:29, 20 September 2024 (UTC)[reply]
If A denotes the total price, B denotes the price of the first product, and C denotes the price of the second product, does the definition Adef
=
B+C
make senee? If it does, can you deduce the definition: HOTmag (talk) 02:13, 22 September 2024 (UTC)[reply]
You can't define something that already has a meaning. So if A, B and C all have independent meanings, as shown by the use of "denote", then none of them can be defined in terms of the other two. It may be true that A = B + C based on these meanings, but that's not a definition. In any case, you can't deduce a definition. If A has no independent meaning then you can define it to be anything, B + C, B - C, or B * C. --RDBury (talk) 05:29, 22 September 2024 (UTC)[reply]
Well, I'm presenting an analogous question (taken from a discipline very close to arithmetic), in my following thread. I hope my new question explains also my old question, but if my old question is still not clear, you can ignore it, and focus on my new question. HOTmag (talk) 13:38, 22 September 2024 (UTC)[reply]

September 22

[edit]

How can we briefly characterize a given set of vectors, as "linearly dependent - every proper sub set being linearly independent", while we only refer to the vectors rather than to their set?

[edit]

For example: S is a set of the following vectors:

A=(1,1,0),
B=(1,0,0),
C=(0,1,0).

Note: A=B+C, and B=A-C, and C=A-B, so the set S is linearly dependent.

Using A,B,C only, i.e without using S, what's the shortest description, claiming that the set S is linearly dependent but every proper sub set of S is linearly independent? HOTmag (talk) 13:30, 22 September 2024 (UTC)[reply]

@HOTmag: I'd just say 'S has k linearly independent elements' (in the give example k=2). --CiaPan (talk) 14:46, 22 September 2024 (UTC)[reply]
Is your response a suggestion of rephrasing my question?
If it's intended to be an answer, then please note: My condition requires to be "using A,B,C only, i.e without using S". Additionally, where does your description claim, that S is linearly dependent? HOTmag (talk) 14:56, 22 September 2024 (UTC)[reply]
One way to characterise the set is "a set of vectors, any one of which can be written in terms of the others in a unique way". The set is just A, B and C, i.e. any property of them is a property of the set of them. --2A04:4A43:900F:F4C3:49F4:4EFB:C442:608F (talk) 15:15, 22 September 2024 (UTC)[reply]
Since my condition requires to be "using A,B,C only, i.e without using S", so I guess you mean the following: "Each vector, can be written as a unique linear combination of the other vectors". Thanks. HOTmag (talk) 15:52, 22 September 2024 (UTC)[reply]
Assuming the vectors are , form the matrix whose columns are . The stated condition is then:
  1. the matrix of minors of is zero, and
  2. each of the columns of the matrix of minors of V has a non-zero entry.
- Tito Omburo (talk) 17:36, 22 September 2024 (UTC)[reply]
Your description, both uses sets, and also becomes longer than the original one indicated in the title. HOTmag (talk) 08:41, 23 September 2024 (UTC)[reply]
You can say, "each of the sets {A,B}, {A,C} and {B,C} is linearly independent".  --Lambiam 21:05, 22 September 2024 (UTC)[reply]
You also have to add that the set {A,B,C} is linearly dependent, but then the description - both uses sets, and also becomes longer than the original one indicated in the title. HOTmag (talk) 08:40, 23 September 2024 (UTC)[reply]
There is a unique linear combination generating 0, and in this linear combination all coefficients are nonzero.2404:2000:2000:8:FDE8:8311:95E3:654D (talk) 00:00, 23 September 2024 (UTC)[reply]
Yes, and by symbolic notation the description even becomes shorter. Thanks. HOTmag (talk) 08:42, 23 September 2024 (UTC)[reply]

September 23

[edit]

Converting map scale into height and width

[edit]

How can I calculate the height and width in meters of a future geographic map of the world based on its desired scale in cm and km? 212.180.235.46 (talk) 15:35, 23 September 2024 (UTC)[reply]

For a map of the world, you will have to specify which map projection it will use, and then to which part(s) of the projection the scale is to apply (and for some projections, how much of the globe to include). Mapping a spherical surface onto a plane surface cannot be done with a constant scale factor. Eg: you might specify a Mercator projection, between latitudes +/-85deg, with a scale of 1:10,000,000 at the equator, which would give you a map about 4m wide along the equator, and (judging by the illustration for Mercator) slightly more than that high. Other projections will vary considerably. -- Verbarson  talkedits 18:07, 23 September 2024 (UTC)[reply]
My own map with van der Grinten projection and equatorial scale of 1:20 000 000 for example is 1.18 m high and 1.94 m wide. Out of curiosity, I started to think about dimensions of imaginative humongous world maps. With that as benchmark, if my calculations are correct, a 1:160 000 scale world map, for example, would be 147 m long and 242 m wide. 212.180.235.46 (talk) 20:35, 23 September 2024 (UTC)[reply]
The scale of a map is the ratio between the distance between two points on the map and that between the corresponding points on the ground. Any map projection necessarily distorts the distances, so the scale you get if you take New York and San Francisco as the two points is generally substantially different from what you get if you take Berlin and Moscow. For maps of the whole Earth it is non-trivial to decide on a "nominal scale". In spite of the name, the van der Grinten projection is not a geometric projection in which points on the map are found by following rays projected along a straight line out of a shrunk globe. The equatorial and central-meridional scales are well defined, though. The length ℓeq of the equator is approximately 40,000 km. If the equatorial scale is chosen to be 1 : Seq, the width of the map is ℓeq / Seq. So for Seq = 160,000 this comes out as about 250 m. The length ℓmer of a meridian, measured from one pole to the other, is about 40,000 km. Letting Scm stand for the central-meridional scale factor, the height of the map will be ℓmer / Scm. If these scale factors are chosen to be equal, the map will have a height of half its width. For a circular map as shown in the article, Scm = 2 Seq.  --Lambiam 10:14, 24 September 2024 (UTC)[reply]


September 26

[edit]

Given the optimal ate pairing e(A,B)=y is to possible to determine I and J such as e(I,J)=2y or even e(I,J)=3y?

[edit]

Simple question : let’s say I have a pairing friendly curve having a very large trace, and that I have a pairing with points A∈ and B∈ such as the optimal ate pairing e(A,B)=y, then is it possible to fully determine 2 point I and J such as e(I,J) equals a target multiple of the finite’s field element y ?

Or does it requires full pairing or full generalized Miller’s inversion and thus would be impossible in practice on a curve like bn254 ? 2A01:E0A:401:A7C0:9CB:33F3:E8EB:8A5D (talk) 15:51, 26 September 2024 (UTC)[reply]

September 27

[edit]

Uses of Pascal's tetrahedron

[edit]

The following is a use of Pascal's triangle:

To find how many ways there are to make a total of circles all black or red, the formula is just . For example, there are ways to make a group of 6 circles, all black or red, classified by whether each circle is black or red. An example is black-red-black-red-black-red.

But how about finding the number of ways to make a group of 6 circles, all black or red, classified by how many are black or red. To find out how many ways there are to make a total of circles all black or red classified by how many are black and how many are red, you use the n+1th row of Pascal's triangle. For , this means we use the seventh row, which is . This means that there is one way to color 6 circles where all of them are black, 6 where 5 are black and one is red, 15 where 4 are black and 2 are red, 20 where 3 are black and 3 are red, 15 where 2 are black and 4 are red, 6 where one is black and 5 are red, and one where all 6 are red.

How about a similar application for Pascal's tetrahedron?? Here is the seventh layer of the tetrahedron:

1
6 6
15 30 15
20 60 60 20
15 60 90 60 15
6 30 60 60 30 6
1 6 15 20 15 6 1

Just as the seventh row of Pascal's triangle can be used for the classification of ways to make 6 circles all of which are black or red classified by how many are black and how many are red, it is likewise true that the seventh layer of Pascal's tetrahedron can be used for... Georgia guy (talk) 14:06, 27 September 2024 (UTC)[reply]

It counts how many ways there are to make 6 circles all of which are black, red, or green, classified by how many are black, how many are red, and how many are green. For example, if you want there to be 2 of each colour, you get ways, which is exactly the middle entry of this layer. Double sharp (talk) 14:55, 27 September 2024 (UTC)[reply]
I would call 1, 6, 15, ... the sixth row, and the top row, with a single 1, the zeroth row. That's what Wikipedia's does in the articles I've seen at least. Anyway, this should be in the article Pascal's pyramid, and can be further extended to Pascal's simplex. The article on the Multinomial theorem is relevant here as well. --RDBury (talk) 17:25, 27 September 2024 (UTC)[reply]
RDBury, is the top row "1" not really a row?? Georgia guy (talk) 21:02, 27 September 2024 (UTC)[reply]
Well, you can call it whatever you want and index it as you like; I'm not going to get into the philosophy of rows. But the formulas are simpler if you start with the top "1" as row 0. --RDBury (talk) 00:55, 28 September 2024 (UTC)[reply]
RDBury, does that statement parallel the statement that trigonometry is simpler if you use radians as opposed to degrees?? Georgia guy (talk) 01:04, 28 September 2024 (UTC)[reply]
The OP was calling what most people call row 6 "the seventh row" and I thought that was worth pointing out for future reference. Getting caught up in what counts as a "row" and whether statements are parallel is a matter for philosophy and linguistics. --RDBury (talk) 02:20, 28 September 2024 (UTC)[reply]

September 28

[edit]

Bitcoin price rigging

[edit]

I have been day trading bitcoin for almost a year now. I read articles from bitcoin news to try and guess when the dips and dives will happen and so far it has helped. I just read and article about bitcoin possibly losing freedom to governments and large institutions being able to rig the price. This has happened recently with the German government of Saxony selling off a hoard of seized coin, The Mt. Gox dispersal, and the US dept of Justice mysteriously moving 30/230k of its seized hoard two days after Trumps bitcoin speech in Nashville raised the price. https://mpost.io/u-s-unloads-2-billion-in-bitcoin-from-silk-road-seizure/

Is it possible for a whale (large bitcoin hodler) to make smaller transactions in a short time to move the price for a larger transaction at a later time? On the upswing this is called 'pump and dump' and 'poop and scoop' on the down swing. Both are illegal in most market trades.

This post may fit better in Humanities where finance is listed or IT where crypto may belong. 2604:3D08:5E7A:6A00:D94:3638:168B:18A0 (talk) 08:38, 28 September 2024 (UTC)[reply]


I'm pretty sure that this isn't a math question. But the article on Pump and dump does explicitly mention cryptocurrencies. --RDBury (talk) 13:22, 28 September 2024 (UTC)[reply]
The reason I posted in math is because I am wondering if a small buy or sell in a quick time frame will actually move the price enough for traders like me who watch the minute candle scale. I use MetaTrader 4 where the price moves in microseconds every time a buy/sell happens. Is there a formula for volume needed to move the price in a small time frame or article about time/volume/price/ ratio calculations?
Pump and dump, Bear raid, Short and distort, and Uptick rule, all help to explain how it is possible for whales to control the price. Fear_of_missing_out#Investing is the main cause of up-spikes since I have been investing and most are followed by dives. https://www.weforum.org/agenda/2024/08/explainer-carry-trades-and-how-they-impact-global-markets/ The carry trade crash the 1st week of August caused another huge dive. Foreign_exchange_market#Carry_trade2604:3D08:5E7A:6A00:D94:3638:168B:18A0 (talk) 17:48, 28 September 2024 (UTC)[reply]
The time needed to move the needle with high-frequency trading is a combination of the latency of the connection between the computers of the flash traders and those of the exchanges, which depends on the current state of the networking technology and the physical distance between the traders and the exchange, as well as any regulations enforcing a time lag. It then will take time before small-time traders can see the needle having moved. A mathematical model will require too many parameters to be of practical use.  --Lambiam 08:51, 29 September 2024 (UTC)[reply]

September 29

[edit]

Is this a possible Groth16/ZkSnark verifier‑side simplification ?

[edit]

Hello,

The verification algorithm is already simple but I was thinking about some costly environments like blockchains having low block limits. This might be naïve thinking but I was wondering at possibility : normally the prover gives 3 elliptic curves points to the verifier A ; B ; C When public inputs are used C/the inputs vector is split.

But as a simplification part, why not completely ditch the C parts of the proof when public inputs are used ? That way, the verifier would have to compute 1 pairing in less for verifying the proof. I’m meaning e(C,verifying_key_part). It seems to me the requirement to pair with public inputs would still ensure the security of the system… Is it because skipping that pairing would allow to forge public inputs ? As far I understand, a malicious prover would still have to satisfy all constraints of the quadratic arithmetic program and thus would have to use public inputs satisfying constraints. Or is it because it would be impossible to rework the protocol to have the prover being able to produce proofs that verify ?

Or even maybe both of the assumptions above ? 2A01:E0A:401:A7C0:9CB:33F3:E8EB:8A5D (talk) 11:51, 29 September 2024 (UTC)[reply]

September 30

[edit]

How to find max(xy) for each r where x + y = r and x, y are positive integers

[edit]

Drawing fractal canopy diagrams had me wondering for a given SVG file size, I could add more branches or more depth. Below are two examples:

2 branches, depth 12
4 branches, depth 8

To make the tree as dense as possible, I wish to maximise the number of leaf nodes. In other words, how can I find max(xy) for each r where x + y = r and x, y are positive integers?

I realise I can use calculus but I'm unsure what equation I should differentiate.

Thanks, cmɢʟeeτaʟκ 00:50, 30 September 2024 (UTC)[reply]

Unfortunately, this is actually quite complicated. We can rewrite the problem as , and consider all real instead of just integers. Notice that . Because is monotonic over , is maximized when is. Its derivative is , which is when . This is a nice closed-form expression, but it's for in terms of . Inverting it is complicated, but Wolfram Alpha gives us where is the Lambert W function. While this is sort of a closed-form expression, it's still unfortunately annoying to work with since is implicitly defined as . In any case, is irrational for all positive rational . The proof of this is annoying so I've put it below, but ultimately what it implies is that when , as far as I can tell, the best you can do is either round for convenience, or take floor/ceiling of and compare values to get the max over integers.
Proof that is irrational for positive rational
  1. Suppose , and let .
  2. and , so as well.
  3. By definition, . Since , we can rearrange to get .
  4. Lindemann's 1882 theorem implies that if is nonzero rational, then is not only irrational, but transcendental as well.
  5. If is rational, then since , is transcendental, while is rational, which is contradictory.
  6. must be irrational and is nonzero, so is irrational.
  7. We conclude that implies is irrational.
GalacticShoe (talk) 02:41, 30 September 2024 (UTC)[reply]
Here is a numerical recipe (Newton's method) for solving for real-valued :
  1. Set
  2. Iterate the replacement until convergence, where
In practice (), two iterations will bring you close enough; then test and to get the optimal integer value.  --Lambiam 10:17, 30 September 2024 (UTC)[reply]
Thanks so much, @GalacticShoe and @Lambiam. I thought analytically I was heading into a dead end. Guess solving it iteratively is still the best for small values. cmɢʟeeτaʟκ 11:45, 30 September 2024 (UTC)[reply]
P.S. It seems there's an interesting trend:
x=1 for r=2 (1 term): 1¹.
x=2 for r=3 to 4 (2 terms): 2¹ and 2².
x=3 for r=5 to 7 (3 terms): 3², 3³ and 3⁴.
x=4 for r=8 to 11 (4 terms): 4⁴, 4⁵, 4⁶ and 4⁷.
x changes when r is the x–1th triangular number + 2. Serendipity? cmɢʟeeτaʟκ 17:06, 30 September 2024 (UTC)[reply]
Unfortunately, serendipity. The number of for each is the sequence OEIS:A108414. Although it starts with , it quickly levels out. The inverse triangular number function you're looking for is , while grows faster than , which in turn grows faster than , hence the number of terms slowing down in growth. GalacticShoe (talk) 03:46, 1 October 2024 (UTC)[reply]
Note that, unlike for Tn, these first differences in the r-values for which x changes do not only always increase but may even decrease.  --Lambiam 03:58, 1 October 2024 (UTC)[reply]
After some testing, it seems that rounding suffices at least for r < 100, possibly more. To simplify it, applying Newton's method twice, we can get this approximation which maximizes for these values of :
GalacticShoe (talk) 04:14, 1 October 2024 (UTC)[reply]

October 1

[edit]

Humanities

[edit]

September 17

[edit]

People guessing keys of melodies using wrong rules

[edit]

Look at Talk:Hail to the Chief.

Back in 2004, I (at that time using 66.32 and 66.245 IP addresses [I got a registered user name on January 1, 2005]) made the first post to the talk page, simply writing the melody. The key is G major.

Years later, another IP (I never bothered to study this talk page until recently) made comments implying that the melody the way I posted it was in D major, using the bad argument that a melody must start on the tonic. It's quite common for melodies to start on the dominant. Is this a common wrong rule some people use?? (Another important fact is that the post I made back then was before Wikipedia adopted a rule that you can't use a number sign for a sharp sign.) Georgia guy (talk) 00:19, 17 September 2024 (UTC)[reply]

It's a shame because it's an incredibly intuitive concept once it's explained the right way: it's just the note that feels like "home" for all intents and purposes! Find the note that sounds okay being hummed throughout, and that's probably the tonic! Remsense ‥  00:51, 17 September 2024 (UTC)[reply]
Remsense, look at the melody I wrote in the talk page of Hail to the Chief back in late 2004. I'm sure the tonic is G. (If you look at lower comments in the same section you'll see someone saying information implying that D is the tonic.) Georgia guy (talk) 01:12, 17 September 2024 (UTC)[reply]
Apologies to you as a Georgia guy, but the example du jour of this has been Sweet Home Alabama, though the reasoning is at least because the chord progression seems like it outlines G (D 〃 C G → V 〃 IV I) instead of D (I 〃 ♭VII IV) to some. Remsense ‥  01:16, 17 September 2024 (UTC)[reply]
@Georgia guy, I'm not exactly sure what you're getting at. Are you saying that File:Hail to the Chief Chorus Sheet Music.png is in G?

I'm not sure how that could be (by the way, you would be arguing it's in G Lydian). It starts with a strong I–V–I that rather firmly establishes D as the tonic—you have to look at the entire harmony to discern the key, not just the melody. In any case, it would be rather untypical to start with the leading tone. The harmonies rather squarely fit into what'd we'd expect from a piece in D, with really no exceptions. Aza24 (talk) 03:21, 17 September 2024 (UTC)[reply]
The melody recorded with note names in Talk:Hail to the Chief § Melody and the First Voice seen in the score at File:Hail to the Chief Chorus Sheet Music.png are not in the same key. The melody rises stepwise to the note sounded at "Chief", the fourth syllable of the text (not counting the two-bar intro "Hail! Hail!"). This note is the tonic. On the talk page of Hail to the Chief this is a G; in the printed score it is a D. The Bass Voice in the score is stubbornly D-D-D-D-D-D-D-D throughout the initial "Hail to the Chief who in Triumph ad-". This is as sure an indication of the tonic as one might hope to get from the music itself. The key signature of the score is also that of D major.  --Lambiam 08:25, 17 September 2024 (UTC)[reply]
Aza24, the song (like most songs) can be in any key; the key depends on how the song is arranged. Here is the first line of the song in each key:
  • D major: A-B-C-D-C-B-A-B-A-F-E-D
  • G major: D-E-F-G-F-E-D-E-D-B-A-G. Lambiam, what notes (assuming the song is in G major) are the notes "Hail! Hail!" that make up the 2-bar intro?? (Also please note that a few years later, someone re-wrote the melody, also on the talk page but in a lower section, in a different key with a description that [if correct] would imply that the melody that I wrote on the talk page was in D; it would imply that the printed score is in A. They were using the argument that a song's first note is likely its tonic.) Georgia guy (talk) 10:18, 17 September 2024 (UTC)[reply]
    Each of the two initial Hail!s takes up a full bar of four beats. Looking at all three voices, the first is D·F♯·D and the second A·E·C♯. Although the first is not a full triad I interpret this as the progression I–V, which is followed by D·F♯·A, unambiguously I. Melodically, A–C♯–A wouldn't have worked well; D–C♯–A is much better.  --Lambiam 16:04, 17 September 2024 (UTC)[reply]
    Lambiam, please note what this discussion is intended to be about. Look further down the talk page (below where I put the melody in G major) and you'll see what I mean. You'll see a comment made by an IP who said something that if it were true, it would imply that the way I put the melody at the talk page (which is in G) was in D. Georgia guy (talk) 16:09, 17 September 2024 (UTC)[reply]
    What is the question? If you want me to comment on a comment by someone who commented on your comment, could you be more precise than "further down the talk page", such as indicating in which thread by which IP when?  --Lambiam 16:32, 17 September 2024 (UTC)[reply]
    Lambiam, look down the talk page for a post dating to late 2008 by the IP 90.24.229.69. (The 66. user who put the notes to the song in G major in late 2004 was me before I got a Wikipedia user name on January 1, 2005.) Georgia guy (talk) 16:34, 17 September 2024 (UTC)[reply]
    The post starting with "Wrong image" then. Could you remind me what the question is?  --Lambiam 18:59, 17 September 2024 (UTC)[reply]
    Lambiam: Yes, look at what someone wrote just after the words "Wrong image". The IP put the melody in a different key but claimed it was in the key that would be equivalent to the statement that the melody I put on the talk page in 2004 was in D major. Please read it. Georgia guy (talk) 19:07, 17 September 2024 (UTC)[reply]
    I did read it. Now what is your question?  --Lambiam 19:44, 17 September 2024 (UTC)[reply]
    Now, the user appears to be thinking that their set of notes of "Hail to the Chief" is in F, not B. This is a mistake. I want to know if this is a common mistake. Georgia guy (talk) 20:18, 17 September 2024 (UTC)[reply]
    It is not a common mistake among people who know something about Western music theory. It is also not a common mistake among people who know nothing about Western music theory and therefore refrain from making statements about what key something is in. But then there are some people who know nothing about Western music theory and yet are happy to make pronouncements that only display their ignorance. I have no material on how common this is for this specific type of error.  --Lambiam 22:31, 17 September 2024 (UTC)[reply]
    Infralapsarianism infiltrates inter-disciplinarily. -- Jack of Oz [pleasantries] 20:20, 18 September 2024 (UTC)[reply]

Trump denied security clearance?

[edit]

Suppose Donald Trump is inaugurated next January. Is there any way he could be denied any security clearance or information, due to his criminal convictions and so on? Could there be any restrictions that he could not overturn? Hayttom (talk) 12:21, 17 September 2024 (UTC)[reply]

Nope. --Golbez (talk) 13:07, 17 September 2024 (UTC)[reply]
The president can constitutionally declare that there is an insurrection and, using the powers of the Insurrection Act, order the military to arrest their opponents. They need not involve Congress. If this doesn't work as planned, it can only be because of insubordination  --Lambiam 16:14, 17 September 2024 (UTC)[reply]
Are acts of the president susceptible to Judicial Review? 2A02:C7B:223:9900:A88D:8EE5:E75B:3C1A (talk) 16:31, 17 September 2024 (UTC)[reply]
By a recent ruling of the Supreme Court, the president enjoys absolute immunity for official acts, which this would be. In light of this, the question is purely theoretical. There is no way that SCOTUS, if not already arrested, would seek to review the acts (and if they do, the president can have them incarcerated too).  --Lambiam 16:43, 17 September 2024 (UTC)[reply]
First, it's presumptive immunity for official acts, and absolute for so-called core acts of the office. The actual status (both legal and practical) of the notion of arresting/harassing/killing political opponents was disputed at the day of the ruling; I don't think anybody has seriously brought up the notion that other branches of government can be extralegally rounded up. Also, according to the Trump v US article you linked (but I didn't read the source and I probably don't understand it), Justice Jackson argued that legislative impeachment powers were reduced relative the judiciary in checking executive abuses of this nature after this ruling. So I'm guessing this is all way more complicated than all this, even if just theoretical. SamuelRiv (talk) 19:09, 17 September 2024 (UTC)[reply]
Actually only Golbez (talk) seemed to understand my question (although they were not very generous with elaboration) so I will try to ask it better: could any institution like the CIA withhold (or try to withhold or at least demonstrate going through the motions of withholding) a president's security clearance on grounds such as their criminal history? Hayttom (talk) 17:47, 17 September 2024 (UTC)[reply]
Nope. I mean, I suppose employees there could try, but they would be failing their job and thus should be fired. --Golbez (talk) 18:27, 17 September 2024 (UTC)[reply]
According to any source online, no conviction jeopardizes the president's security clearance because he doesn't have any (and keep in mind he gets classified briefings still, and would again be automatically granted them now as the major party's nominee, and his suitability to receive them even came up as an issue in 2016.) SamuelRiv (talk) 19:13, 17 September 2024 (UTC)[reply]
(Edit conflict): Addendum to my above: I don't know to what extent this is entirely norms, or norms made legal by default, just like there there ain't no rules says a dog can't play basketball. SamuelRiv (talk) 19:24, 17 September 2024 (UTC)[reply]
I think the 2016 source refers to presidential candidates. Hayttom (talk) 19:20, 17 September 2024 (UTC)[reply]
Yes. Once nominated, the major party candidates get classified briefings. As do presidents. As do ex-presidents, for life. SamuelRiv (talk) 19:27, 17 September 2024 (UTC)[reply]
(I believe I remember George H. W. Bush being quoted a few years before he died that he was no longer in the loop. But I imagine that was by his own request.) Hayttom (talk) 20:06, 17 September 2024 (UTC)[reply]
OK, so we are being told that Trump (as an ex-president, as a candidate, and in the terms of my question, a president) cannot be denied classified briefings. Hayttom (talk) 20:06, 17 September 2024 (UTC) [reply]
Resolved

Buddhist monks and nuns theravada mahayana vajrayana

[edit]

Is there website where they show Buddhist monks and nuns of Theravada, Mahayana and Vajrayana look like and dress like? Donmust90 Donmust90 (talk) 18:24, 17 September 2024 (UTC)[reply]

You can do a web search for images of members of those schools. You can also see images in our articles at Theravada, Mahayana and Vajrayana. However, it's not as simple as that as their dress depends which country they are in and which particular branch of those schools they belong to. Shantavira|feed me 08:15, 18 September 2024 (UTC)[reply]

September 19

[edit]

Plan Tamaulipas

[edit]

I am editing Hurricane Francine and I came across a Mexican organizations known as plans (Tamaulipas and DN-III). I am not sure what they are and I don't know how to research it as I do not speak Spanish (especially not Mexican Spanish). ✶Quxyz 00:32, 19 September 2024 (UTC)[reply]

The reason that the subject line is for specifically Tamaulipas was because I changed the focus of the topic of this request midway through upon realizing the DN-III wikilink goes to DN-III-E which I am not sure is the same. ✶Quxyz 00:36, 19 September 2024 (UTC)[reply]
Plan Tamaulipas: A New Security Strategy for a Troubled State, published by the Wilson Center. Plan DN-III-E , Civil Relief and Aid Plan for Disasters, which is Annexe E of Plan DN-III (National Defence Plan No. III). DuncanHill (talk) 10:08, 19 September 2024 (UTC)[reply]

Medusa with a snake body

[edit]

What was the earliest known depiction or attestation of Medusa (or any other Gorgon) being described as having a snake's body from the waist or hips down? I remember being told by someone at a younger age that sometime after the Roman Republic's era was possibly the first time that Medusa with a snake body was first told, but that depiction remained an uncommon thing until Ray Harryhausen's 1981 film Clash of the Titans permanently cemented the "snake-lower-half" look over the "ugly monstrous woman" look as the default imagery that comes to mind in modern popular culture whenever someone thinks of the names "Medusa" or "Gorgon". I would like to know if there is any merit to this claim; was it invented later-but still long ago such as in Renaissance or Early Modern times, or was it purely an invention by Harryhausen that everyone just latched onto?

And before anyone brings it up, yes, I am aware that some of the oldest known physical descriptions of Medusa were inconsistent between writers of antiquity, with the most descriptive being that of a hideously ugly woman with brass hands, brass or golden wings, boar tusks, writhing snakes for hair, and a stretched tongue, as depicted on surviving pottery and atop the temple to Artemis at Corcyra. I also understand that 3500-or-so years of orally passing down the same tale is bound to create some changes and mutations to the original telling along the way; for example, the hideous monster look started to be shed away in favour of just being a young woman who happens to have living snakes for hair sometime during the Renaissance.

Again, what I am asking for is the earliest known or surviving Medusa description that has her with a snake body with snake-hair, as is common in modern culture now, rather than as just an ugly woman with snake-hair. 72.234.12.37 (talk) 10:53, 19 September 2024 (UTC)[reply]

Peace Day 1919

[edit]

I am putting together an article for Peace Day 1919 on 19 July of that year, which was celebrated in London with a large military parade, sometimes described as "the London Victory Parade". I am having trouble finding the exact route of the parade. I know that the saluting base was outside Buckingham Palace and that the route included Lutyen's temporary Centaph in Whitehall. In Category:1919 London Victory Parade [8], there are photos of a rehearsal marching from Buckingham Palace to the Tower of London and other photos of troops and tanks crossing Westminster Bridge. Any further help would be greatly appreciated. Alansplodge (talk) 11:26, 19 September 2024 (UTC)[reply]

"a seven- mile route that began at the Albert Gate to Hyde Park, turned south to cross the river, winding through London south of the Thames, before eventually returning across Westminster Bridge, past Parliament and Big Ben and turning north onto Whitehall, where the temporary Cenotaph had just been unveiled, into Trafalgar Square and onto the Mall, past the Victoria Memorial where King George V and the royal party would take the salute, then along Constitution Hill to the finish back in Hyde Park". Will look further later. DuncanHill (talk) 11:32, 19 September 2024 (UTC)[reply]
Of course, 1 Albert Gate was and still is the French Embassy, a fitting starting point. DuncanHill (talk) 11:45, 19 September 2024 (UTC)[reply]
Page 3 of the Daily Mirror of 17 July 1919 has: "The line of route is:- From Kensington Gardens by way of South Carriage Road (Hyde Park), Albert Gate, Knights- bridge, Sloane-street, Pont-street, Chesham-place, Belgrave-square South, Upper and Lower Belgrave-streets, Buckingham Palace-road, Victoria- street, Vauxhall Bridge-road, Vauxhall Bridge, Upper Kennington-lane, Kennington-road, Lambeth road, Westminster Bridge, Bridge-street, Parliament-street, Whitehall, Charing Cross, Admiralty Arch, The Mall, Constitution Hill, Hyde Park- corner, Apsley Gate, Hyde Park to Kensington Gardens." - Dumelow (talk) 11:49, 19 September 2024 (UTC)[reply]
Page 16 of The Times for the date of the parade has a map and anticipated timings for each location. These should be available from newspapers.com via WP:TWL - Dumelow (talk) 11:53, 19 September 2024 (UTC)[reply]
The parade is covered in some of the books I used when I took The Cenotaph to FA. See the "war memorials" and "social impact of WWI" sections of User:HJ Mitchell/Library. If you started an article, I'd contribute what I could or if you were looking for something specific I could check the books but I have limited time until after the weekend. HJ Mitchell | Penny for your thoughts? 12:13, 19 September 2024 (UTC)[reply]
Thanks all. It seems that I was being confused by a "Dominion Victory Parade" through the City on 3rd May 1919. [9]
User:HJ Mitchell, many thanks for the offer. I'm at the stage of gathering references and getting my head around it, I'll get back to you when things have progressed. Alansplodge (talk) 19:52, 22 September 2024 (UTC)[reply]

Carrie Chapman Catt's puzzling get-up

[edit]

Here we have a picture of Carrie Chapman Catt (on the right, I think) and Anna Howard Shaw. Dr Shaw is wearing her doctoral gown. What on earth is Ms Catt wearing? Marnanel (talk) 15:12, 19 September 2024 (UTC)[reply]

Could it be her "ratification dress" referred to in the article? Unfortunately I haven't been able to find any other images of it, though it was said to be sapphire blue rather than white. Shantavira|feed me 16:14, 19 September 2024 (UTC)[reply]
During women's suffrage demonstrations, women often wore white dresses. It isn't likely a special type of dress. It is white, which is a symbolism they wanted. You can find many images of women's suffrage marches where nearly all of the women are wearing white dresses. This document begins with another view of the march where you can see many other women dressed in white. 75.136.148.8 (talk) 17:26, 19 September 2024 (UTC)[reply]
The woman on the right is wearing a cape with a flag design below the left shoulder, but what's underneath the cape seems to be an ordinary semi-formal 1917 white dress, as far as can be seen... AnonMoos (talk) 17:50, 19 September 2024 (UTC)[reply]
It's the costume of Super Suffragette. To her right is her obligatory sidekick, Ballot Woman. Clarityfiend (talk) 03:16, 22 September 2024 (UTC)[reply]

September 20

[edit]

Split from BNP 2001-2006

[edit]

I am just curious: how many parties split from Bangladesh Nationalist Party (BNP) during their term from 2001 to 2006? So far, I know that there were two: Liberal Democratic Party lead by Oli Ahmed and Bikolpodhara party lead by Dr. badrozzoha or what ever his name was. 2607:FEA8:55E2:8B00:49D0:D87A:E12A:81A2 (talk) 01:45, 20 September 2024 (UTC)[reply]

Falkland Islands pound

[edit]

The article Falkland Islands pound mentions they use their own currency there, but scotland technically has its own too but they use regular GBP. Can brits use a GBP card without currency conversion (like scotland)? I understand the unlikelihood of people having been there, but was curious. The same would go for the dutch visiting the carib islands. St Maarten accepts ECD, but I dunno if the southern ones use euros alone? Sportsnut24 (talk) 05:37, 20 September 2024 (UTC)[reply]

Scotland does not technically have its own currency. Some Scottish banks issue their own notes, which are fully backed by the Bank of England. You can use GBP in the Falklands. Different parts of the Dutch Caribbean use the Netherlands Antillean guilder and the US Dollar, although it is likely many places will accept euros. CMD (talk) 06:27, 20 September 2024 (UTC)[reply]
Thanks. I was wondering if GBP card in Stanley would work directly or have currency conversion?Sportsnut24 (talk) 08:30, 20 September 2024 (UTC)[reply]
I know they do in some other UK territories, but always best to check directly with your bank. More importantly, if someone gets Falkland Islands cash, they shouldn't expect it to be useful in the UK. CMD (talk) 10:24, 20 September 2024 (UTC)[reply]
I know Gibraltar is fine for GBP. Not been, but I'd use dollars, so it wouldn't matter anyways.Sportsnut24 (talk) 05:47, 21 September 2024 (UTC)[reply]
32 years ago when I went to Gibraltar, I could spend my British pounds all right, but when I then withdrew from an ATM it dispensed Gibraltar pounds, which were worth about 10% less when exchanged at a bank in England. --142.112.148.3 (talk) 18:25, 24 September 2024 (UTC)[reply]
I got a mail from FI that said there is one ATM there and it gives FI pounds but is 1:! with the UK.2A00:F3C:A282:0:1C8B:D2D1:9271:D869 (talk) 16:45, 26 September 2024 (UTC)[reply]

Egyptian zodiac

[edit]

How many of these are there? There's the famous Dendera one, there's a recently re-uncovered one in Esna that's rather -- shall we say very not traditional-formal -- there's one forgotten almost as soon as Petrie illustrated it in Athribis, and there's the pictured one, Belzoni's from Thebes. Which I'm not immediately sure if it is a Zodiac. I'd like there to be a page on these, but I don't know if there are synthetic or contextualizing sources on them like there are few but easily available for Zodiac synagogue mosaic. Temerarius (talk) 22:30, 20 September 2024 (UTC)[reply]

Are they all from the Hellenistic or Roman period? AnonMoos (talk) 01:30, 21 September 2024 (UTC)[reply]
That's something I'm trying to figure out. And in Dendera's case the dating was so contentious, apparently, that a whole "affair" was declared about it. Baron G Cuvier's 1831 account there mentions the possibility of Ense being a "wholly Mesopotamian zodiac" if that's not a self-contradiction. I'd thought the temple had only been found for the first time recently. You can see some pictures here. https://english.ahram.org.eg/News/491999.aspx You can see the artist was familiar with Egyptian formalism but not beholden to it, they're careful yet cartoonish, literate but not respectful of hieroglyphs, not even bothering to put them on a grid or in relief. It's like they wanted the paint to do all the work for them, perhaps a painter before a carver. The simplicity of their smiles make them look child-made. In other words, the oddness makes it seem unquestionably late. Ptolemaic weirdness perhaps. If Mesopotamia inspired the content, it didn't influence the style. Athribis is more pleasant; naive but native.
Temerarius (talk) 03:16, 21 September 2024 (UTC)[reply]
What's with all the black dots like ⬤ on the bottom-line figures? Is there a known explanation?  --Lambiam 08:23, 21 September 2024 (UTC)[reply]
The white black dots? Those are sun-disks, but the closest article I could find is solar symbol. [Edit: maybe Eye of Ra.] The slightly different one on the right is between cow horns and belongs to Hathor, or maybe Isis. Of course Ra is depicted with a sun on his head, but here it seems every god gets a sun-disk, maybe because the date is late and Ra#worship has increased? This is from the Tomb of Seti I, and the figures are made of gold. I guess the nine on the right are the Ennead? Not sure what happened to Osiris, in that case, and I don't know about the eleven on the left but I can identify Thoth (by the beak).
Here's a description of how the clutter in the middle is a zodiac. It's being compared to the Astronomical ceiling of Senenmut's Tomb.  Card Zero  (talk) 09:03, 21 September 2024 (UTC)[reply]
No, by "black dots" I mean black black dots, "black" as in "the colour black", a colour that results from the absence of light, like the colour of this dot: ⬤. Most figures on the bottom line, especially in the left half, display eight or more black dots, in many cases one on each of their shoulders, one on each of their wrists, one on each of their ankles, and two on the bottom edges of their skirts.  --Lambiam 22:04, 21 September 2024 (UTC)[reply]
Oh you mean the black black dots. I missed those. I wonder what they are? In photographs they're brown, the same as the paint used for most of the lines on the figures. Obvious guess: stars? But that's a terrible guess, considering how many are lined up in neat rows. However ... the bull apparently represents the Big Dipper, and if we allow that the nameless man holding the chains is part of it, he has four dots in an oblong, which seem to match up.  Card Zero  (talk) 08:07, 22 September 2024 (UTC)[reply]
Oh, the red brown black black dots. Might be stars, just have to count them. If not, they're probably structural not decorative inlay points, if the figures were separate material ("made of gold".) Not saying I've seen an example like that before though.
Temerarius (talk) 17:57, 22 September 2024 (UTC)[reply]
waiting now for orange red brown black black dots, then yellow orange red brown black black dots … —Tamfang (talk) 19:23, 24 September 2024 (UTC)[reply]
Card Zero, good one! Your source says there are 24 Egyptian zodiacs, where some say three. Too bad it doesn't list them.
Temerarius (talk) 02:56, 22 September 2024 (UTC)[reply]
It also says they're a Greek period thing. So I guess these older "astronomical ceilings" don't count, despite being constellations, and despite the zodiacs being mostly on ceilings or inside coffin lids. Define zodiac, I don't know, I guess it has to be an approximation to the familiar set of constellations and not some earlier set.  Card Zero  (talk) 08:40, 22 September 2024 (UTC)[reply]
Yeah, what exactly counts as a Zodiac and are they definitionally Greek-derived are questions that come up. I'm gonna reread S Langdon's "Babylonian menologies and Semitic calendars" to see how specifically he uses the word. https://archive.org/details/babylonianmenolo0000step That's a good source that collects many deeply intriguing details and connections. Of course, as any work highly synthetic it requires a piecemeal not wholesale comprehension.
Temerarius (talk) 17:50, 22 September 2024 (UTC)[reply]
Searching the tomb name gives the article Tomb of Seti I, and Commons has the color photo which says that the dots are painted red and are the actual stars in the (painted) constellations. (I had initially thought they'd be for inlays as well.) SamuelRiv (talk) 01:01, 23 September 2024 (UTC)[reply]
Petrie worked at the southern Athribis (Upper Egypt), see Athribis, pp. 12-13, plates pp. 66ff. He says that the tombs are of late date, no earlier than Ptolemaic. MinorProphet (talk) 13:00, 22 September 2024 (UTC)[reply]
Thanks. I'll add the plates to that page.
Temerarius (talk) 22:16, 22 September 2024 (UTC)[reply]
There was a long post that was a bit hard to follow but didn't immediately look like trolling to me, what was the matter with it? Was that a known troll?
Temerarius (talk) 23:40, 24 September 2024 (UTC)[reply]
Yeah, general word of thumb is if you see a large amount of text appearing and being reverted on here, it's probably VXFC. GalacticShoe (talk) 00:03, 25 September 2024 (UTC)[reply]
A thumb doesn't have words. The words are in the index.  Card Zero  (talk) 06:22, 25 September 2024 (UTC)[reply]
Looks like I accidentally coined a phrase, it does happen thumbtimes... GalacticShoe (talk) 06:57, 25 September 2024 (UTC)[reply]

September 21

[edit]

The "bird famine of 1880"

[edit]

Referred to in Poems (1894), at that page and the next, as an easily recognisable event. I assume it's some kind of crop failure, but I haven't been able to find anything about that (all sites in a quick search quote from the book). Any ideas? — Alien  3
3 3
15:07, 21 September 2024 (UTC)[reply]

I see a bird famine is a harsh winter where birds can't find food (there are other references, so it was a term with currency). And here we have a description of birds struggling to find food during the unusually harsh winter of 1880.  Card Zero  (talk) 15:55, 21 September 2024 (UTC)[reply]
Thank you, question answered. — Alien  3
3 3
16:42, 21 September 2024 (UTC)[reply]
Yes, probably, but the book ref refers to Hampshire, England, and the poems are presumably by Sarah Morgan Bryan Piatt, who didn't come to Europe (Ireland in fact) until 1882. I suppose it might have been a transatlantic thing. Johnbod (talk) 18:38, 21 September 2024 (UTC)[reply]
See the etymonline entry for blizzard. "it came into general use in the U.S. in this sense in the hard winter of 1880-81." Oh, and of course our article, Hard Winter of 1880–81.  Card Zero  (talk) 19:50, 21 September 2024 (UTC)[reply]
(We do have definitive confirmation that Sarah Piatt is Sarah Morgan Bryan Piatt.)Alien  3
3 3
09:35, 22 September 2024 (UTC)[reply]
According to this article, in 1880 in the south of England, it snowed unusually late in April and May (when birds would be nesting) and unusually early in October. Alansplodge (talk) 13:02, 23 September 2024 (UTC)[reply]
I had a fun time investigating the extent of the winter of 1880-81, which included rumours of wolves in a park in Paris and the destruction by frost of a famous pine forest near Ravenna [10][11], though it was a mild winter in Turkmenistan.  Card Zero  (talk) 16:07, 23 September 2024 (UTC)[reply]
Ignore that, the poem refers to a "red-bird" which is apparently a northern cardinal, setting it firmly in the United States. Alansplodge (talk) 14:32, 23 September 2024 (UTC)[reply]

September 23

[edit]

Margarethe von Helfenstein

[edit]

I have a problem at Margarethe von Helfenstein, when did she die? (I will copy any answers to the talk page there.) Margarethe was the illegitimate daughter of Maximilian I, Holy Roman Emperor, sister of George of Austria and wife of de:Ludwig V. von Helfenstein-Wiesensteig, who was killed in 1525. So far, there are three different answers in the sources

  • The Belgian Nouvelle Biographie Nationale says "morte en Allemagne après 1531" [died in Germany after 1531].[1]
  • Deutsche Biographie's article on Maximilian I says she died in 1525,[2]
  • Some German sources say she died in 1537 in Liège, e.g. "nach dem Tod von Ludwig Helferich zog die Witwe in die Niederlande, nach Lüttich, zu ihrem Bruder Georg von Österreich, Fürstbischof von Lüttich (Georg/Joris war der illegitime Sohn von Kaiser Maximilian I. und Margareta von Edelsheim), starb dort 1537" [After the death of Ludwig Helferich, the widow moved to the Netherlands, to Liège, to her brother George of Austria, Prince-Bishop of Liège (Georg/Joris was the illegitimate son of Emperor Maximilian I and Margareta of Edelsheim), died there in 1537.][3] "Nach 1515 heiratete Graf Ludwig von Helfenstein , der 1525 in Weinsberg ermordet wurde , Maximilians ältestes uneheliches Kind Mar- garethe ( geb. 1480 , gest . 1537 Lüttich )" [After 1515, Count Ludwig von Helfenstein, who was murdered in Weinsberg in 1525, married Maximilian's eldest illegitimate child, Margarethe (born 1480, died 1537 Liège)][4] Oddly George of Austria became Prince-bishop of Liège in 1544.

References

  1. ^ Coenen, Daniel (1999). "de Helfenstein, Marguerite". Nouvelle Biographie Nationale – Volume 5 [Margarethe von Helfenstein] (PDF) (in French). Royal Academy of Science, Letters and Fine Arts of Belgium. pp. 92–93.
  2. ^ "Maximilian I". Deutsche Biographie (in German). Retrieved 23 September 2024.
  3. ^ Naubert, Christiane Benedikte (29 February 2016). Der Bund des armen Konrads: Getreue Schilderung einiger merkwürdigen Auftritte aus den Zeiten der Bauernkriege des sechszehnten Jahrhunderts. (Transkription von Evelyn Hess) Neu herausgegeben, mit Fußnoten und einem Nachwort versehen von Sylvia Kolbe (in German). Engelsdorfer Verlag. p. 350. ISBN 978-3-96008-342-9. Retrieved 23 September 2024.
  4. ^ Tagung, Lehrstuhl für Allgemeine Geschichte des Mittelalters und Historische Hilfswissenschaften in Greifswald Interdisziplinäre (2002). Principes: Dynastien und Höfe im späten Mittelalter : interdisziplinäre Tagung des Lehrstuhls für Allgemeine Geschichte des Mittelalters und Historische Hilfswissenschaften in Greifswald in Verbindung mit der Residenzen-Kommission der Akademie der Wissenschaften zu Göttingen vom 15.-18. Juni 2000 (in German). Thorbecke. p. 283. ISBN 978-3-7995-4514-3. Retrieved 23 September 2024.

TSventon (talk) 13:49, 23 September 2024 (UTC)[reply]

@Deamonpen, Dimadick, Aciram, and Tfjt: TSventon (talk) 13:51, 23 September 2024 (UTC)[reply]

September 24

[edit]

Liliuokalani to Victoria

[edit]

There is a letter from Liliuokalani to Queen Victoria dated to January 31, 1893. I’ve found the return letter from Victoria but not the one sent by Liliuokalani. The citation in this source: Great Britain and the Hawaiian Revolution and Republic, 1893-1898, cites it to “Enclosure in Wodehouse, despatch to Rosebery, 1 Feb. 1893, FO 534/59” and quotes one line, “to avoid violence and bloodshed, and damage to my subjects”. This one gives another snippet from the letter: “friendly intercession and mediation“. Can someone help me find this letter in its entirety? KAVEBEAR (talk) 03:09, 24 September 2024 (UTC)[reply]

Here's the National Archives catalogue entry FO 534/59. The letter might also be pp. 41-2 of FO 58/270. fiveby(zero) 03:37, 24 September 2024 (UTC)[reply]
Anyone with access to the source and can scan it? I submitted a Wikipedia:WikiProject Resource Exchange/Resource Request#Letter to Victoria from Liliuokalani in case anyone there can get the resource as well. KAVEBEAR (talk) 04:11, 24 September 2024 (UTC)[reply]
Tate, Merze (1962). "Great Britain and the Sovereignty of Hawaii". Pacific Historical Review. 31 (4). cites as FO 58/279 "Designs of the United States on Hawaii. Volume 2" which may be more complete and include foreign office notes for the reply (note Tate says "...Victoria opened the Queen of Hawaii's letter and returned it to the foreign office without comment. Since an acknowledgement and a reply of some sort to be sent, the undersecretaries in that office decided on one "with padding" to "the effect that the Queen had received the letter had referred it to her advisor.") fiveby(zero) 04:21, 24 September 2024 (UTC)[reply]

Gordian coin with two scripts

[edit]

https://postimg.cc/9DPF5fd5 This coin reads clearly enough IMP GORDIANUS PIUS, then immediately goes into another script that looks kind of Semitic, but I can't make sense out of it. Any ideas? If you can't read that, I can upload the short video clip where it's more legible--but where? The site where I uplaoded the image doesn't take videos. Temerarius (talk) 23:09, 24 September 2024 (UTC)[reply]

I found numerous examples of Gordian III coins inscribed with IMP GORDIANVS PIVS FEL AVG and the letters on this one, assuming it is but one script, are perhaps too worn to make out properly. Modocc (talk) 00:42, 25 September 2024 (UTC)[reply]
A nice one. --Modocc (talk) 00:52, 25 September 2024 (UTC)[reply]
God, how strange! It does say FEL AUG. But it's copied so poorly it's like just the last bits were done by an illiterate. It's a different die from that gold one. The engraver seems to be splitting the difference at confusion over whether the bit under the P is headband, radial, or knot.
Temerarius (talk) 01:13, 25 September 2024 (UTC)[reply]
Its quite possible that it was indeed done by an illiterate. As the Roman Empire declined, new coins minted in outlying portions of it under semi- or entirely autonomous local rulers naturally tried to copy older coins, but the engravers were sometimes not literate and had little idea of the 'correct' (letter) forms within the designs they were copying, and sometimes didn't fully understand what the 'pictures' represented (or lacked the skill to reproduce them well) so in time copies of copies of copies could degenerate into almost abstract and unrecognisable forms. This kind of 'devolution' can be seen both post-Roman coins and also coins from other cultures in Europe.
We ought to have something describing this in an article, but I haven't been able to find one. {The poster formerly known as 87.81.230.195} 94.1.171.3 (talk) 17:56, 25 September 2024 (UTC)[reply]
From Offa of Mercia#Coinage: There are also surviving gold coins from Offa's reign. One is a copy of an Abbasid dinar struck in 774 by Caliph Al-Mansur,[117] with "Offa Rex" centred on the reverse. It is clear that the moneyer had no understanding of Arabic as the Arabic text contains many errors. Also local copies of Spanish dollars or thalers are probably done by people who did not understand the originals. --Error (talk) 23:42, 25 September 2024 (UTC)[reply]
Here's a blog post showing evolution from a Roman stater to some sort of cubist portrait with a horse with three tails on the reverse.  Card Zero  (talk) 06:06, 26 September 2024 (UTC)[reply]

September 25

[edit]

Sword fight trope in movies

[edit]

Does anyone know where this originated? The hero and villain face each other with swords from a few meters apart. Then they both run towards each other and strike at the same time with their swords. They run past each other and there is a moment of silence where it is unclear what happened. Then the bad guy starts spurting blood and falls over dead. This is a recent example from a movie I saw tonight, but there have been many more. I think it's quite common in anime. 146.200.126.178 (talk) 01:03, 25 September 2024 (UTC)[reply]

It certainly dates back to the silent film era, but I would expect it to originate prior to that in stage combat. Dekimasuよ! 01:37, 25 September 2024 (UTC)[reply]
you may be correct, but I did think it was a trope from Japanese cinema, perhaps as old as you said. I've definitely seen it in Japanese movies and anime over the years, but my memory fails me now. In Kill Bill, Beatrix kills O-Ren in this manner (she slices off the top of her head after a duel) - and I know Tarantino made his film as a homage to Japanese samurai movies. 146.200.126.178 (talk) 02:11, 25 September 2024 (UTC)[reply]
In Japan as well I would expect it to go back to stage combat and through silent movies, although in early Japanese cinema the hero was more likely to fight against a large group of enemies at once. Off the top of my head there is a prominent example in one of Seiji Miyaguchi's scenes in Kurosawa's Seven Samurai (1954, short cut here). Dekimasuよ! 02:31, 25 September 2024 (UTC)[reply]
Off topic perhaps, but the Seagal scene is choreographed really badly, isn't it? At 1:35 we see Seagal from the perspective of the person he is fighting, swinging directly across from the left side of the screen. Seagal's sword jumps from a "down" position to an "up" position at 1:37 (with no blood on the sword). At 1:38 we appear to see Seagal swing the sword directly downward, but by this point the person Seagal's fighting is bleeding from the wrong side, apparently without his shirt being cut. At 1:45 Seagal's sword suddenly has blood dripping from it in massive amounts. Based on the character movements, the person Seagal was fighting against never even swung his sword, simply running straight past—although his arm also jumps from a "down" position to an "up" position at 1:38. At 2:17, the opponent falls dead with his face to the right. At 2:19, his face is to the left. Dekimasuよ! 02:47, 25 September 2024 (UTC)[reply]
The running towards each other is perhaps most famously (influentially) done in Akira Kurosawa's Seven Samurai, in one of the early "recruitment" scenes. It's a samurai movie trope (it may predate it or be used in other genres, I don't know, but everyone knows Kurosawa), and it looks like in the clip they're recreating a number of the shot setups of Kurosawa.
There are several other stage combat tropes being done there, which are quite out of place (even for stage combat purposes). There usually has to be a reason for doing things, in a martial art, a sport, or in stage combat for theater. So randomly switching between different (better) films' of different genres' fight scene styles in the middle of a few minutes, if not a parody (and with Steven Seagal, who can know?) is rather jarring to anyone. That's why you saw it and thought to yourself, "something's very wrong". SamuelRiv (talk) 02:51, 25 September 2024 (UTC)[reply]
On the "silence" bit -- one thing I've noticed about stage combat (I've only done sport and martial arts, but this is from friends and teachers who did theater and some online info) is that a lot of form follows function as an excuse to talk. So in sword fights, the actors will find moments to get up real close and start shouting in each others' faces (The Force Awakens's final fight actually did this pretty decently), or else take a moment to stand apart and taunt each other (Empire Strikes Back did this extremely well). It's when a sword fight happens in complete silence, where the actors don't talk even when the trope would suggest they should, that the audience is signalled that this is beyond intense. Imo a perfect execution of this was in Rob Roy (1995 film), with the final scene being set up for the entire film. (Without that real emotional narrative buildup however, or some great investment where you think anything can happen (like sports), a silent fight scene in a film just gets boring.) (Addendum: this applies to stage and film combat. In sports and modern martial arts, none of that happens (except for weird artifacts of rules of sport fencing). As for a "real life swordfight", which people are asked about sometimes, nobody has been in one for generations, and a lot of the reconstruction is incomplete.) SamuelRiv (talk) 03:08, 25 September 2024 (UTC)[reply]
Real life sword fight. A version of this video (warning, violence - no one died, both idiots got arrested) went viral a few years ago and people were saying that this is probably the first real sword fight caught on video. Iloveparrots (talk) 09:05, 25 September 2024 (UTC)[reply]
Pokemon did it too. Not sure why that jumped into my head, but that's deffo a reference to something else. Iloveparrots (talk) 08:35, 25 September 2024 (UTC)[reply]
See also Diagonal Cut from tvtrpes.org. Alansplodge (talk) 14:58, 25 September 2024 (UTC)[reply]

Convention vs. unconventional

[edit]

If what is generally described as "good” art tends to be unconventional, which may also be described as original or new, which I personally believe is true, how does it avoid becoming conventional over time? Does art need to be in constant flux, changing its form in immeasurable ways, to stay one step ahead of convention? Is that which is conventional the enemy of art, or can it coexist and thrive alongside it? Viriditas (talk) 10:28, 25 September 2024 (UTC)[reply]

You will have to read a lot to get a satisfying answer. I would start with a search such as this: https://scholar.google.com/scholar?q=originality+art HansVonStuttgart (talk) 10:47, 25 September 2024 (UTC)[reply]
The notion that art should be original and therefore at least somewhat unconventional is relatively recent. Before the arrival of Modernism one century ago, Western art was governed by an aesthetic ideal of "beauty" that every art theorist defined in their own way. Innovations were valued only insofar as they brought the art closer to the unattainable ideal.
Just as fads in fashion are driven by a small group of fashion designers and critics, the notion of what is "good" (read, between the lines, "high-priced") art is also subject to fads driven by a small incrowd in the art world. People in this circle are keen to "discover" an unknown artist with a fresh, new, original, inspiring approach, preferably an artist in a small group of like-minded artists who are somewhat of a bunch of rebels, and to promote them – ignoring scores of other suffering artists with equally fresh, new, original and inspiring approaches. If this succeeds and the new star in the firmament shines bright and fetches high prices for their work, others jump on the bandwagon, and what once was original becomes unsellable imitation. It is time for the discovery of the next unknown artist with a fresh, new, original, inspiring approach.  --Lambiam 14:03, 25 September 2024 (UTC)[reply]
Mugato piano key necktie gif. Viriditas (talk) 20:14, 25 September 2024 (UTC)[reply]

"The explanation could explain only the masculine grammatical gender (víkingr) and not the feminine (víking); the masculine is more easily derived from the feminine than the other way around." I'm confused about this sentence. Can someone please explain? I can see both words víkingr and víking being derived from the word "víkin". Why the derivation only works on the masculine form? Thanks! 2600:6C44:117F:95BE:1850:9245:B082:62CE (talk) 10:32, 25 September 2024 (UTC)[reply]

Also, why the masculine form can be derived from the feminine form but not the other way around? I can easily imagine that víkingr and víking can both be derived from each other. I mean it makes sense in my head. What am I missing here? 2600:6C44:117F:95BE:1850:9245:B082:62CE (talk) 10:57, 25 September 2024 (UTC)[reply]

IP editor, have you read the reference for the sentence? It is fairly long and technical.
TSventon (talk) 14:14, 25 September 2024 (UTC)[reply]
I did spend half an hour reading it. It's too difficult for a layman person like me to understand what they're trying to say though. That's why I need an explaination for dummy. 2600:6C44:117F:95BE:EDA0:1EFB:E4CB:1F6A (talk) 04:23, 26 September 2024 (UTC)[reply]
IP editor, I think that Heide and his sources are arguing that there are examples of masculine words like víkingr being derived from feminine words like viking (according to researchers in Old Norse), but not the other way round. Three key quotes are on page 43 Askeberg says: "I do not know any example of a masculine ing-derivation having given origin to a feminine nomen actionis that expresses the person’s action, and such a formation seems unreasonable. A hildingr m. 'king' can not be supposed to have given origin to a *hilding f. 'the quality of being a king' etc" on page 44 Askeberg points out that deverbative ing-derivations are considered younger than the word víkingr, and that it is unlikely that feminine verbal abstracts in so early times could be formed from strong verbs, like víka. and 45 On the other hand, a masculine víkingr 'sea warrior' could well be derived from a feminine víking denoting an activity. Old Norse parallels to such a development would be vellingr m. 'pottage' from *velling f. 'boiling'; geldingr m. 'a castrated ox or ram' from gelding f. 'castration'; [etc.]. TSventon (talk) 15:42, 27 September 2024 (UTC)[reply]
TSventon I'm still confused. Why the word "víkingr" can be derived from the word "víkin" but the word "víking" can't? What they're saying is too technical for me to understand. I need an explanation for dummy. 2600:6C44:117F:95BE:E1D5:4325:36B7:C2A5 (talk) 04:48, 29 September 2024 (UTC)[reply]
"I do not know any example of a masculine ing-derivation having given origin to a feminine nomen actionis that expresses the person’s action" -> if I understand this correctly, this is saying that there is no example of a masculine noun having origin from a feminine verbal noun. This contradicts your claim above. I'm really confused now. 2600:6C44:117F:95BE:E1D5:4325:36B7:C2A5 (talk) 04:54, 29 September 2024 (UTC)[reply]
IP editor, I see you started a discussion at Talk:Vikings#Semi-protected edit request on 26 September 2024 so I have copied my response there. TSventon (talk) 17:37, 27 September 2024 (UTC)[reply]

Secret ballot used in determining guilt

[edit]

I'm currently reading Frankenstein for the first time (some of you might twig why).

The section (Vol. I, Chapter VII) where Justine is being tried for murder includes a reference to ballots: The ballots had been thrown; they were all black, and Justine was condemned. A footnote describes these ballots as "small balls used for secret voting". I'd love to hear more about this system. As an Aussie, I'm proud our name is sometimes attached to the secret ballot used for electing members of legislatures, and that result figures prominently in the searches I've done. Include "black" or "white" in the search terms, and I get lots of hits dealing with race as a factor in determining voter eligibility. Include "jury", and I get lots of hits for systems of choosing jurors. But nothing comes up for its use by juries themselves in criminal trials.

Our secret ballot article makes no reference to this. If Shelley's reference is historically accurate, it would considerably predate its use for electing politicians, and that's something we should definitely include in our article(s). The novel is set mainly in Switzerland, so perhaps this system did not obtain anywhere else. But then, how did an 18-year-old English girl get to know about it? And why would it be necessary anyway, since juries do their deliberations in secret. Or maybe that was different in some places at some times. Or maybe it was a panel of judges, rather than a jury in the 12 Angry Men sense.

Grateful for any information. -- Jack of Oz [pleasantries] 19:08, 25 September 2024 (UTC)[reply]

Our article entitled Blackballing should help. Blueboar (talk) 20:05, 25 September 2024 (UTC)[reply]
And ballot has a picture of "Ancient Greek bronze secret ballots", not sure if that predates electing politicians.  Card Zero  (talk) 20:09, 25 September 2024 (UTC)[reply]
@ Blueboar: Blackballing makes no mention of its application to determining the guilt or innocence of a person charged with a crime, which is my sole interest here. -- Jack of Oz [pleasantries] 23:55, 25 September 2024 (UTC)[reply]
(ec) A case from 1864, the Cantonal Parliament of Zurich voted on a motion for mercy in the case of a murderer. The members of the Cantonal Parliament had to deposit either a white (for mercy) or a black ball (against the motion) in an urn. DuncanHill (talk) 20:12, 25 September 2024 (UTC)[reply]
Diccionario panhispánico del español jurídico has procedimiento administrativo de votación mediante bolas blancas y negras. It mentions honor and church corporations. Actually, I remember seeing in a church museum some such device used for voting.
--Error (talk) 23:30, 25 September 2024 (UTC)[reply]
"[H]ow did an 18-year-old English girl get to know about it?" She (with Percy Shelley) spent the Summer of 1816 staying with Lord Byron in Geneva, where she got the idea for the work and began writing it. Presumably she, well educated by her father William Godwin, was intellectually curious and was able to learn something about the legal system of the country she was living in. {The poster formerly known as 87.81.230.195} 94.1.171.3 (talk) 02:14, 26 September 2024 (UTC)[reply]
This is a really interesting question! As far as I can work out, black and white balls were a known form of secret voting in general in the eighteenth century, and criminal trials with juries that vote secretly started to catch on in France with the Revolution. So those things combined into some jury trials using balls for their secret ballots. The best sources I can find on the topic are "The Advent of the Secret Ballot in Britain and France, 1789–1914: From Public Assembly to Private Compartment" and "Publicity and Secrecy in Jury Proceedings" (PDF download). I haven't read either very closely but they seem likely to tell you much more. If there's good stuff in there, do expand the relevant wiki articles with it! ~ L 🌸 (talk) 03:29, 26 September 2024 (UTC)[reply]
A couple of thoughts - in the story it is a panel of judges, and perhaps the balls were metaphorical. DuncanHill (talk) 12:03, 26 September 2024 (UTC)[reply]
Perhaps so. But the metaphor must have had its genesis in an actual practice. -- Jack of Oz [pleasantries] 17:43, 26 September 2024 (UTC)[reply]

September 26

[edit]

Adolf Uunona

[edit]

Why was the page about Adolf Uunona deleted? Just curious. Also would I be ok to just revive the page myself? Cornishrom20 (talk) 11:19, 26 September 2024 (UTC)[reply]

wp:deny
The following discussion has been closed. Please do not modify it.
Wikipedia:Articles for deletion/Adolf Uunona. Something strange going on here Wikipedia:Articles for deletion/Adolf Hitler Uunona.2A00:23D0:E1D:AD01:9952:C7AB:38E2:BF35 (talk) 11:55, 26 September 2024 (UTC)[reply]
Nothing strange going on at all, there was a "no consensus" AfD, the article was renamed, and then there was an AfD cloesed as delete. It has since been recreated repeatedly and speedily deleted as "Recreation of a page that was deleted per a deletion discussion". See the logs here. DuncanHill (talk) 12:06, 26 September 2024 (UTC)[reply]

I need someone to find me a source cus i cant find any

[edit]

i need sources on the civil parish abolishment of eastcotts cus im tryna update the cardington page cus it says the raf is in the "parish" of eastcotts 94.194.31.200 (talk) 18:01, 26 September 2024 (UTC)[reply]

[NB: Also asked, and answered, on the Help Desk. {The poster formerly known as 87.81.230.195} 94.6.86.81 (talk) 18:21, 26 September 2024 (UTC)][reply]
Try Parishes: Cardington with Eastcotts. Alansplodge (talk) 08:31, 27 September 2024 (UTC)[reply]

Free pentameter

[edit]

When a poem is written in free pentameter, what exactly does that mean? Amisom (talk) 19:54, 26 September 2024 (UTC)[reply]

In English, that would mean five main stresses per line, but few constraints as to where the stresses fall within the line. Old English alliterative verse sometimes approximated towards free tetrameter, though with constraints on the initial consonants of stressed words... AnonMoos (talk) 22:06, 26 September 2024 (UTC)[reply]
Just to build on that a bit: the pentameter part refers to there being five stressed syllables in the line and the free part refers to the line(s) not having a particular rhythm to it. This would be in contrast to meters like iambic pentameter (i.e. the form now often identified with Shakespeare), which detail the exact pattern of stressed and unstressed syllables. Matt Deres (talk) 19:10, 1 October 2024 (UTC)[reply]

September 28

[edit]

Consecration of Church of England churches

[edit]

According to our article Arthur Wagner "Wagner had a lifelong opposition to the consecration of Anglican churches, on the basis that this would "[give] an opening for the State to intervene in their affairs". This view was shared by many Tractarians. On one occasion he complained to Richard Durnford, Bishop of Chichester, that consecration was "a farce". Pusey supported Wagner in his attempts to leave his newly built churches unconsecrated, but to no avail". What opening to the State would consecration give, beyond that already provided by the established status of the Church? Are any CofE churches unconsecrated (as opposed to deconsecrated)? Thank you, DuncanHill (talk) 12:31, 28 September 2024 (UTC)[reply]
Courtesy links:
Church of England
Consecration
Consecration in Christianity
Edward Bouverie Pusey
Richard Durnford
Tractarians

I saw a similar argument about Keble College chapel. According to The Encyclopaedia of Oxford, in a characteristic attempt to keep the college out of the grasp of those whose views might be alien, the council refused to have the chapel consecrated, much to the fury of the then BISHOP OF OXFORD; it remains unconsecrated to this day.[1]: 207  TSventon (talk) 13:12, 28 September 2024 (UTC)[reply]
An example of state intervention was the Public Worship Regulation Act 1874, which Wagner wrote pamphlets against.[2]
@DuncanHill: The local bishop would have had more rights over a consecrated church than over an unconsecrated proprietary chapel. I haven't found any recent sources, but A Practical Treatise on the Law Relating to the Church and Clergy (Henry William Cripps, 1886) says as is said by Lord Coke , as the church is a place dedicated and consecrated to the service of God , and is common to all the inhabitants , it therefore belongs to the bishop to order it in such manner as the service of God may best be celebrated on page 400 and has a section on proprietary chapels on pages 153 and 154. TSventon (talk) 19:47, 30 September 2024 (UTC)[reply]

References

  1. ^ Hibbert, Christopher, ed. (1992). "Keble College". The Encyclopaedia of Oxford. Pan Macmillan. pp. 206–208. ISBN 0-333-48614-5.
  2. ^ Yates, Nigel (2004). "Oxford DNB article: Wagner, Arthur Douglas". Oxford Dictionary of National Biography (online ed.). Oxford University Press. doi:10.1093/ref:odnb/41252. (Subscription or UK public library membership required.)

Why did we stop integrating art in public spaces?

[edit]

So in historical artifacts and buildings you see a deep interlinking of art and function, bridges, light poles and buildings are brimming with art. Why did we heavily reduce this? My guess is that business contributed to art as a pr move and with the advent of the printing press it stopped making economic sense. What do you think? Bastard Soap (talk) 13:25, 28 September 2024 (UTC)[reply]

We didn't. Nanonic (talk) 14:06, 28 September 2024 (UTC)[reply]
And where have you been for the last 12 years? ←Baseball Bugs What's up, Doc? carrots15:24, 28 September 2024 (UTC)[reply]
Ornament_(art)#History says "The history of art in many cultures shows a series of wave-like trends where the level of ornament used increases over a period ... [list of historical increases and decreases] ... to be decisively reduced by the Arts and Crafts movement and then Modernism." Fashion, then, probably explains why we no longer (currently) have intricate decoration on the inward-facing plates of door locks or the insides of door hinges, and this carries over in things like street light poles and bridge railings.  Card Zero  (talk) 18:16, 28 September 2024 (UTC)[reply]
Bauhaus and Brutalist architecture both mention a reduction in decoration. -- Verbarson  talkedits 21:59, 28 September 2024 (UTC)[reply]
The question is… do we have less “art” in public design, or simply a different form of “art”? Blueboar (talk) 22:41, 28 September 2024 (UTC)[reply]
Honestly it seems obvious that we reduced prioritising art in public spaces Bastard Soap (talk) 10:59, 29 September 2024 (UTC)[reply]
Personal observations can be flawed. ←Baseball Bugs What's up, Doc? carrots13:22, 29 September 2024 (UTC)[reply]
You haven't brought up any stats Bastard Soap (talk) 20:42, 29 September 2024 (UTC)[reply]
Nor have you, and you're the one making the claim. ←Baseball Bugs What's up, Doc? carrots08:06, 30 September 2024 (UTC)[reply]
What? Why do you think the advent of printing had anything to do with this? -- asilvering (talk) 20:47, 29 September 2024 (UTC)[reply]
In Britain, outdoor advertising was based on hoardings (billboards): England 1835, by John Orlando Parry
Billboard#History has a reference for flyposting in the late 15th century, reasonably hot on the heels of moveable type. Beyond that, the lag in moving to full-blown advertising is mysterious, but advances in printing must be relevant. History_of_advertising#16th–19th_centuries says "Advances in printing allowed retailers and manufacturers to print handbills and trade cards. For example, Jonathon Holder, a London haberdasher in the 1670s, gave every customer a printed list of his stock with the prices affixed. At the time, Holder's innovation was seen as a 'dangerous practice' and an unnecessary expense for retailers." But further down the page there's this nice picture of public artwork from 1835. Giant version here, because I couldn't read it all properly in our version.
 Card Zero  (talk) 22:33, 29 September 2024 (UTC)[reply]
I don't think that PR is an adequate explanation. Consider Crossness Pumping Station (built 1859-1865) by local government in London. It wasn't a private business trying to drum up income, because it had a monopoly on everybody's sewage, and it didn't need PR because London was desperate to get rid of the stuff. It wasn't even a public building (in the sense that members of the public needed to visit it). Yet it was decorated on the outside, and crazy decorated inside.
I suggest that such decoration takes many skilled person-hours, and that as labour became more expensive, the cost of decoration became prohibitive.[citation needed] -- Verbarson  talkedits 10:42, 30 September 2024 (UTC)[reply]
This article, Ornament and Decoration, says that the Modernist movement of the first years of the 20th-century rejected ornamentation in architecture and other fields, taking the example of Viennese architect Adolf Loos and his 1908 essay, Ornament and Crime:
Adolf Loos campaigned to strip the ornament from language, from dress, and from dwelling. “I have freed mankind from superfluous ornament,” he bragged. “‘Ornament’ was once the synonym for ‘beauty’. Today, thanks to my life’s work, it is a synonym for ‘inferior’.” Espousing a middle-class ethos of functionalism, economic rationality, impersonality, and restraint, modernists redirected investment from luxury expenditures to factories, sanitary facilities, and municipal infrastructures. In place of individual expression they advocated standardized solutions, naked structures, white walls, and crisp geometric forms.
Alansplodge (talk) 11:38, 30 September 2024 (UTC)[reply]
I wondered what the "crime" was. His article says:

"the evolution of culture is synonymous with the removal of ornamentation from objects of everyday use." It was therefore a crime to force craftsmen or builders to waste their time on ornamentation that served to hasten the time when an object would become obsolete (design theory). Loos's stripped-down buildings influenced the minimal massing of modern architecture, and stirred controversy.

I have some questions about this.
  • Does therefore really belong? It would make sense in the opposite direction, rational efficient building is removing ornament -> evolution of culture is removing ornament, but doesn't seem to follow the other way round, as presented.
  • Does, or did, ornament function as planned obsolescence?
  • This word "massing" ... is that a technical architectural term? Or a bad translation from German? Or both? And what does it mean? "Covered in masses"?
 Card Zero  (talk) 12:46, 30 September 2024 (UTC)[reply]
I question his premises. If ornamentation really causes obsolescence (by adversely affecting the function of an object) it must therefore be more than mere decoration (which by definition is not functional). The only way I can understand ornamentation causing 'obsolescence' is by going out of fashion. The decoration of Tower Bridge is well out of fashion, but that does not make the bridge obsolete.
Note Sheffield Town Hall, built in the 1890s, and decorated per the contemporary fashion. A Brutalist extension was added in 1977. Guess which bit was demolished in 2002? -- Verbarson  talkedits 14:59, 30 September 2024 (UTC)[reply]
I couldn't guess with certainty, since Brutalism has its fans and protectors due to its historical interest (reminiscent of the scene in Futurama where there is a concert of classical hip-hop, and how "modern art" is now over 100 years old). Besides, out-of-date ornament may have caused buildings to look offensive in the past, before the notion of "heritage". Certainly in Georgian England there was great destruction of Tudor architecture because everything had to be "improved", meaning neoclassical or approximately Parisian.  Card Zero  (talk) 16:04, 30 September 2024 (UTC)[reply]

I'm looking for a picture of this person. You'd think someone with a school and a prize named after them shouldn't be that difficult, but I'm having no luck. Gråbergs Gråa Sång (talk) 13:45, 28 September 2024 (UTC)[reply]

Gråbergs Gråa Sång I looked in Google books and found a small image here in Ebony May 1984. TSventon (talk) 13:52, 28 September 2024 (UTC)[reply]
@TSventon Fantastic, thanks! Gråbergs Gråa Sång (talk) 13:59, 28 September 2024 (UTC)[reply]

Bitcoin price rigging

[edit]

Wikipedia:Reference_desk/Mathematics#Bitcoin_price_rigging I am told this may be in the wrong forum. 2604:3D08:5E7A:6A00:D94:3638:168B:18A0 (talk) 22:49, 28 September 2024 (UTC)[reply]


September 29

[edit]

Women kidnapped to harems in the 1950s

[edit]
I read a story online in which a Greek woman in the 1950s was almost tricked to being trafficked to a harem in the Arabian Peninsula, after answering an job advertisement in a newspaper. I remember hearding similar stories when I read about white slavery.
Certain athentic cases of European women dissapearing in the Muslim world, such as Gunnel Gummeson, have been speculated to be victims of such kidnappings.
I wonder: are there actual historic cases when European women where known to be kidnapped to harems in that time period? And how probable was it?
Some people have called sutch stories propaganda. But it is factual that Africa women where kidnapped to become slave concubines in harems in the Gulf in that time period (slavery in Saudi Arabia was still legal). So if African women where subjected to this fate, why not European woman? Are there known cases? Thank you --Aciram (talk) 00:01, 29 September 2024 (UTC)[reply]
Circassian sex slaves were much in demand in the Ottoman Empire, which until 1916 included the western region of modern-day Saudi Arabia containing Mecca and Medina. There is no reason to think this ended when slavery became illegal. Quoting from Sexual slavery § Present day, Asia:
"The Trafficking in Persons Report of 2007 from the US Department of State says that sexual slavery exists in the Persian Gulf, where women and children may be trafficked from the post-Soviet states, Eastern Europe, Far East, Africa, South Asia or other parts Middle East.[203][204][205]"
 --Lambiam 09:48, 29 September 2024 (UTC)[reply]
Yes, I realise it is logical and reasonable to assume that there where such cases. Chattel slavery was indeed both legal and in full practice in most Gulf states in the 1950s.
But I am interested in the particular time period of the decades around the 1950s: before the fall of the Societ Union, when modern sex trafficking from Eastern Europe became rampant. Where there such cases in the Interwar period, and the 1950s? It is that particular time period I am interested in. --Aciram (talk) 21:53, 29 September 2024 (UTC)[reply]
There is every reason to think that Ottoman Empire slavery ended when the Ottoman Empire ended. And in the cited modern source (it's misleadingly 3 citations to the same state department report), simply listing countries means nothing -- working through them, you'll see most countries are tier 2 and below, and it seems all will be listed as 2 or more of source, transit, and destination for trafficking. I'm not disputing the problem of trafficking -- I'm asserting that your statements are unsupported.
As to the OP's question of whether European sex trafficking still occurs by force/abduction/kidnapping, it's relatively easy to find individual nightmare cases: The Guardian 2011-02-06 (Romania-to-UK), Vice 2013-04-28 (Bulgaria-to-Italy). More broadly, I found an old UNODC report "Trafficking in Persons to Europe for sexual exploitation": on p.3 it summarizes the notion of coercion (with citations to studies), where as you may expect the majority of victims have come willingly under a range of expectations, but "they may nonetheless end up in exploitative situations through deception, coercion or violence." This de facto sex slavery condition may be something like what you've heard reports of happening to West African migrants in the Gulf. SamuelRiv (talk) 18:51, 29 September 2024 (UTC)[reply]
I don't understand what you mean when you say "There is every reason to think that Ottoman Empire slavery ended when the Ottoman Empire ended", since legal chattel slavery in Saudi Arabia and Yemen ended in 1962, slavery in Kuwait in 1949, slavery in Dubai in 1963, and slavery in Oman in 1970 - and it is well documented that all of these countries certainly still had chattel slaves until the very year of legal emancipation (I have studied that issue).
However, my specific question is: are there known cases when European women where abducted to be used for sexual slavery (slave concubinage being legal) in harems on the Arabian Peninsula in the 1950s? This was a particular time period: prior to the fall of the Soviet East Communist Block, when sex trafficking became rampant. --Aciram (talk) 21:53, 29 September 2024 (UTC)[reply]
I assume the point is that slaves in Saudi Arabia, Yemen, Kuwait, Dubai and Oman were not slaves in the Ottoman Empire after it ended since even if they were part of the Ottoman Empire before, they no longer were. Even slaves in Turkey would not be slaves in the Ottoman Empire. More generally, the slave trade would likely have been significantly affected by the fall of the empire. New routes would likely need to have been developed, and sources may not have been so willing to provide slaves to lesser powers. (Remember this was before any of them became rich and powerful via oil money, I mean a number of them weren't even the modern day states that they are now at the time.) Also the end of the Ottoman Empire didn't happen in a vacuum, WW1 and other related events would likely have significantly affected the trade even of the empire had survived. So while clearly slavery didn't end, it's likely it was quite different from what it was before. Nil Einne (talk) 16:11, 30 September 2024 (UTC)[reply]
I have studied the issue, and the slave trade and use of slaves where not much affected in the Arabian Peninsula by the fall of the Ottoman Empire. Regardless, that is irrelevant to the question of the post: is it confirmed that European women where trafficked to the harems in the Arabian Peninsula in the 1950s or around that time? --Aciram (talk) 16:57, 30 September 2024 (UTC)[reply]

First theatres in England

[edit]

Hello. fr.wikipedia says on that "Le 29 juin 1572, une première ordonnance du Parlement, l'Act for the Punishment of Vagabonds, impose que chaque troupe de comédiens soit sous le patronage d'un noble ou de deux édiles" ["On June 29, 1572, a first ordinance of Parliament, the Act for the Punishment of Vagabonds, required that each troupe of actors be under the patronage of a nobleman or two aediles"] but en.wikipedia write on that "the Mayor and Corporation of London first banned plays in 1572 as a measure against the plague". These two statements are said to be the origin of the birth of theatres in London. Can you tell me which one is correct or give me more information? Already thanks, Égoïté (talk) 19:01, 29 September 2024 (UTC) - sorry for my bad english ![reply]

Probably both are correct.
The Act of Parliament would have applied throughout England and Wales, and governed existing (and future) acting companies, which might have travelled around the country performing in public, and/or performed at private houses of rich patrons, or had a fixed venue (see for example Red Lion (theatre)).
The ban on performances by the authorities in London (followed by their expulsion of 'players' entirely in 1575 – see also The Theatre#History) applied to the City of London only, which occupied (as it still does) an area of about one square mile or so on the north bank of the Thames. These measures prompted theatre companies to move to, and build theatres in, the district of Southwark on the south bank of the Thames (across London Bridge) where the City of London had no authority. {The poster formerly known as 87.81.230.195} 94.6.86.81 (talk) 19:34, 29 September 2024 (UTC)[reply]
We have an article Vagabonds Act 1572, unfortunately it doesn't mention players. DuncanHill (talk) 20:49, 29 September 2024 (UTC)[reply]
DuncanHill, it does now (using this ref). Alansplodge (talk) 13:55, 30 September 2024 (UTC)[reply]

Many thanks for your answers. Égoïté (talk) 18:47, 1 October 2024 (UTC)[reply]

September 30

[edit]

Dalit hindu rape victim

[edit]

I was trying to remember the name of that Dalit/lower caste Hindu rape victim who was from a movie (not in English). She became a MP and was assassinated over legal case. What was her name? Maybe she was Buddhist since she was from near Nepal. Sportsnut24 (talk) 00:59, 30 September 2024 (UTC)[reply]

Phoolan Devi? (She was the top search result when I put "india bandit queen" into Google...) -- AnonMoos (talk) 03:35, 30 September 2024 (UTC)[reply]
Yes, sounds right. Thanks.Sportsnut24 (talk) 12:48, 30 September 2024 (UTC)[reply]

Business terms relating to surprise album

[edit]

I'm uncertain whether business terms product marketing, loyalty marketing, and word-of-mouth marketing are related to surprise album. Regardless, I'm seeking business terms relating to a surprise album, which has little or no prior announcement, marketing or promotion. George Ho (talk) 03:08, 30 September 2024 (UTC)[reply]

Assistance with interpreting scope and manner of a UN event

[edit]

Hello, I see new draft at Wikinews, sister of Wikipedia, about a ceasefire call: n:France, US push for 21-day Hezbollah-Israel ceasefire in Lebanon. I have difficulty understanding structure of the UN organisation or its events. Please view the talk page of the article and assist at your earliest convenience? Thank you in advance. Gryllida (talk, e-mail) 06:25, 30 September 2024 (UTC)[reply]

You will need to take that up with Wikinews. We can only help you here with Wikipedia issues. Shantavira|feed me 08:33, 30 September 2024 (UTC)[reply]
Seeing as Wikinews is created by contributors, I think for practical purposes this person is Wikinews.  Card Zero  (talk) 12:08, 30 September 2024 (UTC)[reply]
We can help with research needed to answer questions arising anywhere, including at other Wikimedia projects.  --Lambiam 12:36, 30 September 2024 (UTC)[reply]
BBC News item: US and allies call for 21-day ceasefire ... "The 12-strong bloc proposed an immediate 21-day pause in fighting" ... "The joint statement was signed" ... "It followed a meeting of world leaders at the UN General Assembly in New York".  Card Zero  (talk) 12:05, 30 September 2024 (UTC)[reply]
Hi, I appreciate the lookup. It was a Statement signed, yes. How and where was it delivered to the Israel and Hezbollah representatives? Gryllida (talk, e-mail) 13:32, 30 September 2024 (UTC)[reply]
I don't know. The article has various hints, such as "the US is negotiating with Lebanon’s government - rather than Hezbollah." I gather you're interested in the "Official responses are expected within hours" part?  Card Zero  (talk) 14:14, 30 September 2024 (UTC)[reply]
I was unable to find anything specific about any presentation to the Israelis, but the statement was drafted and signed at the UN General Assembly, so I imagine that the easiest method would be to hand it to the Permanent Representative of Israel to the United Nations. Alansplodge (talk) 14:25, 30 September 2024 (UTC)[reply]
Also note that Benjamin Netanyahu was present at the UN at the time, so the proposal could have easily been handed over to him. Xuxl (talk) 18:30, 30 September 2024 (UTC)[reply]

Use of fish killed by depth charges

[edit]

this is kind of a weird one, but during the WWII Battle of the Atlantic, are there any known instances of navy sailors collecting and eating some or all of the fish that were killed by depth charges they dropped?

TheAbigail (talk) 13:09, 30 September 2024 (UTC)[reply]

Members of the crew of HMAS Doomba with fish taken on board killed or stunned after a depth charge attack.
Members of the crew of HMAS Doomba with fish taken on board killed or stunned after a depth charge attack. HMAS Doomba in her role as escort and anti-submarine vessel would sweep the harbour approaches with her ASDIC before escorting a convoy to sea and attack any threatening ASDIC returns with depth charges.
Note that once at sea with a convoy, stopping for any reason would leave an escort vessel vulnerable to attack and the convoy's merchant ships unescorted. From 1941, there were convoy rescue ships which saved escorts from having to stop to pick up survivors, so I imagine that stopping to catch stunned fish would be highly unlikely. Alansplodge (talk) 13:33, 30 September 2024 (UTC)[reply]

Parents' Sabbath?

[edit]

Why was a Russian Orthodox Hymn at Queen Elizabeth II’s Funeral? about the Kontakion of the Departed says that it is sung in Russian Orthodox churches on "Parents’ Sabbath, a day of special remembrance for Orthodox Christians who have died". Is there a Russian Wikipedia article that relates to this. A Google search didn't find much. Alansplodge (talk) 16:49, 30 September 2024 (UTC)[reply]

The search term "Кондак усопших" does not turn up any results from the Russian Wikipedia. The kontakion is mentioned in this Russian news article on the funeral service for Prince Philip, which also provides an answer to the "why" question — allegedly because Philip wanted to emphasize his kinship with the Romanovs. The Russian term for Parents’ Sabbath is Родительская суббота, which is more adequately translated as "Parental Saturday", of which there are several in any given year. The Russian Wikipedia has an article on Parental Saturdays, which is skimpy on the liturgy and does not mention any songs.  --Lambiam 06:05, 1 October 2024 (UTC)[reply]
Okay, many thanks for your work. I'll put in a link to that article. The Kontakion of the Departed has a long history in British royal funerals, and I suspect it might have been used even if Philip hadn't had Orthodox roots (his mother, Princess Alice of Battenberg, was an Orthodox nun; I think the Romanov link is rather tenuous but useful to Russian nationalists). Alansplodge (talk) 13:17, 1 October 2024 (UTC)[reply]

Bloody codes

[edit]

The article London Monster says

"Magistrates charged Williams with defacing clothing[Note 1]—a crime that in the Bloody Code carried a harsher penalty than assault or attempted murder."

However Bloody_Code says:

"Leon Radzinowicz listed 49 pages of "Capital Statutes of the Eighteenth Century" divided into 21 categories:[13]

  • Stabbing, maiming and shooting at any person"

Which is correct (or are they both?)

All the best: Rich Farmbrough 21:25, 30 September 2024 (UTC).[reply]

I don't know the answer, but the two statements are not at odds with each other. Theoretically (given just these two statements), the penalty for an attempt to strangle a person could have been a slap on the wrist, provided that the clothing of the victim was not defaced.  --Lambiam 06:14, 1 October 2024 (UTC)[reply]
This article (footnote 48 on page 19) says that "attempted murder" was not legally defined until Lord Ellenborough's Act (Malicious Shooting or Stabbing Act 1803). Alansplodge (talk) 13:57, 1 October 2024 (UTC)[reply]

October 1

[edit]

Language

[edit]

September 17

[edit]

Spanish diphthongs

[edit]

In Spanish, a high vowel (i or u) is normally pronounced as a semivowel (/j/ or /w/) when before another vowel. For example, seria is pronounced /ˈse.rja/. If there is an acute accent, then the vowels form a hiatus and first vowel is stressed, like in sería /se.ˈri.a/. But are there any words where second vowel of hiatus is stressed or both vowels of hiatus are unstressed, like /se.ri.ˈa/ or /ˈse.ri.a/? Is there a way to indicate them in spelling? --40bus (talk) 13:19, 17 September 2024 (UTC)[reply]

I don't speak Spanish and I don't really understand their use of acute accents to indicate the stress. But in Portuguese you can have a word like aula (class) and saúde (health), in which there is a hiatus between the vowels and the second is stressed. And of course, in a word like glória (glory) neither of the final vowels is stressed. 2A02:C7B:223:9900:A88D:8EE5:E75B:3C1A (talk) 16:03, 17 September 2024 (UTC)[reply]
One way to indicate a hiatus might be a silent h between the vowels. —Tamfang (talk) 03:00, 20 September 2024 (UTC)[reply]
Apparently (per English Wiktionary) there is no hiatus in nihonio (nihonium), which thus ends up with two identical syllables. Double sharp (talk) 04:57, 20 September 2024 (UTC)[reply]
The Spanish Wikipedia article es:Diptongo says that an "h" between vowels, although it does not produce a sound, does not impede the formation of a diphthong. --Amble (talk) 05:00, 20 September 2024 (UTC)[reply]
The Wikt pronunciation is automated, so it could be irregular and no-one's noticed to correct it. For niobio, the translit says it's ['njobjo], but the recording has [ni'obio]. Don't know if the speaker being natively bilingual (Spanish-Catalan) has anything to do with anything. — kwami (talk) 06:15, 20 September 2024 (UTC)[reply]
I am not sure about your technical terms but palabras con diptongo says:
Ahora bien, como ya se ha explicado (v. § 3.2j), el sistema de acentuación gráfica del español no tiene como función indicar si una secuencia vocálica se articula en una sola sílaba o en sílabas distintas (prueba de ello es que no distingue gráficamente va.ria.do de res.fri.a.do ni cui.da de hu.i.da, por ejemplo), de forma que la duplicidad gráfica en estos casos carece de justificación y constituye un elemento disgregador de la unidad de representación gráfica del español, cuyo mantenimiento es función esencial de la ortografía. Por ello, a partir de este momento, la convención que establece qué secuencias vocálicas se consideran diptongos, triptongos o hiatos a efectos ortográficos debe aplicarse sin excepciones y, en consecuencia, las palabras antes mencionadas se escribirán obligatoriamente sin tilde, sin que resulten admisibles, como establecía la Ortografía de 1999, las grafías con tilde.
So I understand that the latest version of Spanish orthography is not in the business of distinguishing diphthongs and hiatuses in pronunciation, even if both are present among speakers.
The 1999 version allowed both "guion" and "guión" according to the pronunciation of the speaker. Not anymore. "Guion" for everybody. I think Arturo Pérez-Reverte, although, an academic proclaimed to rebel against this decision.
--Error (talk) 12:51, 20 September 2024 (UTC)[reply]

Identification of Subject based on Chinese text

[edit]

Hi. I'm writing an article on the painter Gao Qifeng, and I was looking through older copies of The Young Companion for free images. Unfortunately, the transliteration system they used does not reflect the modern system (they render his name Kao instead of Gao), and I can't read the original Chinese. I've found two that seem to be his brother, Jianfu (top left, bottom right), based on the glasses. This one may be Qifeng. Would someone who reads Chinese be able to confirm? — Chris Woodrich (talk) 17:53, 17 September 2024 (UTC)[reply]

This website has a very similar photo and gives the name in Chinese characters as 高奇峰. The characters in your link are rather hard to make out and they may be traditional rather than simplified. No, characters 4 and 5 in the caption are qi gao, with feng missing. --Wrongfilter (talk) 19:04, 17 September 2024 (UTC)[reply]
  • Yes, between the scan quality and the magazine being 90 years old, it is hard to read. I do have 高奇峰 in my notes as well, based on ZH-Wiki, so it's good to confirm that. Qi Gao seems to be a confirmation, which is good. (Wish I'd realized they'd written right-to-left... could have caught that). — Chris Woodrich (talk) 19:48, 17 September 2024 (UTC)[reply]
The "feng" character in the magazine caption is written wikt:峯, which seems to be one of the traditional equivalents of simplified wikt:峰 (same visual components, only with the radical on top). The whole caption reads, transposed from right-to-left into left-to-right order: "畫家高奇峯氏近影" (simplified: "画家高奇峰氏近影"), which does seem to translate to what the English above says, "Recent photo of painter Gao Qifeng". Fut.Perf. 20:23, 17 September 2024 (UTC)[reply]

September 19

[edit]

English word to refer to an entity under another one

[edit]

Is there an English word to refer to an entity that is under another one where both of them are not part of each other? Neutralhappy (talk) 19:06, 19 September 2024 (UTC)[reply]

Lower, inferior, underlying, bottom? I may be missing some important aspect of your question, because this seems too easy. Do you want an adjective? Should it exclude the possibility that the two entities are part of the same whole? Should it be limited to two entities?
Notes: lower only specifies having a position with less height, not necessarily underneath the other thing. Inferior is either scientific (as used in anatomy) or has a more common meaning of "less good". Underlying implies the thing is hidden and perhaps supports the thing above. Your best option might be bottom.  Card Zero  (talk) 19:24, 19 September 2024 (UTC)[reply]
Thanks for your reply. What does a "subsidiary" have to do with an organisation as given in this page for the template of an infobox? Does the term "subsidiary" in the page Template:Infobox organization mean a company, or an entity affiliated to the main organisation? Neutralhappy (talk) 21:48, 19 September 2024 (UTC)[reply]
Oh I see, you meant "entity" as in an organisation, and "under" as in "managed by". Context is everything.
Well, in the infobox, subsidiary is listed immediately after "parent organization", and means the opposite. It's wikilinked to subsidiary, which gives the alternate term daughter company. So yes, as you say, a company, or an entity affiliated to the main organisation (and controlled by it).  Card Zero  (talk) 22:04, 19 September 2024 (UTC)[reply]
I could not see the term subsidiary being used to refer to a non-company affiliated organisation, in dictionaries. But the wikilinked article subsidiary only speaks of a company. So can we use the term subsidiary to refer to a non-company affiliated organisation that is under another one? Neutralhappy (talk) 16:46, 20 September 2024 (UTC)[reply]
That statement is difficult to respond to because it's entirely unclear what you mean by a non-company affiliated organization. "Non-company" could mean nonprofit or unincorporated. In general, if a legal entity is formed in some way that it can be owned, then it can be a subsidiary. And if it can't be owned because it's too amorphous and informal, like a unincorporated association, then it can't be a subsidiary and in that situation is merely called an affiliate of another organization. --Coolcaesar (talk) 17:12, 20 September 2024 (UTC)[reply]
Thanks for you reply. Is there a need to add a parameter for "affiliate" in the Template:infobox organization? Or can "subsidiaries" in the infobox be used to show affiliates too? Neutralhappy (talk) 18:54, 20 September 2024 (UTC)[reply]
Put the other organizations under "subsidiaries" if they are subsidiaries, but not under "affiliations". See Nation of Islam for an example. (However, if they are not subsidiaries, put them under "affiliations". See International Union of Painters and Allied Trades. Don't put the same organization under both. Unless that makes good sense somehow.)  Card Zero  (talk) 19:46, 20 September 2024 (UTC)[reply]

September 20

[edit]

Thüringern

[edit]

According to Wiktionary, the names of some German regions derives from the dative plural form of the name of the inhabitants or eponymous tribe. A few examples are:
Angel (Angle, Anglian person): Angeln (the region of Anglia)
Bayer (Bavarian person): Bayern (the region of Bavaria)
Franke (Franconian person): Franken (the region of Franconia)
Hesse (Hessian person): Hessen (the region of Hesse)
Sachse (Saxonian person): Sachsen (the region of Saxony)
Schwabe (Swabian person): Schwaben (the region of Swabia)
Apparently this is not the case for Thuringia:
Thüringer (Thuringian person): Thüringern; actual name: Thüringen (Thuringia).
Is there a reason for it? Are there other examples? Thank you! 195.62.160.60 (talk) 17:56, 20 September 2024 (UTC)[reply]

It could just have been phonetically simplified, due to -Vn being easier to pronounce than -Vrn, I presume. 惑乱 Wakuran (talk) 21:36, 20 September 2024 (UTC)[reply]
Various sources mention an Old German etymon Duringa. An n can have been added in analogy with other region names. BTW, the unstressed ending -ern in Modern German is pronounded almost the same as -en: /-ɐn/ versus /-ən/.  --Lambiam 22:56, 20 September 2024 (UTC)[reply]
Almost, but for speakers of German varieties that have a vocalic reflex of syllable-final + pre-consonantal r, the difference between /ɐ/ versus /ə/ is like day and night, or rather, like blue and green.Austronesier (talk) 09:45, 21 September 2024 (UTC)[reply]
Map of the Thuringian States in 1890
The difference with the other ethnonmys ("tribonyms"?) is that Thüringer uses the derivative suffix -er that makes it a demonym paired with the toponym Thüringen. And there are hundred such pairs -inger ~ -ingen. The use of the suffix -er is a late analogous formation. In Middle High German, the ethnonym is Düringe (plural form), as mentioned in Hugo von Trimberg's Der Renner (next to die Franken, die Sahsen; but note: die Beier, die Oesterrîche also without without n in MHG). Note also the initial d- as the "correct" reflex of early continental West Germanic /θ/ (rendered as th- in medieval Latin documents). The Th- spelling is a learned Latin-based modification of Düringen which was the common spelling before the disintegration of Thuringia in the 16/17th century. –Austronesier (talk) 09:45, 21 September 2024 (UTC)[reply]
Add: The MHG singular was Düring. Based on that form, Düringen perfectly fits the dative plural pattern. It was broken by the late back-formation Thüringer from learned Thüringen. –Austronesier (talk) 10:02, 21 September 2024 (UTC)[reply]
And I suppose it's pronounced with a regular t-sound, nowadays. (Thailand has an aspirated t-sound, but that is probably irrelevant.) 惑乱 Wakuran (talk) 11:02, 21 September 2024 (UTC)[reply]

September 21

[edit]

Translation request: Works and artists from Commons File

[edit]
Works by our artists, The True Record 11 (1912)

Hi. Would it be possible to translate the titles of the works and artists from the page attached here? I haven't had any luck with machine tools.  — Chris Woodrich (talk) 19:43, 21 September 2024 (UTC)[reply]

Crisco 1492, if you have managed to OCR what's there but the subsequent machine translation makes no sense, NB the horizontal script goes not left to right but right to left. (Apologies if this would-be tip insults your intelligence.) -- Hoary (talk) 22:12, 21 September 2024 (UTC)[reply]

September 22

[edit]

Attention theft, Mise-en-scène, and Narrowcasting

[edit]

I see a user added Attention theft, Mise-en-scène, and Narrowcasting to the "see also" section of False dilemma. In what way might these make sense? Are they perhaps all commonly used as metaphors in ways I don't know? On the face of it, attention theft is something adverts do, mise-en-scène is a term from stagecraft and means "environment", and narrowcasting is targeted broadcasting. I suspect some excessive lateral thinking inspired all this - or are some of these terms used in ways that are indeed related to false dilemmas?  Card Zero  (talk) 14:26, 22 September 2024 (UTC)[reply]

That was added in February, by a user who has since been indef'd.[12] You should be safe in expunging those items. ←Baseball Bugs What's up, Doc? carrots18:59, 22 September 2024 (UTC)[reply]
Done. ←Baseball Bugs What's up, Doc? carrots01:52, 23 September 2024 (UTC)[reply]
Ah, thank you kindly.  Card Zero  (talk) 15:13, 23 September 2024 (UTC)[reply]
Resolved

Convergent use

[edit]

There was a mass shooting in Alabama this morning, apparently done using a machine gun. The mayor of Birmingham was quoted saying:[13]

This is not the first occasion, unfortunately, in 2024 where we’ve seen the style of weapons, the number of bullets on the scene, possibly convergent use, etcetera, for automatic weapons being used in our streets,

What does "convergent use" mean here? Web search and wiktionary don't help. 2601:644:8581:75B0:0:0:0:C078 (talk) 20:44, 22 September 2024 (UTC)[reply]

You could maybe try asking him? ←Baseball Bugs What's up, Doc? carrots21:34, 22 September 2024 (UTC)[reply]
The mayor of Birmingham? Yes I'm sure he'd love to take a call from a rando like me, asking something like that. 2601:644:8581:75B0:0:0:0:C078 (talk) 21:45, 22 September 2024 (UTC)[reply]
This source has the mayor saying "possibly converted automatic weapons". I suppose that he may have misspoken, but that this is what he meant to say.  --Lambiam 21:52, 22 September 2024 (UTC)[reply]
That would make sense: semi-automatics converted (illegally) into automatics. ←Baseball Bugs What's up, Doc? carrots21:57, 22 September 2024 (UTC)[reply]
On the audio here, at 3:07, the mayor does say "convergence use", a term he repeats later. A plausible scenario is that the editors of AL.com called the bureau of the mayor for clarification and received this correction in reply. Alternatively, a (corrected) transcript may have been sent to news outlets.  --Lambiam 22:12, 22 September 2024 (UTC)[reply]
In contrast to semiautomatic weapons unethically represented as automatic by sensationalist news broadcasts. —Tamfang (talk) 03:02, 23 September 2024 (UTC)[reply]

Aha, yeah, some kind of garbled machine transcription maybe. That does make sense. I was thrown because I did find a few hits for "convergent use" by web search, but none made sense in this context. Thanks. 2601:644:8581:75B0:0:0:0:C078 (talk) 00:05, 23 September 2024 (UTC)[reply]

In tonight's evening news, the report used the term "converted", which makes sense. ←Baseball Bugs What's up, Doc? carrots01:48, 23 September 2024 (UTC)[reply]
It was not a garbled machine transcription. On the audio of the video I linked to above, the mayor can clearly be heard saying "convergent use".  --Lambiam 12:08, 24 September 2024 (UTC)[reply]

If it's the same mass shooting reported here, it apparently involved multiple people shooting at a single principal victim. That would certainly fit a description as "convergent". I was reminded of the description of military "time on target" weapon fire that I coincidentally read in a war novel just the other day. --142.112.149.110 (talk) 04:11, 28 September 2024 (UTC)[reply]

September 23

[edit]

Charcuterie

[edit]

According to a Wikipedia article, charcuterie is a branch of French cuisine devoted to prepared meat products, such as bacon, ham, sausage, etc. According to another Wikipedia article, lunch meats are precooked or cured meats that are sliced and served cold or hot.

Question: is there a term in English for precooked or cured meats, such as bacon, ham, sausage, etc., in general, regardless of whether they are sliced or made in France? — Kpalion(talk) 13:44, 23 September 2024 (UTC)[reply]

@Kpalion: In a fine example of how the English language is unafraid of adopting words from others, it's charcuterie. Bazza 7 (talk) 14:16, 23 September 2024 (UTC)[reply]
Does it mean that the Charcuterie article is wrong in restricting the term to French cuisine only? — Kpalion(talk) 14:58, 23 September 2024 (UTC)[reply]
It doesn't restrict it; it just doesn't mention it has been incorporated into omnivorous English, whose appetite for foreign words is insatiable. Clarityfiend (talk) 22:31, 23 September 2024 (UTC)[reply]
Interesting — I didn't realize anyone restricted it to just meats. What I tend to think of as a "charcuterie board", often served at outdoor events, has mostly cheeses (a selection of fresh, semi-soft, and sometimes a few aged), dried and fresh fruit, nuts, jams, honeys, and crackers, with maybe just a few little sausagey meaty things. Often taken with little flutes of sparkling wine. Is there a more specific name for this sort? --Trovatore (talk) 23:28, 23 September 2024 (UTC) [reply]
Well, etymologically, char cuit is just Old French for cooked meat. 惑乱 Wakuran (talk) 01:30, 24 September 2024 (UTC)[reply]
That sounds closer to a cheeseboard to me. AlmostReadytoFly (talk) 09:18, 24 September 2024 (UTC)[reply]

Let me put the question differently: would you use the word "charcuterie" (in English) to describe what you see in this picture? — Kpalion(talk) 08:11, 24 September 2024 (UTC)[reply]

I might worry briefly about it being a misnomer on my part, but I would. Remsense ‥  08:13, 24 September 2024 (UTC)[reply]
@Kpalion: It's a good word for that picture, although I can't recall it being used in conversation (in English English). Bazza 7 (talk) 08:50, 24 September 2024 (UTC)[reply]
Are there any alternatives then? — Kpalion(talk) 09:35, 24 September 2024 (UTC)[reply]
Charcuterie is used in conversational English, but can appear affected. OED gives citations back to 1858. The native terms would be cold cuts or cold meats. DuncanHill (talk) 10:07, 24 September 2024 (UTC)[reply]
I would expect "charcuterie" to be used to describe those meats after they had been cut into small portions, and only in the term "charcuterie board" describing a bunch of them along with cheeses and crackers. As is, I would just call them precooked or cured meats. Or DuncanHill's suggestion of cold meats, "cold cuts" would describe them after they had been cut into slices. I definitely find "charcuterie" to be an affected usage for what trays of what appears to be nothing more than a fancy version of Lunchables. --User:Khajidha (talk) (contributions) 13:19, 24 September 2024 (UTC)[reply]
I like "cured meats" best, as it doesn't imply slicing, serving temperature or country of origin. But is it widespread enough to be commonly understood by native speakers of any variety of English? Lunch meat lists several alternative names, but "cured meats" is not one of them. — Kpalion(talk) 13:35, 24 September 2024 (UTC)[reply]
@Kpalion: I think you will struggle to get something which covers all varieties of English. "Lunch meat" is not used in my own English English and may be confused (especially by people of a certain age) with "luncheon meat" (which is not the generic item the luncheon meat article suggests]]).
I agree with your comment about "cured meats". I don't know how widespread the technical term "cured" is; my own experience it that in everyday speech the curing method is more likely to be referred to (e.g. smoked, salted, pickled, etc.). Bazza 7 (talk) 13:51, 24 September 2024 (UTC)[reply]
This page (in Emgland) seems to use "chacuterie" and "cured meats" as synonyms. I believe both terms would be widely understood here. Alansplodge (talk) 14:00, 24 September 2024 (UTC)[reply]
Where does the AMerican term "cold cuts" fit in here? HiLo48 (talk) 00:42, 25 September 2024 (UTC)[reply]
I mentioned "cold cuts" above. The OED defines charcuterie as "Cold cuts of meat, esp. pork, ham, sausages, etc. Also, a shop that sells goods of this kind". DuncanHill (talk) 01:31, 25 September 2024 (UTC)[reply]
And I'm looking for something more generic, yet still recognizable. A term that would cover both cold cuts and a whole grilled sausage (uncut and served hot). So far, "cured meats" seems like the best fit, but would it be understood in America? — Kpalion(talk) 09:07, 25 September 2024 (UTC)[reply]
If it's served hot is it even charcuterie? DuncanHill (talk) 11:09, 25 September 2024 (UTC)[reply]
Well, is it? And if it's not, then what is it? Say, you can buy a link of smoked sausage and then you can either slice it up and serve cold or grill it and serve hot and uncut. Does it only become charcuterie in the first case, at the moment of serving, but not in the other case? Does the term "charcuterie" not apply before serving? — Kpalion(talk) 11:54, 25 September 2024 (UTC)[reply]
@Kpalion: As I hinted above, your quest may be difficult to fulfil. The assumption that there must be a single word is mistaken. It would help if you gave an indication about when or where such a word is needed.
@Alansplodge gave a link above to a specialist retailer which sells "charcuterie". A large supermarket chain, on the other hand, refers to those products as cooked and continental meats; another, more upmarket, uses the same ans adds deli for good measure.
At a personal level, on the other hand, if I buy some smoked sausage from my supermarket's deli counter, slice it and serve it cold, I'd call it "smoked sausage". If I have it whole and hot, then I'd call it "smoked sausage". If I needed to differentiate between the two, I might add "cold" or "hot". The same might apply to bacon, or pork loin, or pastrami, etc. Bazza 7 (talk) 12:21, 25 September 2024 (UTC)[reply]
You said you'd buy your (whole) sausage at a deli counter. So perhaps "deli" is the word I'm looking for? Is it an exact synonym of "charcuterie" or "cured meats", or is there some difference in its scope? — Kpalion(talk) 12:39, 25 September 2024 (UTC)[reply]
See delicatessen. 2A02:C7B:120:C500:65D0:46B7:4AB7:C9BB (talk) 14:25, 25 September 2024 (UTC)[reply]
That seems fairly close, but you'd need to specify deli meats, since typical deli counters and delicatessens also contain other sorts of foods (cheeses, prepared side dishes such as potato salad and slaw, etc.). Deor (talk) 14:29, 25 September 2024 (UTC)[reply]
Although we have delicatessens in Britain, the phrase "deli meats" sounds distinctly American to me, so it depends on the variety of English required. Alansplodge (talk) 08:40, 27 September 2024 (UTC)[reply]

September 24

[edit]

Capricious

[edit]

Wiktionary says this is from a meaning of curly-haired. But a fitting conception of goatishness could have led to a direct borrowing from Latin capra / caprinus at any time. Surely cultural conceptions of goats have more influence on the language than some narrow hair stereotype? Temerarius (talk) 02:52, 24 September 2024 (UTC)[reply]

That's one theory. Here's EO's take on it:[14][15]Baseball Bugs What's up, Doc? carrots05:59, 24 September 2024 (UTC)[reply]
One's hair curling from fear, an etymological theory mentioned in Pianigiani's Vocabolario Etimologico della Lingua Italiana,[16] is not a stereotype. Yet another theory mentioned there relates the term to the Latin verb caperāre, "to wrinkle".  --Lambiam 12:03, 24 September 2024 (UTC)[reply]
Imagine you'd never heard "capricious" before, and you hear it now. First guess would be "goatish," right? That'd be the case for everyone, everywhere, except that little slice of Italian history.
Temerarius (talk) 02:10, 27 September 2024 (UTC)[reply]
Or you might think of the Isle of Capri, which may have the same etymology anyway. ←Baseball Bugs What's up, Doc? carrots02:25, 27 September 2024 (UTC)[reply]
Or someone might think of a Capricorn, said to be pragmatic, organized, risk-averse and afraid of failure.[17]  --Lambiam 08:24, 27 September 2024 (UTC)[reply]
"Capricious" is from "caprice," which derives from French "caprice," which derives from Italian "capriccio," a sudden start or motion or a sudden shiver of horror. The derivation of "capriccio" is disputed. The OED says it is "apparently < capro goat, as if 'the skip or frisk of a goat.'" However, Merriam-Webster says it "likely comes from the Italian capo, meaning “head,” and riccio, the word for “hedgehog.” The implication was that someone who shuddered in fear or horror was said to have a “hedgehog head,” meaning that the person’s hair stood on end like the spines of a hedgehog. The link between a whim and a shiver of horror is notably tenuous, though, and a possible link to Italian capra, meaning “goat,” has also been suggested, given the whimsy goats seem to employ in their gamboling." John M Baker (talk) 17:14, 27 September 2024 (UTC)[reply]
And there was me thinking that it referred to the impulsive, changeable and/or perverse behaviour of residents of Capri. {The poster formerly known as 87.81.230.105} 94.6.86.81 (talk) 13:43, 29 September 2024 (UTC)[reply]

September 25

[edit]

Is this an error of style?

[edit]

This is an excerpt from my version of Harry Potter and the Order of the Phoenix (I have the audiobook read by Stephen Fry): "... as though this was the signal Harry had been waiting for, he jumped to his feet, at the same time pulling from the waist-band of his jeans a thin wooden wand as if he were unsheathing a sword."

Is there a reason why 'was' is used in the first bolded bit and 'were' in the second bolded bit? Would this be considered an error of style? 150.203.2.213 (talk) 06:13, 25 September 2024 (UTC)[reply]

Here's an extensive discussion on was vs. were.[18] However, from reading it, I'm not sure what the answer to your question is. ←Baseball Bugs What's up, Doc? carrots07:10, 25 September 2024 (UTC)[reply]
I guess that the signal could be considered actual, and the sword fictional, if I am to hazard a guess. For what it's worth. 惑乱 Wakuran (talk) 11:01, 25 September 2024 (UTC)[reply]
Ohh okay, I can see how that might motivate the difference. Thank you! 150.203.2.213 (talk) 12:04, 25 September 2024 (UTC)[reply]
Some stylists recommend not repeating a formula in close proximity, except when done to achieve a rhetorical effect.  --Lambiam 13:18, 25 September 2024 (UTC)[reply]
Sure, but this case is akin to using "He and I did ..." in one place, and "Me and him did ..." in another. -- Jack of Oz [pleasantries] 17:48, 26 September 2024 (UTC)[reply]
Not really. The subjunctive is dying out, and it's not reasonable to call every instance of "if I was..." an error. --Viennese Waltz 18:01, 26 September 2024 (UTC)[reply]
It might still feel a bit off to use the subjunctive seemingly randomly. 惑乱 Wakuran (talk) 19:05, 26 September 2024 (UTC)[reply]
The issue that bothered the OP may be the intermingled use of was and were. Had the quoted fragment started with "... as though this were the signal ...", I surmise it would not have engendered the raising of even a single bushy brow.  --Lambiam 08:13, 27 September 2024 (UTC)[reply]


September 29

[edit]

Short audio translation request

[edit]

Japanese clip Zarnivop (talk) 18:06, 29 September 2024 (UTC)[reply]

What's the context? Anime? J-Drama? My Japanese isn't good enough to catch more than snippets, but it sounds rather theatrical. 惑乱 Wakuran (talk) 18:58, 29 September 2024 (UTC)[reply]
It's smoething a soldier says after he scored a kill, taken from a voice pack of a mod. Zarnivop (talk) 19:32, 29 September 2024 (UTC)[reply]
I hear kujikenu kokoro ga asu no shōri ni tsunagaru no, or 「くじけぬ心が明日の勝利につながるの」 (with asu being a short form of ashita, 明日). GalacticShoe (talk) 19:21, 29 September 2024 (UTC)[reply]
From a quick online search, I'm presuming this is from Soulcalibur V. There's a wiki page which apparently has this and other voice lines, if you need to reference them. GalacticShoe (talk) 19:33, 29 September 2024 (UTC)[reply]
So "A resilient spirit will lead to victory tomorrow"? (Google translate) Zarnivop (talk) 19:33, 29 September 2024 (UTC)[reply]
Something along those lines, yup. GalacticShoe (talk) 19:36, 29 September 2024 (UTC)[reply]
A profusion of thanks! Zarnivop (talk) 21:41, 29 September 2024 (UTC)[reply]

September 30

[edit]

Memento Aomori

[edit]

According to our article on Aomori, the original name of this Japanese city was 善知鳥村 Utō-mura. Now, how to you get that pronunciation from those kanji? If I'm not mistaken the last character 村 should be -mura (village). But the rest? Thank you! 95.238.49.112 (talk) 14:56, 30 September 2024 (UTC)[reply]

By morpho-ambulating helplessly through jawiki, it seems it corresponds to an old form of ja:ウトウ, the name for the Rhinoceros auklet, who I deem a cute little guy. Apparently, that's ultimately a loanword from Ainu, neat! Remsense ‥  15:10, 30 September 2024 (UTC)[reply]
I suspect it is a nanori. ColinFine (talk) 17:34, 1 October 2024 (UTC)[reply]

"Turn it up now"

[edit]

In 1950s England, was this phrase:

  • Equivalent to "settle down", or
  • In its French translation, one of the few phrases known to a beginner, for some reason?

I ask because I'm curious about the phrase as it appears at the foot of this page in one of the Nigel Molesworth books.  Card Zero  (talk) 17:52, 30 September 2024 (UTC)[reply]

"Turn it up!" means "stop it", "settle down", "lay off", etc. The French looks like Molesworth's attempt. DuncanHill (talk) 18:31, 30 September 2024 (UTC)[reply]
Oh, so it really does mean that. Interesting in the light of wikt:turn up sense #6 (and the song Turn down for what, which by all reports intended that sense). I see Molesworth's sense of "turn up" is missing from Wiktionary, although the nautical sense 5 (fasten lines down) might be related?  Card Zero  (talk) 19:18, 30 September 2024 (UTC)[reply]
"turn it up", not "turn up". OED has "transitive. slang (chiefly British). To give up, abandon (an activity). Formerly also intransitive: †to throw up or abandon one's work, to give up (obsolete). Now only in imperative as turn it up: used as a warning to desist, esp. from objectionable talk; ‘shut up’, ‘come off it’. DuncanHill (talk) 19:45, 30 September 2024 (UTC)[reply]
Seems counterintuitive. ←Baseball Bugs What's up, Doc? carrots21:10, 30 September 2024 (UTC)[reply]
An idiom (which this is), by definition does not mean what it would if read literally, and has to be learned from its cultural context and use. {The poster formerly known as 87.81.230.195} 94.6.86.81 (talk) 21:35, 30 September 2024 (UTC)[reply]
I just wonder where it came from. Typically, "turn it up" would mean to increase something, not decrease it. A more fitting expression would seem to be "dial it down" or "dial it back". English is weird. ←Baseball Bugs What's up, Doc? carrots22:15, 30 September 2024 (UTC)[reply]
The usage is cited back to 1819, rather earlier than your dials. A related usage is "transitive. To give up, renounce, abandon, cast off, discard (an associate). Now rare (slang in later use)" which goes back to 1541. DuncanHill (talk) 22:34, 30 September 2024 (UTC)[reply]
Yes. My question is why? What is "it" that's being "turned up"? ←Baseball Bugs What's up, Doc? carrots22:59, 30 September 2024 (UTC)[reply]
The "it" is whatever activity the speaker wishes to be ceased. Perhaps it originates from a once familiar activity where 'turning up' was a thing: one possibility that occurs to me is that on a sailing vessel with fore-and-aft rig (like most yachts), one can come to a standstill by turning the bows up to point directly into the wind. (This application of 'up' is still in use, as anyone following the current activities in Barcelona will know.)
I believe there is a technical term for such indeterminate 'its', which I've forgotten. Another example: when we say "It's raining", what exactly is "it"? {The poster formerly known as 87.81.230.195} 94.6.86.81 (talk) 01:47, 1 October 2024 (UTC)[reply]
Maybe you're trying to remember "dummy pronoun"? --Antiquary (talk) 08:38, 1 October 2024 (UTC)[reply]

October 1

[edit]

Entertainment

[edit]

September 20

[edit]

I hope this question is in the correct place?

[edit]

I apologise in advance if this is the wrong place.

I'm thinking of writing an article about a book. The problem is, the publisher, Bonnier Books, does have a small arm of the business that deals with self publishing.

The book is 'Kill the Black One First: A memoir of hope and justice' by Michael Fuller ISBN 978-1-78870-286-7. How can I tell if the book is self published? Obviously, if it was self published then I won't write the article. Knitsey (talk) 16:00, 20 September 2024 (UTC)[reply]

The self-publishing platform Type & Tell closed down in 2017, so this doesn't seem to be a problem. Also the Waterstones page quotes a newspaper review, which is unlikely for something self-published. And I can't find evidence of this, but his article says the book was published on the Blink imprint, and hence not on Type & Tell (which was sort of kept in isolation by being assigned to Bonnier Books Ventures, a separate company from Bonnier Books UK which has the Blink imprint, and separate from the parent Bonnier Books).  Card Zero  (talk) 18:12, 20 September 2024 (UTC)[reply]
I just looked in the flyleaf and it mentions Blink there. I should have double checked, sorry about that. I can't tell from this [19] whether that means it's part of the self publish arm? Thank you so much for taking the time to answer @Card Zero:. Knitsey (talk) 18:38, 20 September 2024 (UTC)[reply]
I looked at all the imprints listed there, and none of them say anything suggesting "self publishing", although this one says "commercial" which sounds like the opposite. (Some of the others don't say "commercial". I don't know what that means.) I also searched the site for the author and the title, and they don't admit that the book ever existed, which is an aggravating disregard for the value of information. But where did you hear that they do self publishing, and was it under the name "Type & Tell"? That is not this, so don't worry, unless you've heard they still do it.  Card Zero  (talk) 18:48, 20 September 2024 (UTC)[reply]
Oh heck, I'm not sure. I googled something like 'is Bonnier Books self publishing' and I think I got the explanation that a small part of the publisher does do self publishing, but mostly it's a regular publisher.
I'm wondering if Blink is something to do with Waterstones? I think it was initially released as exclusive to them?
I think I will search around for some book reviews and see where I get to. I know The Times reviewed it, I don't have a subscription but I can access it through the library.
Thanks so much for your help. Self publishing wasn't really something I had come across before I started editing Wikipedia! Knitsey (talk) 19:01, 20 September 2024 (UTC)[reply]
Worth keeping in mind is that being self-published is an indicator that a book is probably not notable, but that self-published status is not definitive. Fifty Shades of Grey is not my preferred personal style, but it was originally self published and has since gone on to be a genuine best seller with sequel novels and a film series. The Martian was also a self-published novel that was adapted into an acclaimed film starring Matt Damon that was more my style. The thing that matters most is whether the book complies with WP:NBOOK, which usually requires multiple reviews by competent book reviewers published in reliable, independent sources. Cullen328 (talk) 07:37, 22 September 2024 (UTC)[reply]
Hi Cullen328, I didn't know that. I just assumed if it was self published it wouldn't be notable. Fifty Shades didn't really feature on my radar, not my cup of tea, but that's a good example.
I will have a good look around to see if it complies with WP:NBOOK before I start writing anything. Thank you for the info. Knitsey (talk) 12:20, 22 September 2024 (UTC)[reply]
The thing that matters is reliable secondary source coverage. While it's true that a lot of the time a book being self-published means few are going to pay attention to it for a variety of reasons, this isn't guaranteed. I think Cullen328's examples do illustrate that even in the ebook era, it still tends to be advantageous to have a publisher hence why those examples and others like The Rabbit Who Wants to Fall Asleep may have became notable while self-published but were able to get a conventional publisher once they received enough attention. Still for a variety of reasons including simply author's preference, there's nothing stopping a book remaining self-published even after it receives a lot of attention. Articles we have on books which seem to remain self-published include Natural Cures "They" Don't Want You to Know About, Stolen Valor and Canada's Stonehenge. To some extent Dangerous (book) although it was originally not going to be self-published. Nil Einne (talk) 08:33, 24 September 2024 (UTC)[reply]
This has been an eye opener. I think that because self-published sources are generally a no-no for referencing, I had it stuck in my head that self-published books wouldn't be suitable for articles. Which obviously isn't the case if the book in question has enough significant coverage.
I need to get this week over with then I will start looking for references to see if it might be suitable for an article. Thank you everyone for the help. Knitsey (talk) 09:53, 24 September 2024 (UTC)[reply]

September 25

[edit]

German parody

[edit]

I came across a recording here of Max Ehrlich delivering a parody of Alexander Moissi, who I came across during one of my earliest collaborations on WP, Busoni's Turandot Suite: but what is he saying? The info says "Textvorlagen: Johann Wolfgang von Goethe (1749-1832), Friedrich Schiller (1759-1805)", with a link to DNB which also mentions Albert Bassermann and Max Pallenberg, but these might be red herrings. Any ideas? MinorProphet (talk) 02:16, 25 September 2024 (UTC)[reply]

After an introduction he starts with a famous monologue from Schiller's Wilhelm Tell (act 4, scene 3). As far as I can tell the text is original, but the delivery is very exaggerated. I don't have time right now to listen to the rest, maybe tonight. --Wrongfilter (talk) 13:27, 25 September 2024 (UTC)[reply]
The declamation of the monologue ends at 1:15. What follows is a dialogue between Alexander Moissi and Guido Tielscher, who appear very clueless about Schiller's play. For instance, they understand the "Hohle Gasse" (a sunken lane) to be an underground railway, and they respond to Küssnacht, which could be literally translated as "kiss night", by "I'd rather kiss by day". Goethe appears in part 2 (Der König in Thule), and that is indeed a parody of Bassermann and Pallenberg. --Wrongfilter (talk) 14:46, 26 September 2024 (UTC)[reply]
@Wrongfilter: Thanks very much indeed for your knowledgeable insights. Just looking at de:Guido Thielscher's photo makes me laugh, he was obviously possessed of a wicked sense of humour. Thanks also for the link to part 2. Goethe used his "König in Thule" poem in Faust, Part 1. It appears that both Bassermann[20] and Pallenberg[21] appeared as Mephisto in productions of Faust by Max Reinhardt. I recognised Pallenberg's name from Reinhardt's massive 1911 pantomine-spectacle The Miracle (play) (article badly needs updating), but not in the film of the play The Miracle (1912 film) (another of my early collaborations), the world's first full-colour feature film. MinorProphet (talk) 12:26, 27 September 2024 (UTC)[reply]

September 28

[edit]

Examples of violent movie scenes that weren't actually that violent

[edit]

Two examples that spring to mind are the shower scene in Psycho and the chainsaw scene (which also happens in a shower) in Scarface. If you rewatch them, you don't actually see metal entering flesh, but it's edited in such a clever manner that you think that you saw more graphic gore and mutilation on-screen than you did. I've seen discussions online where people swear that they saw something they didn't and that these movies must have been censored for violence after the fact. Amazing filmmaking, I must say. But can anyone tell me some other examples? Iloveparrots (talk) 00:37, 28 September 2024 (UTC)[reply]

Prior to the mid or late 1960s, movies and TV shows with violent scenes seldom had any apparent penetration or any blood. Some guy would fire a gun and the other guy would immediately drop dead, with no blood. Most any western would do for that example. I'm thinking of the scene in "Fort Apache", where the Indians surround Henry Fonda's character and kill him, but with no closeup or slo-mo or anything like that. More recently, I'm thinking of "West Side Story", where the gang members were stabbing each other with switchblades, again with the victims falling over dead immediately after one knife stab but little or no blood. ←Baseball Bugs What's up, Doc? carrots02:13, 28 September 2024 (UTC)[reply]
Also, consider how fistfights were and are filmed, with guys seemingly slugging each other but filmed from an angle that allows the actors to "swing and miss" but to look like they could be making contact. Though there were occasional slipups. In one "Superman" episode, Frank Richards took a swing at Phyllis Coates, who was standing too close to him and he actually knocked her out. Oops! ←Baseball Bugs What's up, Doc? carrots02:19, 28 September 2024 (UTC)[reply]
This is difficult in two ways: what blood and gore actually features in any given movie? (Censorship boards may help with that.) What do people generally misremember happening in it? That one is hard to source. I found 13 horrific moments of implied violence in movies, which is the same sort of idea, although in some cases it's plainly implicit, and in many others (Reservoir Dogs!) the movie has lots of blood in it elsewhere.  Card Zero  (talk) 03:28, 28 September 2024 (UTC)[reply]
There was also 1967's "Bonnie and Clyde", which I haven't seen for a long time, but as I recall it was an early use of slo-mo along with little explosions to depict them being riddled with bullets. In contrast, the old movie "The Big Sleep", as I recall, had the villain run out the door yelling in vain to his men not to shoot. You didn't see any bullets hit him, but they penetrated the door he had closed behind him. ←Baseball Bugs What's up, Doc? carrots11:05, 28 September 2024 (UTC)[reply]
See all 'A-Team Firing' on TVTropes which talks about the trope of everyone shooting but no-one actually dying. 'Bloodless Carnage' where no entrance or exit wounds or even blood is shown. And also 'Non-Lethal Warfare' Nanonic (talk) 12:24, 28 September 2024 (UTC)[reply]
Tarantino explained that violence is more violent if you don't show it. That is why he pans away or shuts a door when he wants something to be extremely unsettling. He was praised by critics by panning away in Reservoir Dogs and shutting the door in Pulp Fiction, but he wanted to explain that it was a trick that was around for decades before he copied it in his films. When it comes down to it, your imagination is far more revolting than anything that can be put on film. 12.116.29.106 (talk) 12:39, 28 September 2024 (UTC)[reply]

October 1

[edit]

Miscellaneous

[edit]


September 17

[edit]

Worldle

[edit]

Why, when i enter "worldle" (no quotes and NOT wordle) into the wikipedia search text box does it say it is a web based number game? It isn't. The article correctly says it is a geography game. -- 2A00:23CC:D222:4701:2124:F3FE:E0E9:CBB (talk) 23:13, 17 September 2024 (UTC)[reply]

It looks correct to me. ←Baseball Bugs What's up, Doc? carrots01:08, 18 September 2024 (UTC)[reply]
The description in the search box is based on the {{Short description}} in the article. I've changed that to say "geography game".-Gadfium (talk) 03:54, 18 September 2024 (UTC)[reply]
I see your edit in the diff, but I don't see it in the article. ←Baseball Bugs What's up, Doc? carrots06:29, 18 September 2024 (UTC)[reply]
It's only visible in the wiki text. If you still see {{Short description|2022 web-based geography game}}, you may need to clear your cache.  --Lambiam 09:34, 18 September 2024 (UTC)[reply]
Thx 2A00:23CC:D222:4701:2124:F3FE:E0E9:CBB (talk) 06:06, 18 September 2024 (UTC)[reply]

September 18

[edit]

Samsung Bespoke AI Laundry Blue Silicone Cup

[edit]

We replaced our old washer/dryer with a Samsung Bespoke AI Laundry unit. It comes with a blue silicone cup. I can't find any mention of it in the installation or user guides. I have searched videos. I found one where the blue cup was sitting on top of the unit, but was never mentioned. I'm trying to figure out what the cup is for. I assume it has something to do with the operation of the unit or it wouldn't be included. But, if that was the case, it would be mentioned somewhere. It isn't even mentioned in the packing list, but I can see that at least one other person received one with their unit. Can anyone find any resource that mentions what the cup is supposed to be used for? 75.136.148.8 (talk) 12:06, 18 September 2024 (UTC)[reply]

Of course, as soon as I ask, Google image search leads me to the answer. It is a Samsung Aqua Pebble. 75.136.148.8 (talk) 12:21, 18 September 2024 (UTC)[reply]
Resolved


September 20

[edit]

Astrology again

[edit]

I did some research and found some statements using weasel words:

  • Some tropical astrologers counter that the signs remember the influence of the constellations that corresponded with them two thousand years ago (Abell, George; Science and the Paranormal: Probing the Existence of the Supernatural, p. 86);
  • Many so-called tropical astrologers are aware of precession but choose to ignore it, arguing that somehow the 'signs remember the influence of the constellations that corresponded with them two thousand years ago (Hines, Terence; Pseudoscience and the Paranormal; quoting Abell but some has become many);
  • Some astrologers have adopted a position that attempts to reconcile the traditional sidereal scheme with the fact of precession by suggesting that the signs remember the influence of the constellations that corresponded to them 2000 years ago (when the traditional system was first established) (Wilson, Fred; The Logic and Methodology od Science and Pseudoscience; p. 57).

Who are these "some/many (tropical) astrologers"?-- Carnby (talk) 10:36, 20 September 2024 (UTC)[reply]

I see the earliest version (on archive.org) is a mimeographed 1979 yearbook of the Association for the Education of Teachers in Science, where the piece is also by (I assume) George O. Abell. Unfortunately he is not contactable for comment, unless we try a medium. It may be necessary to write this off as an unknown source. On the other hand if any astrologer (including a more modern one!) can be found to have trotted out the same idea, Abell's comment becomes valid again to quote as a counterpoint, even without knowing the source of the verbatim quote. (Depending what your project is here, exactly. I assume you're revising a related article?)  Card Zero  (talk) 13:22, 20 September 2024 (UTC)[reply]
(edit conflict) :Astrologers have one argument,[22], astronomers have another.[23]. The arguments are evaluated at astrology and science. 2A00:23C5:E161:9200:D824:D0C6:1E60:F0FC (talk) 13:37, 20 September 2024 (UTC)[reply]
See Sidereal and tropical astrology. As you can infer from the cited references, "tropical astrologers" are mainly found in India. AstroLynx (talk) 14:08, 20 September 2024 (UTC)[reply]
Interesting hint. I looked at some Indian astrology books and found some citations of The Fixed Stars And Constellations In Astrology, by Vivian Robson, which has the advantage of being just over 100 years old and hence fully browsable. I wonder whether it makes an equivalent statement to the quote anywhere. (Having examined it, probably not, but like any of these books, the theory being presented is muddled and hard to pin down. In this case the theory seems to be that the constellations were originally named after the influence of the signs: but also that both signs and constellations, despite now being separated, continue to have influence. Why not!)  Card Zero  (talk) 15:17, 20 September 2024 (UTC)[reply]
(edit conflict) [above comment amended 15:44] The book gives some attributes to named stars and charts alleged effects of their passage through the sun signs. There is also treatment of the effects of constellations as distinct from sun signs. There is a lot of confusion among non-Hindus in their terminology. This appears to stem from the fact that India employs both "lunar" calendars and "solar" calendars (in "scare" quotes) in which the moon plays no part. However, if you read Indian New Year's days and follow the link to Vaisakhi#Date you will see the new year festival is moving forward - (proleptic) Gregorian 5 April in AD 1469 to 29 April in AD 2999. That's 24 days in 1,530 years or a whole year in ((365.2422/24) x 1530) = 23,284 years, which is roughly how long the equinox takes to move from the constellation of Aries (or any constellation) back to the same constellation. 2A00:23C5:E161:9200:54BE:4B3D:E69:3BFA (talk) 16:16, 20 September 2024 (UTC)[reply]
I think it's the other way round though, I'm seeing tropical equated with Western and sidereal with Hindu or Vedic astrology.  Card Zero  (talk) 16:20, 20 September 2024 (UTC)[reply]
I think we can safely assume that authors, whether Hindu or not, know what they're talking about. A glance at some Wikipedia articles reveals that some Wikipedia editors (such as AstroLynx, above) are making this mistake. What is causing it? Well, the Indian national calendar is aligned with the Gregorian, so the New Year always falls on 21 March, but the lunar calendars may be aligned with the "sidereal" as opposed to "tropical" (sun sign) calendar, so that the adheek (thirteenth) month is added when two new (or full) moons fall within one sidereal (as opposed to tropical, "sun sign") month. AstroLynx's claim that '"tropical astrologers" are mainly found in India' may mean no more than that, of the population of India who are astrologers, a goodly proportion live within the tropics. 92.28.94.86 (talk) 18:47, 20 September 2024 (UTC)[reply]
Here I think tropical is from Greek (wikt:τροπή) and means "turning". Or so one of the authors says.  Card Zero  (talk) 18:54, 20 September 2024 (UTC)[reply]
That's perfectly true. The Greek word tropos means "turn", and relates to the fact that when it reaches the northern tropic (Cancer) the sun turns and begins moving south, and when it reaches the southern tropic (Capricorn) it turns and begins moving north. 92.28.94.86 (talk) 19:08, 20 September 2024 (UTC)[reply]
As was noted above, I indeed made an error, "tropical astrologers" who account for the precession of the equinoxes are mainly Western, "sidereal astrologers" who neglect precession are mainly Hindu/Indian. AstroLynx (talk) 19:46, 20 September 2024 (UTC)[reply]
In the terms our article uses, "Sidereal astrology accounts for the Earth's axial precession and maintains the alignment between signs and constellations ... whereas tropical astrology ... does not take axial precession into consideration." So I'm afraid it's the other other way round. Confusingly, the tropical system doesn't turn. Except by staying fixed relative to the very slowly turning stars.  Card Zero  (talk) 21:15, 20 September 2024 (UTC)[reply]

September 21

[edit]

Little big curiosity

[edit]

Hello. I turn to Wikipedia so that it can come to my rescue let's say, can help me better understand the object of my curiosity. Now, with the understanding that the thing itself has no value as obvious, the image in the link depicts Arizona's fake electors casting their “votes” for Trump in 2020. If you look at the image, I even tried to enlarge it but could not understand much, to cast this fake vote these “electors” had to write down the candidate's name? It looked like this to me. Granted that this is a curiosity for its own sake, but still a curiosity, I would like to understand more if possible and I rely on the good heart You Users. Thank you very much. https://eu.azcentral.com/story/news/politics/arizona/2022/01/22/how-arizonas-trump-electors-planned-deliver-him-victory/6604574001/ Andreoto (talk) 13:20, 21 September 2024 (UTC)[reply]

Arizona's electoral votes look [like this]. If you are looking at handwritten votes, it is just garbage. Ignore it. As with the question above, states usually place all votes in one certified letter to Congress. 75.136.148.8 (talk) 15:15, 21 September 2024 (UTC)[reply]
By "handwritten votes” are you referring to the image in the link i.e. the fake electors who cast their votes by writing the name?151.44.148.9 (talk) 18:57, 21 September 2024 (UTC)[reply]
Are you referring to the article that is hidden behind a paywall? 75.136.148.8 (talk) 01:36, 22 September 2024 (UTC)[reply]
Yes. Exactly. 93.65.152.244 (talk) 03:00, 22 September 2024 (UTC)[reply]

Passports

[edit]

Today, I went to the post office for a passport renewal. I’ve noticed that you have to take an oath. Then, I found out that you’re supposed to take one when you get your passport the first time. Do you have to take an oath every time you renew your passport? TWOrantulaTM (enter the web) 17:59, 21 September 2024 (UTC)[reply]

What country? "Fuck me, how much?" is the traditional British oath. DuncanHill (talk) 18:01, 21 September 2024 (UTC)[reply]
That depends entirely on which country you are a citizen of, and that country's laws and regulations. Cullen328 (talk) 18:03, 21 September 2024 (UTC)[reply]
The OP says he's American, so American federal law and/or policies would apply. ←Baseball Bugs What's up, Doc? carrots22:03, 21 September 2024 (UTC)[reply]
As a US citizen living outside the USA I find that my passport needs to be renewed every 10 years at a US embassy or consulate that will require personal attendance, showing the expiring passport and a renewal fee in cash. It seems they regard the oath I gave over half a century ago in another country as persistently binding so I have never been asked to repeat it. The requirement may have been different for the OP who renewed at a post office and not at an embassy. Philvoids (talk) 11:05, 22 September 2024 (UTC)[reply]
Currently renewing my passport in Australia. No oathing involved, except at the sloppily designed website I am forced to use. HiLo48 (talk) 11:32, 22 September 2024 (UTC)[reply]

This [24] says

Before a passport is issued to any person by or under authority of the United States such person shall subscribe to and submit a written application which shall contain a true recital of each and every matter of fact which may be required by law or by any rules authorized by law to be stated as a prerequisite to the issuance of any such passport. If the applicant has not previously been issued a United States passport, the application shall be duly verified by his oath before a person authorized and empowered by the Secretary of State to administer oaths.

which suggests the in person oath only needs be before the first passport so you could probably make a fuss and refuse an oath after your first passport if you really wanted to. That said I cannot find any info on what this actual oath is or even if it's generally administered on most first applications. I assume it's just to confirm that the information in your application is true etc.

(Most stuff related to US passports and oaths seem to refer to the Oath of Allegiance (United States) which is required for naturalisation not a passport. Someone who isn't a US citizen would need to take such an oath before they can be naturalised which of course means they need to do it before they can get a passport. But that's a somewhat different thing and isn't part of the passport process per se. Instead it's the certificate of naturalisation which is required for the passport.)

That said various embassy sources talking about a first passport do mention an appointment [25] or interview [26] so there's definitely room for an oath. Interesting enough, the US code suggests that someone issued a passport when they were a baby would theoretically not need to make an oath, at most only their parents. (Although the NZ one suggests the process might be the same one for anyone applying for the first time once over the age of 16 as someone who's never had a passport.)

Nil Einne (talk) 07:43, 24 September 2024 (UTC)[reply]

Oh, this is just swearing that you've told the truth on your application? That's completely different from what it sounded like in the thread.
My guess is it would be an "oath or affirmation"; I've never heard of any time you had to "swear" for an official US government reason that you didn't have the opportunity to instead "affirm". Presumably this was originally meant to accommodate those who took a strict view of Matthew 5:34. --Trovatore (talk) 22:29, 24 September 2024 (UTC)[reply]
Neither of my passports required oaths. I'm a citizen of the UK and one other Commonwealth country. Citizenship requires an oath or pledge (or one by your parents if you are getting your citizenship only via them). Passports require citizenship, but no separate oath. Komonzia (talk) 18:53, 22 September 2024 (UTC)[reply]
The current PDF form to apply to renew a passport is DS-82 04-2022. Above where the applicant will sign is a statement:

I declare under penalty of perjury all of the following: 1) I am a citizen or non-citizen national of the United States and have not performed any of the acts listed under "Acts or Conditions" on page 4 of the instructions of this application form (unless explanatory statement is attached); 2) the statements made on the application are true and correct; 3) I have not knowingly and willfully made false statements or included false documents in support of this application; 4) the photograph submitted with this application is a genuine, current photograph of me; and 5) I have read and understood the warning on page 4 of the instructions to the application form.

But adults renewing passports usually do so by mail, and that is the intended purpose of DS-82. Perhaps the original poster used a different form because, for some reason, the application had to be filed at a post office.
This, of course, isn't exactly an oath, but I expect the penalty for falsehoods would be similar.
Jc3s5h (talk) 22:51, 24 September 2024 (UTC)[reply]

What is the name of this fallacy?

[edit]

What is the name of this fallacy?

Weed is illegal
Cigar is legal and its worse than weed.
Both are drugs.
So both must be legal.

The thing is that he picked a side (weed must be legal for being less harmfull then a legal drug), but the other side can be picked (cigar must be illegal for being more harmfull than a illegal drug).2804:1B3:9700:B438:DC3:8910:3C:AA15 (talk) 20:39, 21 September 2024 (UTC)[reply]

There's implied premises (from being in the shared society etc). Let me write this as a more formal chain of logic -- I am using the active because you used it, as in "[generally all] A are B", but you could also do this with something like "[generally all] A should be B".) Begin with the definitions (you can draw Venn diagrams to follow along):
  • Cigar is a type of drug; weed is a type of drug. -- define "drugs" as our universal set in this exercise, cigars and weed will be individual elements.
  • Some drugs are good and some drugs are bad mmkay? -- This is me saying that "drugs" is an ordered set, with a quantity like "good-for-you"ness, in which drugs that are "bad" are below some threshold number. So for this example, we'll say the "good-for-you" scale is from -10 to 10, and drugs rated -1 or below are "bad", while those 0 or higher are "not bad". All drugs have a rating and must be either bad or not bad (The Law of the excluded middle, so the set of "bad-drugs" joined with the set of "not-bad-drugs" makes up our universal set of all "drugs", with no overlap.)
Now let's lay out our premises in this abstract, formal argument, beginning with the ones you noted:
  • cigar is legal.
  • weed is illegal.
  • Cigar is worse than weed. -- (worse = a drug that is more bad, per the "good-for-you" quantity we defined. So in my example scale in my note, if weed rates 2 in badness (which by my definition is not a "bad drug"), then this premise means that cigar must be 1 or lower.)
Now I will add a final implied premise, that of society's law, which you'll see will be critical if we want to do anything useful:
  • All bad-drugs are illegal ("not legal"). -- (Just as with "bad" and "not bad" drugs, all drugs must be either legal or illegal; this premise states that the "bad drugs" set lies entirely within the "not legal" set in your Venn diagram.)
(Note also that neither you nor I ever specify in our premises whether weed or cigar a "bad drug" or "not-bad-drug".)
Given all these premises, can we now draw conclusions that stem from the legality and illegality of cigar and weed? We know from my final society's-law premise that if cigar were a "bad drug", it would automatically be illegal. (However, observe that if weed were a "not-bad drug" it would not imply that it were legal or illegal.) So we can make this syllogism (based on the Law of the Excluded Middle from our strict definitions of bad/not bad drugs and legal/illegal -- remember Venn diagrams if you get lost):
  1. Cigar is a legal-drug;
  2. All bad-drugs are illegal;
  3. Therefore cigar is a not-bad-drug.
So now if you think about our "good-for-you"ness quantity scale again cigar being "not bad" means it rates 0--10, and if it's worse than weed then weed must rate 1--10, so it also must be "not bad":
  1. Cigar is a not-bad-drug;
  2. Cigar is worse than weed;
  3. Therefore weed is not-bad-drug.
BUT, what does weed being a "NOT-bad-drug" tell you about whether it is legal or illegal, going all the way back to my society's-law premise in the beginning?
To try to conclude something of the legality of weed in this simple constructed example is a type of formal fallacy, although I'm not sure what the specific name is. SamuelRiv (talk) 22:04, 21 September 2024 (UTC)[reply]
And your example of reversing the argument can be done by changing my "implied in society's law" premise above to its inverse: instead of "bad drugs are illegal", if one says "good (not-bad) drugs are legal" as your implied premise, then you can do the same argument (which will imply weed is bad because it is not legal) to lead to a similar nonconclusion about whether cigars should be legal or illegal.
However, if you use both of those implied premises of society, that both "bad drugs are illegal" and also "not-bad drugs are legal", then these two will imply "cigar is not-bad" and "weed is bad" respectively, and thus "cigar is worse than weed" creates a logical contradiction. If you use your Venn diagrams (every thing is sets) and think about the excluded middle again, however, you'll see why using both premises is a contradiction from the very beginning. SamuelRiv (talk) 22:28, 21 September 2024 (UTC)[reply]
And if you use neither of my implied premises of society, you can't do anything with formal logic with just the premises laid. The reason for this is that there are no other premises that show the connection between "bad drugs" and "legal drugs" -- what relationship they have with each other. If you can think of any other premise that includes both "bad/good drugs" and "legal/illegal drugs" in it (or two premises that share a third property), you can choose that instead and then you should be able to follow a logical argument again. SamuelRiv (talk) 15:03, 22 September 2024 (UTC)[reply]
(ec)Every sentence in that argument is flawed. One flaw is the statement that marijuana is illegal. The degree of its legality is defined by various state and federal laws. Another is the opinion that cigars are "worse than" marijuana. Another is defining them as "drugs", which would be decided by law, not by the author's personal opinion. And another is the assertion that both "must be legal". Clearly, the legality of each item has been defined by law. Even forgetting that there's no statement of which country this applies to. I might say the fallacy is "starting with dubious assumptions to reach a dubious conclusion." ←Baseball Bugs What's up, Doc? carrots22:10, 21 September 2024 (UTC)[reply]
The validity of a given syllogism concerns the question whether, given that the premises are valid, the conclusion follows logically from these premises. So the following syllogism is valid: P1: The Moon is made of green cheese. P2: Some Moon rocks are stored in Houston Texas. C: Therefore, some green cheese is stored in Houston Texas. Scientists generally believe that P1 is false, which has been confirmed by elaborate testing of the Moon rocks brought back by the Apollo 11 mission and other missions. (I doubt this included tasting them, though.) But this does not invalidate the argument itself. A classical syllogism has only two premises, but I think we can apply the same criterion to an argument with more than two premises. The cigar–weed argument of the OP is IMO defective in at least two respects. The first one is that there is an implicit, omitted premise, namely: For a substance to be illegal, it must be worse than all legal substances. (I interpret "must" here as "ought to", an obligation of lawmakers, not a logical necessity.) Granting this premise, though, as pointed out by others above, the conclusion still does not follow. The premises combined allow a world in which cigars "must" be illegal and weed remains illegal. All this does not answer the OP's question for a name to this type of defect.  --Lambiam 07:46, 22 September 2024 (UTC)[reply]
I haven't thought it through very carefully, but I assume it's just Jumping to conclusions, something we should all be wary of.
I can construct a simpler equivalent statement, if that helps: South Street, Kent is north of North Street, Kent, therefore both are in the south. Some concepts involved are logical consistency and point of view. Maybe begging the question? I don't think anybody mentioned the third possibility, which that weed should be legal but cigars illegal (South Street might be in the north while North Street is in the south).  Card Zero  (talk) 12:01, 22 September 2024 (UTC)[reply]
The premises do not contain material that can be used for reasoning about the deontic status of the legality of weed. If our knowledge of the world is that cigars should be illegal and weed is illegal, this includes possible worlds in which weed should be legal. The premises also allow for another possibility no one mentioned, to wit that cheese is illegal but should be legal.  --Lambiam 19:22, 22 September 2024 (UTC)[reply]
Well, the involvement of morality is accidental, and purely decorative. We're instructed to assume that cigars (or the variable named cigar) is "worse" than weed, and it's implicit that this is the same as "more harmfull" and that items more harmfull than an unknown threshold are labelled "must be illegal". So given these conditions, and the four possible combinations of legal/illegal against cigar and weed, three of those combinations are allowed, and all we know for sure is that the combination cigar is legal and weed must be illegal is ruled out.  Card Zero  (talk) 15:38, 23 September 2024 (UTC)[reply]
Looking online, it seems like most people only bother with naming the informal fallacies. I'd guess that since any invalid syllogism (that's deductive non-probabilistic(?)) can be reduced to a contradiction or tautology, it seems to be just called a formal fallacy, non sequitur, or invalid argument (see e.g. IEP).
Of course, the notion that one can apply formal categorical deduction to the inherently metaphorical and fluid structure that is natural language is itself a fallacy (which oddly I can't find a name for in our stupidly long list of informal fallacies -- the closest I can come up with is the ad-hoc-named (as they all are) noncentral fallacy, which gets the essential point across.)
The reason I gave OP a detailed formal analysis is because OP asked for a formal analysis of the logic in their question. The actual person they were talking about will almost certainly not change a single thing about their opinion once one points out the logical fallacy in their specific wording -- they might just slightly change their wording future, however. SamuelRiv (talk) 15:35, 22 September 2024 (UTC)[reply]
The only question by the OP that I see is "What is the name of this fallacy? ". I see no mention of an actual person having an opinion.  --Lambiam 19:31, 22 September 2024 (UTC)[reply]
As has been pointed out, the only way to check whether something is legal is to check the law. As Baseball Bugs puts it, The degree of its legality is defined by various state and federal laws. The next thing to check is whether the lawyer who is quoting the law to you has a "Conflict of Interest". Lord Falconer certainly did when he claimed that "custom" associated with royal marriages was stated by the Marriage Act to remain unchanged. What the Act actually says is that the law relating to royal marriages remains unchanged - Falconer wasn't speaking as an impartial High Court Judge but as a member of Tony Blair's government. These pronouncements say more about how gullible the government thinks the population is than about the actual law. This section has been so beautifully argued that one wishes that the bald claim in Wedding of Prince Charles and Camilla Parker Bowles "that a civil marriage would in fact be valid" could be dissected in the same manner. Most people are fair and reasonable, but whoever wrote in this morning's News International

They are working for themselves. They are not working for the Royal Family. They are hanging onto their royal titles and people are seeing them for what they are.

clearly is not. So let's give Harry a warm welcome when he comes for the 2024 WellChild awards tomorrow week. An invitation was recently extended to readers to apply to the High Court to view the papers in the case against Camilla Parker Bowles (case no. AC-24-LON-002619). Has anyone done this? I think we will all be interested to know how the case is progressing. 82.32.75.206 (talk) 18:30, 22 September 2024 (UTC)[reply]

This has no relevance whatsoever to the question, which is not about whether something is legal, or how to find out whether something is legal, but about the name of a fallacy. Or are you are proposing to name it the "Wedding of Prince Charles and Camilla Parker Bowles fallacy"?  --Lambiam 19:37, 22 September 2024 (UTC)[reply]
Lambiam, I've struck the tirade. It's just WP:LTA/VXFC riding one of their favorite hobby horses. Feel free to follow WP:BMB and remove their utterings on sight. Favonian (talk) 18:40, 23 September 2024 (UTC)[reply]

What I was talking about (I am the op) is that he is using the fact that weed is also a drug, and a "even worse drug" (cigar) is legal to say weed is illegal. But you could use the same argument saying that weed is a drug that is illegal and cigar is also a drug and "is even worse" so cigars must be legal.
Basically....
1-Right now A work like X
2-Something similar B works like Y.
Just those 2 arguments alone dont say anything if this types of things need to work like X or like Y. The guy pick a side without giving any information why and tell everything must work like that.

The fixed version of the argument without the fallacy I want the name would be one of those:

1-Cigar is legal and should be legal because of A, B and C.
2-Cirgar is worse than weed.
3-Weed is illegal
4-So weed must be legal.

or (if the guy was pro cigar being illegal)

1-Weed is illegal and must be illegal because of A, B and C.
2-Cigar is legal.
3-Cigar is worse than weed.
4-So cigar must be illegal.
— Preceding unsigned comment added by 2804:1B3:9700:B438:1943:8952:7B61:5EB0 (talk) 20:21, 22 September 2024 (UTC)[reply]

In the first of the two, did you mean to write, "4-So weed must be legal."?
If A, B and C argue that weed is a pretty bad substance and that such substances must be illegal, there is no fallacy.  --Lambiam 21:25, 22 September 2024 (UTC)[reply]
Yes I meant to write that. Corrected. 2804:1B3:9700:B438:165:DA1C:DC58:2EB5 (talk) 21:39, 22 September 2024 (UTC)[reply]
That's what 2804:1B3:9700:B438:DC3:8910:3C:AA15 said originally. ←Baseball Bugs What's up, Doc? carrots22:00, 22 September 2024 (UTC)[reply]
If you look at my answer above -- and I had to clean it up a bit -- it does explain how all this works in a syllogism. As I outline, you need to specify the implied premise that makes the relationship between legality of drugs and badness of drugs, and then you can proceed deductively. Your instinct about the logic is correct -- it is a formal deductive fallacy -- and as say in my later comment, they're usually all called the same thing.
The switch that you do between your first set of arguments and the second set in this comment, is what I outlined in my responses above by using the logical inverse of "society's" implicit premise: "bad drugs are illegal" inverts to "not-bad (or good) drugs are legal", and then you can follow the different premises for deduction to again come to a contradiction. SamuelRiv (talk) 22:35, 22 September 2024 (UTC)[reply]
It's hilarious how people get so worked up about something they apparently know very little about, or pretend not to. Acid? Mitsubishis/AK-47s/Rolls Royces? Pub grub/high-quality coke? Shrooms (Liberty caps/Mexicana)? Mephedrone? K? DMT? Squidgy black? Massive nugs of hilarious, revelatory skunk? Tried them all, and I still enjoy raving to techno into the dawn with the ghost of Aldous Huxley. Actually, I once tried reading the opening chapter of The Doors of Perception quietly to myself in the reception area of a police station whilst on probation: and a burly copper appeared and stood silently at the front door, ready to turn people away, while doors banged very loudly and ominously throughout the building. It's as if people were aware of what I was thinking. Most strange. MinorProphet (talk) 21:57, 24 September 2024 (UTC)[reply]
PS Please do not feed the trolls. MinorProphet (talk) 21:57, 24 September 2024 (UTC)[reply]
PPS The devil smokes cigars, the gods smoke weed. MinorProphet (talk) 21:57, 24 September 2024 (UTC)[reply]
If you take the first three as being "true", the fourth thing does not follow logically. Why? Because legislators do not necessarily use logic to arrive at their conclusions. ←Baseball Bugs What's up, Doc? carrots22:38, 24 September 2024 (UTC)[reply]

September 24

[edit]

Download as PDF not working

[edit]

Hi everyone. The "Download as PDF" is not working. You are formidable, and your work is great, thank you for that. 105.100.190.117 (talk) 10:32, 24 September 2024 (UTC)[reply]

It is working for me now. In future, should you encounter a technical problem, Wikipedia:Village pump (technical) is the best go-to place.  --Lambiam 11:28, 24 September 2024 (UTC)[reply]

September 28

[edit]

Previous voting mode of the electoral College of the state of Georgia

[edit]

As of 2020, the system has changed. But until 2016, how was the vote cast? Did one have to write the names of the candidates (president and vice president) on the ballot? Thank you. Andreoto (talk) 08:43, 28 September 2024 (UTC)[reply]

I've noticed a few of these questions. Federal documents are publicly avaialble. This is the 2016 electoral vote for Georgia. In each question, assuming you are the one asking over and over, you ask if they write down the names. All they did here was sign a shared document that was hand delivered to Congress. 12.116.29.106 (talk) 12:44, 28 September 2024 (UTC)[reply]
Yes, it was always me. I'm an “enthusiast” of the American Electoral College, in this case I'm intrigued by the way the Electors cast their votes. Those are the certificates, of course, but first they vote obviously in two separate ballots. Voting can be done in different ways, the mode changes from state to state, but you always vote on ballots. Voting can consist of putting a signature, writing a name, checking a box etc. That's what I meant. Thank you very much for your helpfulness and responses. — Preceding unsigned comment added by Andreoto (talkcontribs) 18:36, 28 September 2024 (UTC)[reply]
I can give second-hand account of what it is like to vote as an elector. My father was an elector in 1984 for Missouri because he was high up in the machinists union and they publicly backed Reagan in both 1980 and 1984. He drove down to Jefferson City. The group met in a conference room in the Capitol Building. They were sworn in and then a bunch of hoopla. Then, they sat at a long table. A stack of pre-printed papers were passed down. Each elector had a box of pens. They used a separate pen for each paper. It was the same document, a vote for Reagan and Bush. He said there were about 20 copies of it total. Then, they were asked to sign a small card with just their name, not everyone's name on it. That was given to them with a pair of pens as proof they were electors. There were snacks and they were sent on their way. At no time did he secretly write down names on a stub of paper and slip it into a secret ballot. All he did was sign his name on a pre-printed collective vote over and over. He didn't go to Washington DC. He didn't vote there. Missouri sent at least one of the collectively signed documents to Federal Congress to announce their electoral votes. My opinion is that Missouri is not weird in how they did it. I expect most states operate in a very similar manner. 12.116.29.106 (talk) 16:38, 30 September 2024 (UTC)[reply]
Thank you so much for your testimony and for sharing this family memory with me. I read you with pleasure. Thank you again for your answers which I found very helpful. — Preceding unsigned comment added by Andreoto (talkcontribs) 18:24, 30 September 2024 (UTC)[reply]

October 1

[edit]

Figure at Swedish National Museum

[edit]

At the Swedish National Museum in Östermalm, Stockholm, Sweden there is a small clay figure at a display case. It's an adult male figure, wearing a suit and tie, with thick black eyeglasses, reading an opened book on its hands. I have a picture of it but have not uploaded it to Wikipedia or Commons. Does anyone know whom this is a figure of? JIP | Talk 00:11, 1 October 2024 (UTC)[reply]

I think you will find the museum people very helpful at info@nationalmuseum.se phone +46(0)8-519 543 00 . Specifically you can ask their Image Services at images@nationalmuseum.se, visiting address: Holmamiralens väg 2 (by appointment only). Philvoids (talk) 11:08, 1 October 2024 (UTC)[reply]
You don't need to upload it to commons, you can upload it to a free image hosting service like imgur and post the link to the image here. --Viennese Waltz 11:50, 1 October 2024 (UTC)[reply]
imgbb doesn't even require any signup.  Card Zero  (talk) 12:31, 1 October 2024 (UTC)[reply]
That's good to know. I've never uploaded images (it's years since I used a camera) but I believe Commons doesn't require signup either. There's a form to be filled in, so you can be certain you're not violating any copyrights. And the description can be updated, useful for future reference. 2A02:C7B:10B:4800:CB6:FD0D:1A20:91AF (talk) 13:04, 1 October 2024 (UTC)[reply]

Usage of hello

[edit]

In a telephone conversation between two officers of an Armed Force of differing seniority, is it correct on the part of the junior officer to respond to the call by saying "Hello"? What is the correct way of beginning the telephonic conversation by the junior officer? Sumalsn (talk) 07:15, 1 October 2024 (UTC)[reply]

How one starts a telephone conversation in a military context depends on whether one is making the call or responding to a call, whether there is an operation in progress and whether the line is secure. The word "Hello" itself conveys nothing but it would be correct to use in "Hello can you hear me?" During a predefined operation there may be routine calls that merely announce "Observation post reporting, no enemy sighting." An unexpected call might be initiated with "Hello this is General Threestar's office with new orders to General Twostar" to which the response might be either "Hallo this is Twostar, what the blazes is going on?" or "Hello the general is unavailable, shall I connect you to Lieutenant Standin?". Philvoids (talk) 11:00, 1 October 2024 (UTC)[reply]
Our protocol was to answer the phone with: (Name of Unit) (Name of building) (Your rank and last name) speaking. How may I help you? 12.116.29.106 (talk) 11:04, 1 October 2024 (UTC)[reply]
Should the same protocol apply to the incoming call "All your base are belong to us, you have no chance to survive make your time"? Philvoids (talk) 11:19, 1 October 2024 (UTC)[reply]
How would the junior officer know a senior officer is calling him? (unless he has all of the Armed Force in his contacts list) FWIW we said "Good morning. This is Major Gump." 196.50.199.218 (talk) 12:07, 1 October 2024 (UTC)[reply]
Caller ID? ←Baseball Bugs What's up, Doc? carrots16:41, 1 October 2024 (UTC)[reply]
This all assumes that the soldiers speak American/English. They may say something completely different. -- SGBailey (talk) 21:09, 1 October 2024 (UTC)[reply]